Pediatrics Practice Questions

Réussis tes devoirs et examens dès maintenant avec Quizwiz!

UTI age spec

...

development surveilance

...

enema abn vs norm

...

health maintenance

...

health promotion

...

herschbaums

...

intussception

...

nephritis s&s tx

...

Normal digoxin level (Peds)

0.5 to 2 ng/ mL Digoxin toxicity is present when level is greater than 2 ng/ mL.

Antibiotics are prescribed for a child with otitis media who underwent a myringotomy with insertion of tympanostomy tubes. The nurse provides discharge instructions to the parents regarding the administration of the antibiotics. Which statement, if made by the parents , indicates that they understood the instructions? 1. "Administer the antibiotics until they are gone." 2. "Administer the antibiotics if the child has a fever." 3. "Administer the antibiotics until the child feels better." 4. "Begin to taper the antibiotics after 3 days of a full course."

1. "Administer the antibiotics until they are gone." A myringotomy is the insertion of tympanoplasty tubes into the middle ear to promote drainage of purulent middle ear fluid and to equalize pressure and keep the ear aerated. The nurse must instruct parents regarding the administration of antibiotics . Antibiotics need to be taken as prescribed, and the full course needs to be completed. Options 2, 3, and 4 are incorrect. Antibiotics are not tapered, but are administered for the full course of therapy.

The nurse is teaching a parent how to administer antibiotics at home to a toddler with acute otitis media. Which statement by the parent indicates that teaching has been successful? 1. "I'll give the antibiotics for the full 10-day course of treatment." 2. "I'll give the antibiotics until my child's ear pain is gone." 3. "Whenever my child is cranky or pulls on an ear, I'll give a dose of antibiotics." 4. "If the ear pain is gone, there's no need to see the physician for another examination of the ears."

1. "I'll give the antibiotics for the full 10-day course of treatment." Antibiotics must be given for the full course of therapy, even if the child feels well; otherwise, the infection won't be eradicated. Antibiotics should be taken at prescribed intervals to maintain blood levels and not as needed for pain. A reexamination at the end of the course of antibiotics is necessary to confirm that the infection is resolved.

After the birth of her first neonate, a mother asks the nurse about the reddened areas ("stork bites") at the nape of the neonate's neck. How should the nurse respond? 1. "They're normal and will disappear as the baby's skin thickens." 2. "They're a common congenital abnormality." 3. "They commonly result from a traumatic delivery." 4. "They're caused by a blockage in the apocrine glands."

1. "They're normal and will disappear as the baby's skin thickens." Capillary hemangioma ("stork bites") may appear on the neonate's upper eyelids, the bridge of the nose, or the nape of the neck. They result from vascular congestion and disappear as the skin thickens. They aren't associated with congenital abnormalities, traumatic delivery, or blocked apocrine glands.

The mother of a hospitalized 3-year-old girl expresses concern because her daughter is wetting the bed. What should the nurse tell her? 1. "It's common for a child to exhibit regressive behavior when anxious or stressed." 2. "Your child is probably angry about being hospitalized. This is her way of acting out." 3. "Don't worry. It's common for a 3-year-old child to not be fully toilet-trained." 4. "The nurses probably haven't been answering the call button soon enough. They will try to respond more quickly."

1. "It's common for a child to exhibit regressive behavior when anxious or stressed." Young children commonly demonstrate regressive behavior when anxious, under stress, or in a strange environment. While the child could be deliberately wetting the bed out of anger, her behavior most likely isn't under voluntary control. It's appropriate to expect a 3-year-old child to be toilet-trained, but it isn't appropriate to expect the child to be able to utilize a call button to summon the nurse.

Which statement indicates that a family of a dying 4-year-old may be ready to consider organ donation? 1. "My wife and I feel that our real daughter has moved on even though her body is still functioning." 2. "Those physicians aren't doing everything they can for our daughter. I know she's still in there." 3. "When will our daughter wake up and be with us?" 4. "How can some parents allow their children to be cut up like a piece of meat and given away?"

1. "My wife and I feel that our real daughter has moved on even though her body is still functioning." Statements indicating that the family has accepted the grave condition of their child is a green light for approaching them about organ donation. Statements that represent the family's nonacceptance of the child's prognosis, the lack of understanding of treatments that are being given, or the misunderstanding of organ and tissue donation are indications that the family isn't ready to be approached or to make a decision.

A 10-year-old child visits the pediatrician's office for his annual physical examination. When the nurse asks how he's doing, he becomes quiet and states that his grandmother died last week. Which statements by the client show that he understands the concept of death? 1. "Once you die you never come back" 2. "All people must die." 3. "My grandmother's death has been hard to understand." 4. "My grandmother died because she was sick and nothing could make her better." 5. "My grandmother is dead, but she'll come back." 6. "My grandmother died because someone in the family did something bad."

1. "Once you die you never come back" 3. "My grandmother's death has been hard to understand." 4. "My grandmother died because she was sick and nothing could make her better." By age 10, most children know that death is irreversible and final. However, a child may still have difficulty understanding the specific death of a loved one. School-age children should be able to identify cause and effect relationships, such as when a terminal illness causes someone to die. Adolescents, not school-age children, understand that death is a universal process. Preschoolers see death as temporary and may think of death as a punishment.

A boy, age 2, is diagnosed with hemophilia, an X-linked recessive disorder. His parents and newborn sister are healthy. The nurse explains how the gene for hemophilia is transmitted. Which statement by the father indicates an understanding of X-linked recessive disorders? 1. "Our newborn daughter may be a carrier of the trait." 2. "If we have more sons, all of them will have hemophilia." 3. "All of our offspring will carry the trait for hemophilia." 4. "Our daughter will develop hemophilia when she gets older."

1. "Our newborn daughter may be a carrier of the trait." X-linked recessive genes behave like other recessive genes. A normal dominant gene hides the effects of an abnormal recessive gene. However, the gene is expressed primarily in male offspring because it's located on the X chromosome. Male offspring of a carrier mother and an unaffected father have a 50% chance of expressing the trait. These parents may produce normal offspring who neither express nor carry the trait for hemophilia. Female offspring typically don't express the trait, although they may carry it.

The parents of a child with cystic fibrosis ask the nurse why their child must receive supplemental pancreatic enzymes. Which response by the nurse would be most appropriate? 1. "Pancreatic enzymes promote absorption of nutrients and fat." 2. "Pancreatic enzymes promote adequate rest." 3. "Pancreatic enzymes prevent intestinal mucus accumulation." 4. "Pancreatic enzymes help prevent meconium ileus."

1. "Pancreatic enzymes promote absorption of nutrients and fat." Pancreatic enzymes are given to a child with cystic fibrosis to aid fat and protein digestion. They don't promote rest or prevent mucus accumulation or meconium ileus.

A nursing instructor asks a nursing student to describe the formal operations stage of Piaget's cognitive developmental theory. The appropriate response by the nursing student is: 1. "The child has the ability to think abstractly." 2. "The child begins to understand the environment." 3. "The child is able to classify, order, and sort facts." 4. "The child learns to think in terms of past, present, and future."

1. "The child has the ability to think abstractly." In the formal operations stage, the child has the ability to think abstractly and logically. Option 2 identifies the sensorimotor stage. Option 3 identifies the concrete operational stage. Option 4 identifies the preoperational stage.

A child, age 10, is hospitalized for treatment of acute osteomyelitis. After assessing swelling and tenderness of the left tibia, the nurse initiates antibiotic therapy as prescribed. The child's left leg is immobilized in a splint. What is an appropriate expected outcome for this child? 1. "The client will change position with minimal discomfort." 2. "The client will bear weight on the affected limb." 3. "The client will ambulate with crutches." 4. "The client will participate in age-appropriate activities."

1. "The client will change position with minimal discomfort." To prevent pressure sores, the child must turn and change positions periodically. However, during the acute phase of osteomyelitis, moving the affected leg may cause extreme pain and discomfort. Therefore, the nurse must support and handle the leg gently during turning and repositioning. Weight bearing is contraindicated because it may cause pathologic fractures. Ambulating with crutches is an inappropriate outcome because the child is restricted to bed rest and the affected leg is immobilized to limit the spread of infection. Participation in age-appropriate activities isn't a realistic outcome because an acutely ill child isn't likely to be interested in activities; this outcome would be suitable after the acute disease phase ends.

PreOp for tonsillectomy

1. Assess for signs of active infection. 2. Assess bleeding and clotting studies because the throat is vascular. 3. Prepare the child for a sore throat postoperatively, and inform the child that he or she will need to drink liquids. 4. Assess for any loose teeth to decrease the risk of aspiration during surgery.

The mother of a 4-year-old child calls the clinic nurse and expresses concern because the child has been masturbating . Using Freud's psychosexual stages of development, the appropriate response by the nurse is which of the following? 1. "This is a normal behavior at this age." 2. "Children usually begin this behavior at age 8 years." 3. "This is not normal behavior, and the child should be seen by the physician." 4. "The child is very young to begin this behavior and should be brought to the clinic."

1. "This is a normal behavior at this age." According to Freud's psychosexual stages of development, between the ages of 3 and 6 the child is in the phallic stage. At this time, the child devotes much energy to examining his or her genitalia, masturbating, and expressing interest in sexual concerns. Therefore options 2, 3, and 4 are incorrect.

A 2-year-old child is treated in the emergency room for a burn to the chest and abdomen. The child sustained the burn by grabbing a cup of hot coffee that was left on the kitchen counter. The nurse reviews safety principles with the parents before discharge. Which statement by the parents indicates an understanding of measures to provide safety in the home? 1. "We will be sure not to leave hot liquids unattended." 2. "I guess my children need to understand what the word hot means." 3. "We will be sure that the children stay in their rooms when we work in the kitchen." 4. "We will install a safety gate as soon as we get home so the children cannot get into the kitchen."

1. "We will be sure not to leave hot liquids unattended." Toddlers, with their increased mobility and development of motor skills, can reach hot water or hot objects placed on counters and stoves and can reach open fires or stove burners above their eye level. The nurse should encourage parents to remain in the kitchen when preparing a meal, use the back burners on the stove, and turn pot handles inward and toward the middle of the stove. Hot liquids should never be left unattended or within the child's reach, and the toddler should always be supervised. The statements in options 2, 3, and 4 do not indicate an understanding of the principles of safety.

A child, age 4, who has been hospitalized with asthma is to receive oral theophylline at home. Before discharge, the nurse teaches the parents about signs and symptoms of theophylline toxicity and the need to report these promptly. Which statement by the parents indicates effective teaching? 1. "We will report irritability and restlessness." 2. "We will report a slow pulse and lethargy." 3. "We will report a rash and swelling of the hands and feet." 4. "We will report coughing and wheezing."

1. "We will report irritability and restlessness." Theophylline causes bronchodilation by promoting adrenergic activation. Signs and symptoms of theophylline toxicity reflect adrenergic stimulation and include irritability, restlessness, tachycardia, and insomnia. Theophylline also may cause such adverse GI effects as nausea, vomiting, diarrhea, and epigastric pain. Rash and swelling of the hands and feet aren't associated with theophylline toxicity. Coughing and wheezing are signs of asthma, not theophylline toxicity.

The nurse is teaching parents about the nutritional needs of their full-term infant, age 2 months, who's breast-feeding. Which response shows that the parents understand their infant's dietary needs? 1. "We won't start any new foods now." 2. "We'll start the baby on skim milk." 3. "We'll introduce cereal into the diet now." 4. "We should add new fruits to the diet one at a time."

1. "We won't start any new foods now." Because breast milk provides all the nutrients a full-term infant needs for the first 6 months, the parents shouldn't introduce new foods into the infant's diet at this point. They shouldn't provide skim milk because it doesn't have sufficient fat for infant growth. The parents also shouldn't provide solid foods, such as cereal and fruit, before age 6 months because an infant's GI tract doesn't tolerate them well.

A mother brings her 8-month-old son to the pediatrician's office. When the nurse approaches to measure the child's vital signs, he clings to his mother tightly and starts to cry. The mother says, "He used to smile at everyone. I don't know why he's acting this way." Which response by the nurse would help the mother understand her child's behavior? 1. "Your baby's behavior indicates stranger anxiety, which is common at his age." 2. "Children who behave that way are developing shy personalities." 3. "Children at his age begin to fear pain." 4. "Your baby's having a temper tantrum, which is common at his age."

1. "Your baby's behavior indicates stranger anxiety, which is common at his age." Stranger anxiety, common in infants ages 6 to 8 months, may cause the child to cry, cling to the caregiver, and turn away from strangers. Typically, it occurs when the child starts to differentiate familiar and unfamiliar people. The child's behavior doesn't necessarily indicate shyness. According to Piaget, fear of pain characterizes the operational stage of development in school-age children, not infants. Temper tantrums are typical in toddlers who are trying to assert their independence; during a temper tantrum, children may kick, scream, hold their breath, or throw themselves onto the floor.

An infant, age 6 months, is brought to the clinic for a well-baby visit. The mother reports that the infant weighed 7 lb (3.2 kg) at birth. Based on the nurse's knowledge of infant weight gain, which current weight would be within the normal range for this infant? 1. 14 lb (6.4 kg) 2. 21 lb (9.5 kg) 3. 10.5 lb (4.8 kg) 4. 17.5 lb (7.9 kg)

1. 14 lb (6.4 kg) Birth weight typically doubles by age 6 months and triples by age 12 months. Therefore, an infant who weighed 7 lb (3.2 kg) at birth should weigh 14 lb (6.4 kg) at age 6 months.

When assessing the chest of a 4-month-old infant, the nurse identifies which ratio of the anteroposterior-to-lateral diameter as normal? 1. 1:1 2. 1:3 3. 2:1 4. 3:1

1. 1:1 In an infant, the anteroposterior diameter normally equals the lateral diameter. In a toddler, the anteroposterior diameter should be less than the lateral diameter.

What should be the initial bolus of crystalloid fluid replacement for a pediatric client in shock? 1. 20 ml/kg 2. 10 ml/kg 3. 30 ml/kg 4. 15 ml/kg

1. 20 ml/kg Fluid volume replacement must be calculated to the child's weight to avoid overhydration. Initial fluid bolus is administered at 20 ml/kg, followed by another 20 ml/kg bolus if there is no improvement in fluid status.

A toddler develops acute otitis media and is prescribed cefpodoxime proxetil (Vantin). What is the usual pediatric dosage for this indication? 1. 5 mg/kg by mouth (P.O.) every 12 hours 2. 15 mg/kg P.O. every 12 hours 3. 30 mg/kg P.O. every 12 hours 4. 45 mg/kg P.O. every 12 hours

1. 5 mg/kg by mouth (P.O.) every 12 hours The usual dosage for cefpodoxime proxetil in treating acute otitis media in children ages 6 months to 12 years is 5 mg/kg P.O. every 12 hours.

GI interventions for CF

1. A child with cystic fibrosis requires a high-calorie, high-protein, and well-balanced diet to meet energy and growth needs; multivitamins and vitamins A, D, E, and K are also administered. 2. Monitor weight and for failure to thrive. 3. Monitor stool patterns and for signs of intestinal obstruction. 4. The goal of treatment for pancreatic insufficiency is to replace pancreatic enzymes; pancreatic enzymes are administered within 30 minutes of eating and administered with all meals and all snacks (enzymes should not be given if the child is NPO). 5. The amount of pancreatic enzymes administered depends on the physician's preference and usually is adjusted to achieve normal growth and a decrease in the number of stools to two or three daily (additional enzymes are needed if the child is consuming high-fat foods). 6. Enteric-coated pancreatic enzymes should not be crushed or chewed; capsules can be taken apart and the contents can be sprinkled on a small amount of food for administration. 7. Monitor for constipation, intestinal obstruction, and rectal prolapse. 8. Monitor for signs of gastroesophageal reflux; place the infant in an upright position after eating, and teach the child to sit upright after eating.

Cystic Fibrosis

1. A chronic multisystem disorder (autosomal recessive trait disorder) characterized by exocrine gland dysfunction 2. The mucus produced by the exocrine glands is abnormally thick, tenacious, and copious, causing obstruction of the small passageways of the affected organs, particularly in the respiratory, gastrointestinal, and reproductive systems. 3. Common symptoms are associated with pancreatic enzyme deficiency and pancreatic fibrosis caused by duct blockage, progressive chronic lung disease as a result of infection, and sweat gland dysfunction resulting in increased sodium and chloride sweat concentrations. 4. An increase in sodium and chloride in sweat and saliva forms the basis for one diagnostic test, the sweat chloride test. 5. Cystic fibrosis is a progressive and incurable disorder, and respiratory failure is a common cause of death; organ transplantations may be an option to increase survival rates.

The charge nurse on the adolescent unit must decide which nurse should admit a new client. Based on the present client care assignments, who is the best candidate to admit the client? 1. A nurse who just discharged two clients with newly diagnosed diabetes 2. A nurse whose patient with asthma has decreasing oxygen saturation levels 3. A nurse caring for a client who is paralyzed and has no visiting family 4. A nurse who is about to start a complicated wet-to-dry dressing change

1. A nurse who just discharged two clients with newly diagnosed diabetes Having just discharged two clients, this nurse's patient load is low, and she's able to accept a new assignment. The patient with asthma requires constant monitoring by the nurse until the situation is resolved. Simple tasks and procedures are often more time consuming when patients with paralysis are involved because they can't directly aid in their own care. Additional time must also be allotted for the nurse about to undertake a complicated procedure, such as a wet-to-dry dressing change.

A 14-year-old child with type 1 diabetes checks his blood glucose level at 9:00 p.m. before going to bed. It has been 4 hours since his dinner and his regular insulin dose. His blood glucose level is 60 mg/dl, and he states that he feels a little shaky. Which of the following should the nurse suggest? 1. A snack of an 8-oz glass of milk and graham crackers with peanut butter before going to sleep 2. Going to sleep to decrease the metabolic demands on the body 3. Taking a dose of glucagon 4. Doing nothing because the glucose level is unreliable because the child measured it himself

1. A snack of an 8-oz glass of milk and graham crackers with peanut butter before going to sleep Milk is a readily absorbed form of carbohydrate and will elevate blood glucose level rapidly, thus alleviating hypoglycemia. Crackers and peanut butter contain complex carbohydrates and will maintain blood glucose level. Decreased activity and sleep aren't effective for hypoglycemia. Glucagon should be reserved for more severe signs of hypoglycemia, such as disorientation and unconsciousness. To avoid rapid deterioration, steps should always be taken whenever hypoglycemia is suspected, regardless of who's performing the procedure.

The charge nurse is making client care assignments. Which client is most appropriate for a licensed practical nurse? 1. A stable 6-month-old with pneumonia 2. A newly admitted 1-month-old with bronchiolitis 3. A newly admitted 15-year-old with diabetic ketoacidosis 4. A 12-year-old admitted for chemotherapy

1. A stable 6-month-old with pneumonia Of the clients listed, the most appropriate assignment for a licensed practical nurse is the stable 6-month-old admitted with pneumonia. A licensed practical nurse can care for clients who are stable and aren't receiving chemotherapy. Because they require close assessment, a newly admitted infant with bronchiolitis, a 15-year-old with diabetic ketoacidosis, and a 12-year-old who requires chemotherapy should be cared for by a registered nurse.

Parents bring their infant to the clinic, seeking treatment for vomiting and diarrhea that has lasted for 2 days. On assessment, the nurse detects dry mucous membranes and lethargy. What other finding suggests a fluid volume deficit? 1. A sunken fontanel 2. Decreased pulse rate 3. Increased blood pressure 4. Low urine specific gravity

1. A sunken fontanel In an infant, signs of fluid volume deficit (dehydration) include sunken fontanels, increased pulse rate, and decreased blood pressure. They occur when the body can no longer maintain sufficient intravascular fluid volume. When this happens, the kidneys conserve water to minimize fluid loss, which results in concentrated urine with a high specific gravity.

A mother of a 3-year-old asks a clinic nurse about appropriate and safe toys for the child. The nurse tells the mother that the most appropriate toy for a 3-year-old is which of the following? 1. A wagon 2. A golf set 3. A farm set 4. A jack set with marbles

1. A wagon Toys for the toddler must be strong, safe, and too large to swallow or place in the ear or nose. Toddlers need supervision at all times. Push-pull toys, large balls, large crayons , large trucks, and dolls are some of the appropriate toys. A farm set, a golf set, and jacks with marbles may contain items that the child could swallow.

Integumentary signs and symptoms of CF

1. Abnormally high concentrations of sodium and chloride in sweat is noted. 2. Parents report that the infant tastes "salty" when kissed. 3. Dehydration and electrolyte imbalances can occur, especially during hyperthermic conditions.

A 4-year-old, 40-lb (18.1-kg) child is brought to the pediatrician's office. He has upper respiratory symptoms and has had a fever for 2 days. He's diagnosed with a viral illness, and the mother is instructed to treat him with rest, fluids, and antipyretics. Which of the following would be the most appropriate medication dosage schedule? 1. Acetaminophen 225 mg (10 to 15 mg/kg/dose) q4h with intermittent doses of ibuprofen 180 mg (10 mg/kg/dose) q6h for temperature > 102.5° F (39.2° C) 2. Aspirin 290 mg (65 mg/kg/24 hours) q6h with intermittent doses of acetaminophen 225 mg q4h 3. Acetaminophen 140 mg (5 to 10 mg/kg/dose) q4h for a temperature lower than 102.5° F 4. Acetaminophen 225 mg (10 to 15 mg/kg/dose) q4h with intermittent doses of ibuprofen 90 mg (5 mg/kg/dose) q 6h for a temperature higher than 102.5° F

1. Acetaminophen 225 mg (10 to 15 mg/kg/dose) q4h with intermittent doses of ibuprofen 180 mg (10 mg/kg/dose) q6h for temperature > 102.5° F (39.2° C) The correct dosage schedule for acetaminophen is 10 to 15 mg/kg/dose every 4 hours, and for ibuprofen it's 10 mg/kg/dose every 6 hours for a temperature higher than 102.5° F. Aspirin shouldn't be given to children because of the association between aspirin use in children with influenza virus or chickenpox and Reye's syndrome (a life-threatening condition characterized by vomiting and lethargy that may progress to delirium and coma). A dosage of acetaminophen 5 to 10 mg/kg/dose isn't correct. Ibuprofen 5 mg/kg/dose is the correct dosage for a child with a temperature lower than 102.5° F.

The nurse is teaching an adolescent with inflammatory bowel disease about treatment with corticosteroids. Which adverse effects are concerns for this client? 1. Acne 2. Hirsutism 3. Mood swings 4. Osteoporosis 5. Growth spurts 6. Adrenal suppression

1. Acne 2. Hirsutism 3. Mood swings 4. Osteoporosis 6. Adrenal suppression Adverse effects of corticosteroids include acne, hirsutism, mood swings, osteoporosis, and adrenal suppression. Steroid use in children and adolescents may cause delayed growth, not growth spurts.

The mother of a 4-year-old child tells the nurse that her child is a very poor eater. What is the nurse's best recommendation for helping her increase her child's nutritional intake? 1. Allow the child to feed herself. 2. Use specially designed dishes for children — for example, a plate with the child's favorite cartoon character. 3. Only serve the child's favorite foods. 4. Allow the child to eat at a small table and chair by herself.

1. Allow the child to feed herself. The best recommendation is to allow the child to feed herself because the child's stage of development is the preschool period of initiative. Special dishes would enhance the primary recommendation. It's important to offer new foods and choices, not just serve her favorite foods. Using a small table and chair would also enhance the primary recommendation.

Which action should the nurse include in the care plan for a 2-month-old with heart failure? 1. Allow the infant to rest before feeding. 2. Bathe the infant and administer medications before feeding. 3. Weigh and bathe the infant before feeding. 4. Feed the infant when he cries.

1. Allow the infant to rest before feeding. Because feeding requires so much energy, an infant with heart failure should rest before feeding. Bathing and weighing the infant and administering medications should be scheduled around feedings. An infant expends energy when crying; therefore, it's best if the infant doesn't cry.

A maternity nurse is providing instructions to a new mother regarding the psychosocial development of the newborn infant. Using Erikson's psychosocial development theory, the nurse instructs the mother to: 1. Allow the newborn infant to signal a need. 2. Anticipate all the needs of the newborn infant. 3. Attend to the newborn infant immediately when crying. 4. Avoid the newborn infant during the first 10 minutes of crying.

1. Allow the newborn infant to signal a need. According to Erikson, the caregiver should not try to anticipate the newborn infant's needs at all times but must allow the newborn infant to signal needs. If a newborn infant is not allowed to signal a need, the newborn will not learn how to control the environment. Erikson believed that a delayed or prolonged response to a newborn infant's signal would inhibit the development of trust and lead to mistrust of others.

After a car accident, a child, age 10, is treated in the emergency department for a fractured clavicle and evaluated for a possible head injury. Alert and oriented, she keeps asking what will happen to her. Which nursing diagnosis is most appropriate? 1. Anxiety related to separation from parents and an unfamiliar environment 2. Hypothermia related to head injury 3. Interrupted family processes related to maturational crisis 4. Risk for infection related to sepsis

1. Anxiety related to separation from parents and an unfamiliar environment The nature of the accident, the child's pain, and the unfamiliar facility environment support a nursing diagnosis of Anxiety. A diagnosis of Hypothermia related to head injury isn't appropriate because the child is alert and oriented, indicating that a head injury, if present, isn't severe and is unlikely to cause hypothermia. Unlike the homecoming of a new baby or riding a bicycle for the first time, a car accident isn't a maturational crisis. Risk for infection related to sepsis isn't a plausible nursing diagnosis at this time.

A 14-year-old client reports right lower quadrant pain, nausea, vomiting, and a low-grade fever for the past 12 hours. A physical examination reveals rebound tenderness and a positive psoas sign. Based on these findings, what would the nurse suspect? 1. Appendicitis 2. Pancreatitis 3. Cholecystitis 4. Constipation

1. Appendicitis Right lower quadrant pain, rebound tenderness, nausea, vomiting, a positive psoas sign, and a low-grade fever are findings consistent with acute appendicitis. The other disorders may mimic appendicitis; however, the pain of pancreatitis is usually localized in the left upper quadrant. Cholecystitis is associated with right upper quadrant pain. Constipation wouldn't cause a fever.

Actions to Take in the Event of an Acute Asthma Attack

1. Assess airway patency and respiratory status. 2. Administer humidified oxygen by nasal cannula or face mask. 3. Administer quick-relief (rescue) medications. 4. Initiate an intravenous (IV) line. 5. Prepare the child for a chest radiograph if prescribed. 6. Prepare to obtain a blood sample for determining arterial blood gas levels if prescribed.

A healthy 2-month-old infant is being seen in the local clinic for a well-child checkup and initial immunizations. The nurse should anticipate administering which immunizations? 1. DTaP (diphtheria, tetanus, and acellular pertussis) 2. MMR (measles, mumps, and rubella) 3. OPV (oral polio vaccine) 4. HBV (hepatitis B vaccine) 5. Varicella (chickenpox) vaccine 6. HIB (Haemophilus influenzae type b vaccine)

1. DTaP (diphtheria, tetanus, and acellular pertussis) 4. HBV (hepatitis B vaccine) 6. HIB (Haemophilus influenzae type b vaccine) At age 2 months, the American Academy of Pediatrics recommends the administration of DTaP, IPV (inactivated polio vaccine), HBV, HIB, and pneumococcal conjugate vaccine. The MMR immunization should be administered at 12 to 15 months. The IPV — not the OPV — is currently used to minimize spread of polio. Infants may receive the varicella vaccine any time after the child's 1st birthday.

asthma s&s tx

1. Asthma is a chronic inflammatory disease of the airways. 2. Asthma is classified based on disease severity; management includes medications, environmental control of allergens, and child and family education. 3. The allergic reaction in the airways caused by the precipitant can result in an immediate reaction with obstruction occurring, and it can result in a late bronchial obstructive reaction several hours after the initial exposure to the precipitant. 4. Mast cell release of histamine leads to a bronchoconstrictive process, bronchospasm, and obstruction. 5. Diagnosis is made based on the child's symptoms, history and physical examination, chest radiograph, and laboratory tests. 6. Precipitants may trigger an asthma attack. 7. Status asthmaticus is an acute asthma attack , and the child displays respiratory distress despite vigorous treatment measures; this is a medical emergency that can result in respiratory failure and death if not treated.

The nurse is conducting an examination of a 6-month-old baby. During the examination, the nurse should be able to elicit which reflex? 1. Babinski's 2. Startle 3. Moro's 4. Dance

1. Babinski's Babinski's reflex may be present the entire first year of life. The startle reflex disappears around 4 months of age; the Moro reflex, by 3 or 4 months of age; and the dance reflex, after the 3rd or 4th week.

Epiglottitis

1. Bacterial form of croup 2. Inflammation of the epiglottis occurs, which may be caused by Haemophilus influenzae type b or Streptococcus pneumoniae; children immunized with H. influenzae type b (Hib vaccine) are at less risk for epiglottitis. 3. Occurs most frequently in children 2 to 8 years old, but can occur from infancy to adulthood 4. Onset is abrupt, and the condition occurs most often in the winter. 5. Considered an emergency situation because it can progress rapidly to severe respiratory distress

The nurse is caring for a 17-year-old girl with cystic fibrosis who has been admitted to the hospital to receive antibiotics and respiratory treatment for exacerbation of a lung infection. The client has a number of questions about her future and the consequences of the disease. Which statements about the course of cystic fibrosis are true? 1. Breast development is frequently delayed. 2. The client is at risk for developing diabetes. 3. Pregnancy and child-bearing aren't affected. 4. Normal sexual relationships can be expected. 5. Only males carry the gene for the disease. 6. By age 20, the client should be able to decrease the frequency of respiratory treatment.

1. Breast development is frequently delayed. 2. The client is at risk for developing diabetes. 4. Normal sexual relationships can be expected. Cystic fibrosis delays growth and the onset of puberty. Children with cystic fibrosis tend to be smaller than average size and develop secondary sex characteristics later in life. In addition, clients with cystic fibrosis are at risk for developing diabetes mellitus because the pancreatic duct becomes obstructed as pancreatic tissues are destroyed. Clients with cystic fibrosis can expect to have normal sexual relationships, but fertility becomes difficult because thick secretions obstruct the cervix and block sperm entry. Males and females carry the gene for cystic fibrosis. Pulmonary disease commonly progresses as the client ages, requiring additional respiratory treatment — not less.

RESPIRATORY SYNCYTIAL VIRUS (RSV)

1. Bronchiolitis is an inflammation of the bronchioles that causes production of thick mucus that occludes bronchiole tubes and small bronchi. 2. RSV causes an acute viral infection and is a common cause of bronchiolitis (other organisms that cause bronchiolitis include adenoviruses, parainfluenza viruses, and human metapneumovirus). 3. RSV, although not airborne, is highly communicable and is usually transferred by direct contact with respiratory secretions. 4. RSV occurs primarily in the winter and spring. 5. RSV is rarer in children older than 2 years, with a peak incidence at approximately 6 months of age. 6. At-risk children include children older than 1 year of age who have a chronic or disabling condition. 7. Identification of the virus is done via testing of nasal or nasopharyngeal secretions. 8. Prevention measures include encouraging breast-feeding; avoiding tobacco smoke exposure; using good handwashing techniques; and administering palivizumab (Synagis), a monoclonal antibody, to high-risk infants. Palivizumab is administered via intramuscular injection monthly for a 5-month period (usually from November to March).

A 14-month-old child weighing 26 lb (11.8 kg) is admitted for traction to treat congenital hip dislocation. When preparing the client's room, the nurse anticipates using which traction system? 1. Bryant's traction 2. Buck's extension traction 3. Overhead suspension traction 4. 90-90 traction

1. Bryant's traction Bryant's traction is used to treat femoral fractures or congenital hip dislocation in children younger than age 2 who weigh less than 30 lb (13.6 kg). Buck's extension traction is skin traction used for short-term immobilization or to correct bone deformities or contractures; overhead suspension traction is used to treat fractures of the humerus; and 90-90 traction is used to treat femoral fractures in children older than age 2.

A child with diabetes insipidus receives desmopressin acetate (DDAVP). When evaluating for therapeutic effectiveness, the nurse would interpret which finding as a positive response to this drug? 1. Decreased urine output 2. Increased urine glucose level 3. Decreased blood pressure 4. Relief of nausea

1. Decreased urine output The primary action of DDAVP is to stimulate water reabsorption by the kidneys, thereby decreasing the urine output. DDAVP has no effect on glucose levels, blood pressure, or nausea.

The nurse is working on the pediatric unit. Which client care assignment best demonstrates primary care nursing? 1. Caring for the same client from admission to discharge 2. Caring for different clients each shift to gain nursing experience 3. Taking vital signs for every client hospitalized on the unit 4. Assuming the charge nurse role instead of participating in direct client care

1. Caring for the same client from admission to discharge Primary care nursing requires that the primary nurse care for the same client (to whom she's assigned) during her scheduled shift. The associate nurse is assigned to the client care assignment when the primary nurse has a day off or during the evening and night shifts. Caring for different clients each shift doesn't promote continuity of care. Taking vital signs for every client on the floor is an example of team nursing, in which each member of the team is assigned one specific task for each client. The charge nurse may be directly involved in client care.

The nurse is taking a history from the parents of a 11-year-old girl admitted with Reye's syndrome. Which illness would the nurse expect the parents to report their child having the previous week? 1. Chickenpox 2. Bacterial meningitis 3. Strep throat 4. Lyme disease

1. Chickenpox Reye's syndrome commonly occurs about 1 week after a child has had a viral infection, such as chickenpox (varicella) or influenza. Children with flulike symptoms or chickenpox who receive aspirin are at increased risk for Reye's syndrome. Bacterial meningitis and strep throat are caused by bacteria and don't lead to Reye's syndrome. Lyme disease is caused by a spirochete and isn't implicated in Reye's syndrome.

Assessment findings with asthma

1. Child has episodes of dyspnea, wheezing, breathlessness, chest tightness, and cough , particularly at night or in the early morning or both. 2. Acute asthma attacks a. Episodes include progressively worsening shortness of breath, cough, wheezing, chest tightness, decreases in expiratory airflow secondary to bronchospasm, mucosal edema, and mucus plugging; air is trapped behind occluded or narrow airways, and hypoxemia can occur. b. The attack begins with irritability, restlessness, headache, feeling tired, or chest tightness; just before the attack, the child may present with itching localized at the front of the neck or over the upper part of the back. c. Respiratory symptoms include a hacking, irritable, nonproductive cough caused by bronchial edema. d. Accumulated secretions stimulate the cough; the cough becomes rattling, and there is production of frothy, clear, gelatinous sputum. e. The child experiences retractions. f. Hyperresonance on percussion of the chest is noted. g. Breath sounds are coarse and loud, with crackles, coarse rhonchi, and inspiratory and expiratory wheezing; expiration is prolonged. h. Child may be pale or flushed, and the lips may have a deep, dark red color that may progress to cyanosis (also observed in the nail beds and skin, especially around the mouth). i. Restlessness, apprehension, and diaphoresis occur. j. Child speaks in short, broken phrases. k. Younger children assume the tripod sitting position; older children sit upright, with the shoulders in a hunched-over position, the hands on the bed or a chair, and the arms braced to facilitate the use of the accessory muscles of breathing (child refuses to lie down). l. Exercise-induced attack : Cough, shortness of breath, chest pain or tightness, wheezing, and endurance problems occur during exercise. m. Severe spasm or obstruction: Breath sounds and wheezing cannot be heard (silent chest), and cough is ineffective (represents a lack of air movement). n. Ventilatory failure and asphyxia: Shortness of breath, with air movement in the chest restricted to the point of absent breath sounds , is noted; this is accompanied by a sudden increase in the respiratory rate.

A baby boy has just had surgery to repair his cleft lip. Which nursing intervention is the most important during the immediate postoperative period? 1. Clean the suture line carefully with a sterile solution after every feeding. 2. Lay the infant on his abdomen to help drain fluids from his mouth. 3. Allow the infant to cry to promote lung reexpansion. 4. Give the baby a pacifier to suck for comfort.

1. Clean the suture line carefully with a sterile solution after every feeding. To avoid an infection that could adversely affect the cosmetic outcome of the repair, the suture line must be cleaned very gently with a sterile solution after each feeding. Laying an infant on his abdomen after a cleft lip repair will put pressure on the suture line, causing damage. The infant can be positioned on his side to drain saliva without affecting the suture line. Crying puts tension on the suture line and should be avoided by anticipating the baby's needs, such as holding and cuddling him. Hard objects such as pacifiers should be kept away from the suture line because they can cause damage.

When developing a postoperative care plan for an infant scheduled for cleft lip repair, the nurse should assign highest priority to which intervention? 1. Comforting the child as quickly as possible 2. Maintaining the child in a prone position 3. Restraining the child's arms at all times, using elbow restraints 4. Avoiding disturbing any crusts that form on the suture line

1. Comforting the child as quickly as possible After surgery to repair a cleft lip, the primary goal of nursing care is to maintain integrity of the operative site. Crying causes tension on the suture line, so comforting the child as quickly as possible is the highest nursing priority. Parents may help by cuddling and comforting the child. The prone position is contraindicated after surgery because rubbing on the sheet may disturb the suture line. Elbow restraints may cause agitation; if used to prevent the child from disturbing the suture line, they must be removed, one at a time, every 2 hours so that the child can exercise and the nurse can assess for skin irritation. Crusts forming on the suture line contribute to scarring and must be cleaned carefully.

A child is to receive phenytoin (Dilantin), 5 mg/kg by mouth each day. When teaching the parents about the medication regimen, the nurse should use which approach? 1. Conduct brief teaching sessions, provide written materials during each visit, and repeat information as appropriate. 2. Ask the parents to spend an entire day at the facility so they can learn every detail about their child's care. 3. Call the parents at home and explain everything, allowing time for them to ask questions. 4. Send the parents the drug packaging insert so they can become familiar with the medication.

1. Conduct brief teaching sessions, provide written materials during each visit, and repeat information as appropriate. Effective teaching methods include providing simple instructions in short sessions, providing written materials, repeating information, and allowing time for questions. The other options are ineffective teaching strategies that may be overwhelming for the parents and frustrating for the nurse.

A neonate born several hours ago shows signs of a tracheoesophageal fistula (TEF). During the initial assessment, what does the nurse expect to find? 1. Continuous drooling 2. Diaphragmatic breathing 3. A slow response to stimuli 4. Passage of frothy meconium

1. Continuous drooling Signs of a TEF include continuous drooling, excessive oral secretions, and choking and coughing, which are especially pronounced during feeding. TEF doesn't cause diaphragmatic breathing, a slow response to stimuli, or passage of frothy meconium.

Reproductive signs and symptoms of CF

1. Cystic fibrosis can delay puberty in girls. 2. Fertility can be inhibited by the highly viscous cervical secretions, which act as a plug and block sperm entry. 3. Males are usually sterile , caused by the blockage of the vas deferens by abnormal secretions or by failure of normal development of duct structures.

A child, age 4, with a recent history of nausea, vomiting, and diarrhea is admitted to the pediatric unit with a diagnosis of gastroenteritis. During the physical examination, the nurse detects tenting. This finding supports a nursing diagnosis of: 1. Deficient fluid volume related to dehydration. 2. Risk for injury related to capillary fragility. 3. Ineffective peripheral tissue perfusion related to peripheral cyanosis. 4. Activity intolerance related to hypoxia.

1. Deficient fluid volume related to dehydration. Tenting, which indicates decreased skin turgor, is normal in geriatric clients and results from decreased elastin content. However, in other adults and in children, tenting more commonly results from dehydration. Therefore, this finding supports a nursing diagnosis of Deficient fluid volume related to dehydration. The other options are inappropriate because capillary fragility, altered tissue perfusion, and hypoxia rarely are associated with gastroenteritis.

A hospitalized infant, age 10 months, begins to choke while eating and quickly becomes unconscious. A foreign object isn't visible in the infant's airway, but respirations are absent and the pulse is 50 beats/minute and thready. The nurse attempts rescue breathing, but the ventilations are unsuccessful. What should the nurse do next? 1. Deliver four back blows. 2. Deliver four chest thrusts. 3. Perform chest compressions. 4. Deliver four abdominal thrusts.

1. Deliver four back blows. If rescue breathing is unsuccessful in a child under age 1, the nurse should deliver four back blows, followed by four chest thrusts, to try to expel the object from the obstructed airway. The nurse shouldn't perform chest compressions because the infant has a pulse and because chest compressions are ineffective without a patent airway for ventilation. The nurse shouldn't use abdominal thrusts for a child under age 1 because they can injure the abdominal organs.

A disabled school-age child whose parents are overprotective may display which characteristics? 1. Dependency, fearfulness, and lack of outside interests 2. Extreme independence, defiance, and a high level of risk taking 3. Shyness and loneliness 4. Pride and confidence in one's ability to cope

1. Dependency, fearfulness, and lack of outside interests Disabled children whose parents are overprotective tend to have marked dependency, fearfulness, inactivity, and lack of outside interests. Children who are raised by oversolicitous and guilt-ridden parents are often overly independent, defiant, and high-risk takers. Children who are reared by parents who emphasize the child's deficits and tend to isolate the child may appear shy and lonely. Children who are reared by parents who establish reasonable limits have pride and confidence in their ability to cope successfully.

A 4-year old child diagnosed with leukemia is hospitalized for chemotherapy. The child is fearful of the hospitalization . Which nursing intervention would be most appropriate to alleviate the child's fears? 1. Encourage the child's parents to stay with the child. 2. Encourage play with other children of the same age. 3. Advise the family to visit only during the scheduled visiting hours. 4. Provide a private room, allowing the child to bring the favorite toys from home.

1. Encourage the child's parents to stay with the child. Although the preschooler already may be spending some time away from parents at a day care center or preschool, illness adds a stressor that makes separation more difficult . The child may ask repeatedly when parents will be coming for a visit or may constantly want to call the parents. Options 3 and 4 increase stress related to separation anxiety. Option 2 is unrelated to the subject of the question and, in addition, may not be appropriate for a child who may be immunocompromised and at risk for infection.

The nurse is teaching a group of parents about otitis media. When discussing why children are predisposed to this disorder, the nurse should mention the significance of which anatomical feature? 1. Eustachian tubes 2. Nasopharynx 3. Tympanic membrane 4. External ear canal

1. Eustachian tubes In a child, the eustachian tubes are short and lie in a horizontal plane, promoting entry of nasopharyngeal secretions into the tubes and thus setting the stage for otitis media. The nasopharynx, tympanic membrane, and external ear canal have no unusual features that would predispose a child to otitis media.

At a previous visit, the parents of an infant with cystic fibrosis received instruction in the administration of pancrelipase (Pancrease). At a follow-up visit, which finding in the infant suggests that the parents are not administering the pancreatic enzymes as instructed? 1. Fatty stools 2. Liquid stools 3. Bloody stools 4. Normal stools

1. Fatty stools Pancreatic enzymes normally aid in food digestion in the intestine. In a child with cystic fibrosis, however, these natural enzymes cannot reach the intestine because mucus blocks the pancreatic duct. Without these enzymes, undigested fats and proteins produce fatty stools. Treatment with pancreatic enzymes should result in stools of normal consistency; noncompliance with the treatment produces fatty stools. Noncompliance doesn't cause liquid stools, or bloody stools.

A 6-year-old child with a history of varicella and aspirin intake is brought to the emergency department. The nurse suspects Reye's syndrome. Which assessment findings are consistent with this syndrome? 1. Fever, decreased level of consciousness (LOC), and impaired liver function 2. Joint inflammation, red macular rash with a clear center, and low-grade fever 3. Peripheral edema, fever for 5 or more days, and "strawberry tongue" 4. Red, raised "bull's eye" rash, malaise, and joint pain

1. Fever, decreased level of consciousness (LOC), and impaired liver function Reye's syndrome occurs in children with a history of a viral infection, varicella, or influenza. It's often associated with the administration of aspirin. The child presents with fever and decreased LOC, which can lead to coma and death. As the disease progresses, the child also develops impaired liver function. A child with joint pain, a red macular rash with a clear center, and a low-grade fever probably has rheumatic fever. A child presenting with peripheral edema, fever for more than 5 days, and a "strawberry tongue" probably has Kawasaki disease. A child with a red, raised "bull's eye" rash, malaise, and joint pain should be tested for Lyme disease.

When teaching parents about fifth disease (erythema infectiosum) and its transmission, the nurse should provide which information? 1. Fifth disease is transmitted by respiratory secretions. 2. Fifth disease has an unknown transmission mode. 3. Fifth disease is transmitted by respiratory secretions, stool, and urine. 4. Fifth disease is transmitted by stool.

1. Fifth disease is transmitted by respiratory secretions. Fifth disease is transmitted by respiratory secretions. The transmission mode for roseola is unknown. Rubella is transmitted by respiratory secretions, stool, and urine. Intestinal parasitic conditions, such as giardiasis and pinworm infection, are transmitted by stool.

Interventions for RSV

1. For a child with bronchiolitis, interventions are aimed at treating symptoms and include airway maintenance, cool humidified air and oxygen, adequate fluid intake, and medications. 2. For a hospitalized child with RSV, isolate the child in a single room or place in a room with another child with RSV. 3. Ensure that nurses caring for a child with RSV do not care for other high-risk children. 4. Use contact and standard precautions during care; using good handwashing techniques and wearing gloves and gowns are necessary. 5. Monitor airway status and maintain a patent airway. 6. For most effective airway maintenance, position the child at a 30- to 40-degree angle with the neck slightly extended to maintain an open airway and decrease pressure on the diaphragm. 7. Provide cool, humidified oxygen as prescribed. 8. Monitor pulse oximetry levels. 9. Encourage fluids; fluids administered intravenously may be necessary until the acute stage has passed. 10. Periodic suctioning may be necessary if nasal secretions are copious; use of a bulb syringe for suctioning may be effective. Suctioning should be done before feeding to promote comfort and adequate intake. 11. Administer ribavirin (Virazole), an antiviral medication, if prescribed (administered via the inhalation route). **Cough suppressants are administered with caution because they can interfere with the clearance of respiratory secretions.

An 8-year-old child has just returned from the operating room after having a tonsillectomy. The nurse is preparing to do a postoperative assessment. The nurse should be alert for which signs and symptoms of bleeding? 1. Frequent clearing of the throat 2. Breathing through the mouth 3. Frequent swallowing 4. Sleeping for long intervals 5. Pulse rate of 98 beats/minute 6. Blood red vomitus

1. Frequent clearing of the throat 3. Frequent swallowing 6. Blood red vomitus A classic sign of bleeding after tonsillectomy is frequent swallowing; this occurs because blood drips down the back of the throat, tickling it. Other signs include frequent clearing of the throat and vomiting of bright red blood. Vomiting of dark blood may be seen if the child swallowed blood during surgery but doesn't indicate postoperative bleeding. Breathing through the mouth is common because of dried secretions in the nares. Sleeping for long intervals is normal after a client receives sedation and anesthesia. A pulse rate of 98 beats/minute is in the normal range for this age-group.

A nurse is caring for a child after a tonsillectomy. The nurse monitors the child, knowing that which of the following indicates that the child is bleeding? 1. Frequent swallowing 2. A decreased pulse rate 3. Complaints of discomfort 4. An elevation in blood pressure

1. Frequent swallowing A tonsillectomy is the surgical removal of the tonsils. Frequent swallowing, restlessness, a fast and thready pulse, and vomiting bright red blood are signs of bleeding . An elevated blood pressure and complaints of discomfort are not indications of bleeding.

The nurse is preparing to administer an I.V. containing dextrose 10% in 1/4 normal saline solution to a 6-month-old client. The nurse should select which tubing to safely administer the solution? 1. I.V. tubing with a volume-control chamber 2. I.V. tubing with a macrodrip chamber 3. I.V. tubing with a special filter 4. Standard I.V. tubing used for adults

1. I.V. tubing with a volume-control chamber Because infants have a small circulating blood volume, inadvertent administration of extra I.V. fluid can cause fluid volume excess. To prevent this from occurring, I.V. tubing with a volume-control chamber (such as a Buretrol or Solu-set) should always be used for infants and children to closely regulate the amount of fluid infused. The volume-control chamber should be filled only with enough I.V. fluid for the next two 2 hours. A microdrip chamber that allows for 60 drops/ml (as opposed to a macrodrip chamber, which allows for 10 to 20 drops/ml, depending on the manufacturer) should be used to infuse the smaller amounts of I.V. fluids an infant needs. A filter is typically used only for the administration of total parenteral nutrition (TPN) and certain blood products. Standard I.V. tubing for adults should be avoided for infants because of the inability to closely regulate the amount of fluid infused.

Respiratory interventions for CF

1. Goals of treatment include preventing and treating pulmonary infection by improving aeration, removing secretions, and administering antibiotic medications. 2. Monitor respiratory status including lung sounds and the presence and characteristics of a cough. 3. Chest physiotherapy (percussion and postural drainage) on awakening and in the evening (more frequently during pulmonary infection) needs to be done every day to maintain pulmonary hygiene; chest physiotherapy should not be performed before or immediately after a meal. 4. A Flutter Mucus Clearance Device (a small, hand-held plastic pipe with a stainless steel ball on the inside) facilitates the removal of mucus and may be prescribed; store away from small children because if the device separates, the steel ball poses a choking hazard. 5. Hand-held percussors or a special vest device that provides high-frequency chest wall oscillation may be prescribed to help loosen secretions. 6. A positive expiratory pressure mask may be prescribed; use of this mask forces secretion to the upper airway for expectoration. 7. The child should be taught the forced expiratory technique (huffing) to mobilize secretions for expectoration. 8. Bronchodilator medication by aerosol may be prescribed; the medication opens the bronchi for easier expectoration (administered before chest physiotherapy when the child has reactive airway disease or is wheezing). Medications that decrease the viscosity of mucus may also be prescribed. 9. A physical exercise program with the aim of stimulating mucus expectoration and establishing an effective breathing pattern should be instituted. 10. Aerosolized or intravenous antibiotics may be prescribed and administered at home through a central venous access device. 11. Oxygen may be prescribed during acute episodes; monitor closely for oxygen narcosis (signs include nausea and vomiting, malaise, fatigue, numbness and tingling of extremities, substernal distress) because a child with cystic fibrosis may have chronic carbon dioxide retention. 12. Monitor for hemoptysis; more than 250 mL in 24 hours for an older child (less for a younger child) needs to be reported to the health care provider and treated immediately. 13. Hemoptysis may be controlled with measures such as bedrest, antibiotics if an infection is present, blood replacement therapy, and vitamin K; if hemoptysis persists, the site of bleeding may need to be cauterized.

When assessing a child with juvenile hypothyroidism, the nurse expects which finding? 1. Goiter 2. Recent weight loss 3. Insomnia 4. Tachycardia

1. Goiter Juvenile hypothyroidism results in goiter, weight gain, sleepiness, and a slow heart rate. It doesn't cause weight loss, insomnia, or tachycardia.

When examining school-age and adolescent children, the nurse routinely screens for scoliosis. Which statement accurately summarizes how to perform this screening? 1. Have the child stand firmly on both feet and bend forward at the hips, with the trunk exposed. 2. Listen for a clicking sound as the child abducts the hips. 3. Have the child run the heel of one foot down the shin of the other leg while standing. 4. Have the child shrug the shoulders as the nurse applies mild pressure to the shoulders.

1. Have the child stand firmly on both feet and bend forward at the hips, with the trunk exposed. To screen for scoliosis, a lateral curvature of the spine, the nurse has the child stand firmly on both feet with the trunk exposed and examines the child from behind, checking for asymmetry of the shoulders, scapulae, or hips. The nurse then asks the child to bend forward at the hips and inspects for a rib hump, a sign of scoliosis. The nurse would listen for a clicking sound while the child abducts the hips when screening for congenital hip dysplasia. The heel-to-shin test evaluates cerebellar function. Having the child shrug the shoulders against mild resistance helps evaluate the integrity of cranial nerve XI.

A child, age 2, is brought to the emergency department after ingesting an unknown number of aspirin tablets about 30 minutes earlier. On entering the examination room, the child is crying and clinging to the mother. Which data should the nurse obtain first? 1. Heart rate, respiratory rate, and blood pressure 2. Recent exposure to communicable diseases 3. Number of immunizations received 4. Height and weight

1. Heart rate, respiratory rate, and blood pressure The most important data to obtain on a child's arrival in the emergency department are vital sign measurements. The nurse should gather the other data later.

The nurse is teaching bicycle safety to a child and his parents. What protective device should the nurse tell the parents is most important in preventing or lessening the severity of injury related to bicycle crashes? 1. Helmet 2. Knee pads 3. Elbow pads 4. Reflectors

1. Helmet A well-fitting helmet is the most important safety feature to stress to children and parents. According to the American Academy of Pediatrics, wearing a helmet correctly can prevent or lessen the severity of brain injuries resulting from bicycle crashes. Knee pads, elbow pads, and reflectors are also important safety devices but they aren't as important as a helmet.

A nurse and a nursing assistant are caring for a group of clients on the adolescent unit. Which task could the nurse safely delegate to the nursing assistant? 1. Helping a client into a wheelchair 2. Administering acetaminophen (Tylenol) for a client's fever 3. Assisting a physician during the first postoperative dressing change 4. Reviewing discharge instructions for a newly diagnosed diabetic client

1. Helping a client into a wheelchair Moving a client into a wheelchair is within the scope of practice of the nursing assistant. Only licensed personnel are authorized to administer medications. As a registered nurse, it's important to personally assess the client's surgical wound to be able to monitor for adverse changes. Also, it's important to provide adequate client education about a newly diagnosed disease to ensure complete compliance; the nursing assistant may not have the knowledge to do so.

Which finding would be least suggestive of necrotizing enterocolitis (NEC) in an infant? 1. Hepatomegaly 2. Distended abdomen 3. Gastric retention 4. Blood in the stool

1. Hepatomegaly Hepatomegaly is most commonly observed in neonatal sepsis, not NEC. A distended abdomen, gastric retention, and blood in the stool are all signs of NEC and should be monitored closely in infants who are at risk.

Signs and symptoms of epiglottitis

1. High fever 2. Sore, red, and inflamed throat (large, cherry red, edematous epiglottis) and pain on swallowing 3. Absence of spontaneous cough 4. Drooling 5. Agitation 6. Muffled voice 7. Retractions and child struggles to breathe 8. Inspiratory stridor aggravated by the supine position 9. Tachycardia 10. Tachypnea progressing to more severe respiratory distress (hypoxia, hypercapnia, respiratory acidosis, decreased level of consciousness) 11. Tripod positioning: While supporting the body with the hands, the child leans forward , thrusts the chin forward and opens the mouth in an attempt to widen the airway

Twenty-four hours after birth, a neonate hasn't passed meconium. The nurse suspects which condition? 1. Hirschsprung's disease 2. Celiac disease 3. Intussusception 4. Abdominal wall defect

1. Hirschsprung's disease Failure to pass meconium is an important diagnostic indicator for Hirschsprung's disease. Hirschsprung's disease is a potentially life-threatening congenital large-bowel disorder characterized by the absence or marked reduction of parasympathetic ganglion cells in a segment of the colorectal wall; narrowing impairs intestinal motility and causes severe, intractable constipation leading to partial or complete colonic obstruction. The other options aren't associated with failure to pass meconium.

A nurse practicing in a nurse-managed clinic suspects that an 8-year-old client's chronic sinusitis and upper respiratory tract infections may be due to allergies. She orders an immunoglobulin assay. Which of the following would the nurse expect to find elevated? 1. Immunoglobulin E 2. Immunoglobulin D 3. Immunoglobulin G 4. Immunoglobulin M

1. Immunoglobulin E Immunoglobulin (Ig) E is predominantly found in saliva and tears as well as intestinal and bronchial secretions. Increased levels of IgE may be found in allergic disorders. IgD's physiologic function is unknown and constitutes only 1% of the total number of circulating immunoglobulins. IgG is elevated in the presence of viral and bacterial infections. IgM is the first antibody activated after an antigen enters the body, and is especially effective against gram-negative organisms.

Which assessment finding is an early sign of heart failure in a toddler? 1. Increased respiratory rate 2. Increased urine output 3. Decreased weight 4. Decreased heart rate

1. Increased respiratory rate Increased respiratory and heart rates are the earliest signs of heart failure. Decreased urine output and increased weight are later signs.

The nurse is evaluating a child with acute poststreptococcal glomerulonephritis (APSGN) for signs of improvement. Which finding typically is the earliest sign of improvement? 1. Increased urine output 2. Increased appetite 3. Increased energy level 4. Decreased diarrhea

1. Increased urine output Increased urine output, a sign of improving kidney function, typically is the first sign that a child with APSGN is improving. Increased appetite, an increased energy level, and decreased diarrhea aren't specific to APSGN.

A baby undergoes surgery to correct an esophageal atresia and tracheoesophageal fistula. Which nursing diagnosis has the highest priority during the first 24 hours postoperatively? 1. Ineffective airway clearance 2. Imbalanced nutrition: Less than body requirements 3. Interrupted breast-feeding 4. Hypothermia

1. Ineffective airway clearance Ineffective airway clearance is the priority nursing diagnosis in the immediate postoperative period. The infant's airway must be carefully assessed and frequent suctioning may be necessary to remove mucus while taking care not to pass the catheter as far as the suture line. Assess breath sounds, respiratory rate, skin color, and ease of breathing. Because of the risk of edema and airway obstruction, keep a laryngoscope and endotracheal intubation equipment readily available. Imbalanced nutrition, Interrupted breast-feeding, and Hypothermia are also important during the postoperative period but only after a patent airway is ensured.

LARYNGOTRACHEOBRONCHITIS/croup

1. Inflammation of the larynx, trachea, and bronchi 2. Most common type of croup; may be viral or bacterial and most frequently occurs in children younger than 5 years 3. Common causative organisms include parainfluenzae virus types 1 and 2, respiratory syncytial virus (RSV), Mycoplasma pneumoniae, and influenza. 4. Characterized by gradual onset that may be preceded by an upper respiratory infection

A 3-month-old with meningococcal meningitis has just been admitted to the pediatric unit. Which nursing intervention has the highest priority? 1. Instituting droplet precautions 2. Administering acetaminophen (Tylenol) 3. Obtaining history information from the parents 4. Orienting the parents to the pediatric unit

1. Instituting droplet precautions Instituting droplet precautions is a priority for a newly admitted infant with meningococcal meningitis. Acetaminophen may be prescribed but administering it doesn't take priority over instituting droplet precautions. Obtaining history information and orienting the parents to the unit don't take priority.

A 2-year-old child is brought to the emergency department with suspected croup. Which of the following assessment findings reflects increasing respiratory distress? 1. Intercostal retractions 2. Bradycardia 3. Decreased level of consciousness 4. Flushed skin

1. Intercostal retractions Clinical manifestations of respiratory distress include tachypnea, tachycardia, restlessness, dyspnea, intercostal retractions, and cyanosis.

Which parameter would not be an appropriate indicator of pain relief in an adolescent? 1. Intermittent sleeping 2. Change in behavior 3. Statement of decreased pain 4. Change in vital signs

1. Intermittent sleeping Sleeping isn't a reliable indicator of pain relief because the teen may use sleep as a coping mechanism. Positive changes in behavior and vital signs and a statement of less pain are indicators of an effective response to pain medication.

A 10-month-old infant with tetralogy of Fallot (TOF) experiences a cyanotic episode. To improve oxygenation during such an episode, the nurse should place the infant in which position? 1. Knee-to-chest 2. Fowler's 3. Trendelenburg's 4. Prone

1. Knee-to-chest TOF involves four defects: pulmonary stenosis, right ventricular hypertrophy, ventricular-septal defect (VSD), and dextroposition of the aorta with overriding of the VSD. Pulmonary stenosis decreases pulmonary blood flow and right-to-left shunting via the VSD, causing desaturated blood to circulate. The knee-to-chest position reduces venous return from the legs and increases systemic vascular resistance, maximizing pulmonary blood flow and improving oxygenation status. During a cyanotic episode, the child with TOF typically assumes this position instinctively. Fowler's, Trendelenburg's, and the prone positions don't improve oxygenation.

A preschool child is admitted to the pediatric unit with acute nephritis. Which electrolyte replacement agent is used as an adjunct to treatment for this condition? 1. Magnesium sulfate 2. Calcium glubionate 3. Potassium chloride 4. Sodium lactate

1. Magnesium sulfate Magnesium sulfate is an electrolyte that is used as an adjunct to treat acute nephritis. It also is used to treat seizures and severe toxemia. Calcium glubionate, potassium chloride, and sodium lactate aren't therapeutic in acute nephritis and, in fact, may worsen the condition.

Interventions for epiglottitis

1. Maintain a patent airway. 2. Assess respiratory status and breath sounds , noting nasal flaring, the use of accessory muscles, retractions, and the presence of stridor. 3. Assess temperature by the axillary route, not the oral route. 4. Monitor pulse oximetry. 5. Prepare the child for lateral neck films to confirm the diagnosis (accompany the child to the radiology department). 6. Maintain NPO status. 7. Do not leave the child unattended. 8. Avoid placing the child in a supine position because this position would affect the respiratory status further. 9. Do not restrain the child or take any other measure that may agitate the child. 10. Administer intravenous fluids as prescribed; insertion of an intravenous line may need to be delayed until an adequate airway is established because this procedure may agitate the child. 11. Administer intravenous antibiotics as prescribed; these are usually followed by oral antibiotics. 12. Administer analgesics and antipyretics (acetaminophen [Tylenol] or ibuprofen [Motrin]) to reduce fever and throat pain as prescribed. 13. Administer corticosteroids to decrease inflammation and reduce throat edema as prescribed. 14. Nebulized epinephrine (racemic epinephrine) may be prescribed for severe cases (causes mucosal vasoconstriction and reduces edema); heliox (mixture of helium and oxygen) may also be prescribed to reduce mucosal edema. 15. Provide cool mist oxygen therapy as prescribed; high humidification cools the airway and decreases swelling. 16. Have resuscitation equipment available, and prepare for endotracheal intubation or tracheotomy for severe respiratory distress. 17. Ensure that the child is up to date with immunizations, including Hib conjugate vaccine **If epiglottitis is suspected, no attempts should be made to visualize the posterior pharynx, obtain a throat culture, or take an oral temperature. Otherwise, spasm of the epiglottis can occur leading to complete airway occlusion

Interventions for croup

1. Maintain a patent airway. 2. Assess respiratory status and monitor pulse oximetry; monitor for nasal flaring, sternal retraction, and inspiratory stridor 3. Monitor for adequate respiratory exchange; monitor for pallor or cyanosis. 4. Elevate the head of the bed and provide rest. 5. Provide humidified oxygen via a cool air or mist tent as prescribed for a hospitalized child 6. Instruct the parents to use a cool air vaporizer at home; other measures include having the child breathe in the cool night air or the air from an open freezer or taking the child to a cool basement or garage. 7. Provide and encourage fluid intake; intravenous fluids may be prescribed to maintain hydration status if the child is unable to take fluids orally. 8. Administer analgesics as prescribed to reduce fever. 9. Teach the parents to avoid administering cough syrups or cold medicines, which may dry and thicken secretions. 10. Administer corticosteroids if prescribed to reduce inflammation and edema. 11. Administer nebulized epinephrine (racemic epinephrine) as prescribed; this may be prescribed for children with severe disease experiencing stridor at rest, retractions, or difficulty breathing. 12. Administer antibiotics as prescribed, noting that they are not indicated unless a bacterial infection is present. 13. Heliox (mixture of helium and oxygen) may be prescribed; this medication reduces the work of breathing, reduces airway turbulence, and helps to relieve airway obstruction. 14. Have resuscitation equipment available. 15. Provide appropriate reassurance and education to the parents or caregivers. Isolation precautions should be implemented for a hospitalized child with an upper respiratory infection until the cause of the infection is known.

A 9-month-old infant is admitted with diarrhea and dehydration. The nurse plans to assess the child's vital signs frequently. Which other action would provide the most important assessment information? 1. Measuring the infant's weight 2. Obtaining a stool specimen for analysis 3. Obtaining a urine specimen for analysis 4. Inspecting the infant's posterior fontanel

1. Measuring the infant's weight Frequent weight measurement provides the most important information about fluid balance and the infant's response to fluid replacement. Although the results of stool or urine analysis may provide some information, they typically aren't available for at least 24 hours. The posterior fontanel usually closes from ages 6 to 8 weeks and therefore doesn't reflect fluid balance in a 9-month-old infant.

GI signs and symptoms of CF

1. Meconium ileus in the newborn is the earliest manifestation. 2. Intestinal obstruction (distal intestinal obstructive syndrome) caused by thick intestinal secretions can occur; signs include pain, abdominal distention, nausea, and vomiting. 3. Stools are frothy and foul-smelling. 4. Deficiency of the fat-soluble vitamins A, D, E, and K, which can result in easy bruising, bleeding, and anemia, occurs. 5. Malnutrition and failure to thrive is a concern. 6. Demonstration of hypoalbuminemia can occur from diminished absorption of protein , resulting in generalized edema. 7. Rectal prolapse can result from the large, bulky stools and increased intra-abdominal pressure. 8. Pancreatic fibrosis can occur and places the child at risk for diabetes mellitus.

A 15-month-old child has just received his routine immunizations, including diphtheria, tetanus, and acellular pertussis (DTaP); inactivated polio vaccine (IPV); measles, mumps, and rubella (MMR); varicella; and pneumococcal conjugate vaccine (PCV). What information should the nurse give to the parents before they leave the office? 1. Minor symptoms can be treated with acetaminophen (Tylenol). 2. Minor symptoms can be treated with aspirin (A.S.A.). 3. Call the office if the toddler develops a temperature above 103° F (39.4° C), seizures, or difficulty breathing. 4. Soreness at the immunization site and mild fever are common. 5. The immunizations prevent the toddler from contracting their associated diseases. 6. The toddler should restrict his activity for the remainder of the day.

1. Minor symptoms can be treated with acetaminophen (Tylenol). 3. Call the office if the toddler develops a temperature above 103° F (39.4° C), seizures, or difficulty breathing. 4. Soreness at the immunization site and mild fever are common. Minor symptoms, such as soreness at the immunization site and mild fever, can be treated with acetaminophen or ibuprofen. Aspirin should be avoided in children because of its association with Reye's syndrome. The parents should notify the clinic if serious complications (such as a temperature above 103° F, seizures, or difficulty breathing) occur. Minor discomforts, such as soreness and mild fever, are common after immunizations. Immunizing the child decreases the health risks associated with contracting certain diseases; it doesn't prevent the toddler from acquiring them. Although the child may prefer to rest after immunizations, it isn't necessary to restrict his activity.

The nurse formulates a nursing diagnosis of Risk for infection for a child with Down syndrome. Which condition typically seen in children with this syndrome supports this nursing diagnosis? 1. Muscular hypotonicity 2. Muscle spasticity 3. Increased mucus viscosity 4. Hypothyroidism

1. Muscular hypotonicity Several conditions make the child with Down syndrome highly vulnerable to respiratory infections. For example, the hypotonicity of chest muscles leads to diminished respiratory expansion and pooling of secretions, while an underdeveloped nasal bone impairs mucus drainage. Down syndrome isn't associated with muscle spasticity or increased mucus viscosity. Although hypothyroidism is common in children with Down syndrome, it doesn't increase the risk of infection.

A mother brings her son to the pediatrician's office for evaluation of chronic stomach pain. The mother states that the pain seems to go away when she tells the child that he can stay home from school. The physician diagnoses school phobia. Which other behaviors or symptoms may present in the child with school phobia? 1. Nausea 2. Headaches 3. Weight loss 4. Dizziness 5. Fever

1. Nausea 2. Headaches 4. Dizziness Children with school phobia commonly complain of vague symptoms, such as stomachaches, nausea, headaches, and dizziness, to avoid going to school. These symptoms typically don't occur on weekends. A careful history must be taken to identify a pattern of school avoidance. Signs and symptoms such as weight loss and fever are more likely to have a physiologic cause and are uncommon in the child with school phobia.

When caring for children who are sick, who have sustained traumas, or who are suffering from nutritional inadequacies, the nurse should know the correct hemoglobin (Hb) values for children. Which range would be inaccurate? 1. Neonates: 10.6 to 16.5 g/dl 2. 3 months: 10.6 to 16.5 g/dl 3. 3 years: 9.4 to 15.5 g/dl 4. 10 years: 10.7 to 15.5 g/dl

1. Neonates: 10.6 to 16.5 g/dl To sustain them until active erythropoiesis begins, neonates have Hb concentrations higher than those of older children. The normal value of Hb for neonates is 18 to 27 g/dl. Disease as well as such nonpathologic conditions as age, sex, altitude, and the degree of fluid retention or dehydration can affect Hb values. The values for a 3-month-old, a 3-year-old, and a 10-year-old are correct as stated above.

The physician prescribes acetaminophen (Tylenol) elixir, 160 mg every 4 hours, for a 14-month-old child who weighs 20 lb (9.08 kg). This drug, supplied in a bottle labeled 160 mg/tsp, has a safe dosage of 10 mg/kg/dose. The nurse should administer how many milliliters? 1. None because this isn't a safe dose 2. 2.5 ml 3. 5 ml 4. 7.5 ml

1. None because this isn't a safe dose For this client, the safe dose of this drug is 90.8 mg (9.08 kg × 10 mg/kg = 90.8 mg). Therefore, the prescribed dose isn't safe.

The mother of a school-age child reports that her child is having some problems in school. Which action would be the priority? 1. Obtain more information from the mother and the child. 2. Refer the child to the school psychologist for testing. 3. Talk to the child's health care provider to understand the child better. 4. Talk to the child's teacher to gain a perspective on the situation.

1. Obtain more information from the mother and the child. In this situation, the nurse needs more information before proceeding and should question the mother and child about the problems. Referring the child to the school psychologist and talking to the child's health care provider and teacher are all important components of a treatment plan, but obtaining more information comes first.

An infant goes into cardiac arrest. When delivering chest compressions as part of cardiopulmonary resuscitation (CPR), where should the rescuer place the fingers? 1. One fingerbreadth below the nipple line, directly over the sternum 2. Directly over the xiphoid process 3. Over the lower third of the sternum 4. Directly over the left nipple

1. One fingerbreadth below the nipple line, directly over the sternum When delivering chest compressions to an infant, the rescuer should place the fingers one fingerbreadth below the nipple line, directly over the sternum. Compared to the heart of an older child or adult, the infant's heart is higher and more horizontal relative to anatomical landmarks. Placing the fingers over the xiphoid process could cause injury; placing them over the lower third of the sternum or directly over the nipple could make chest compressions ineffective.

Which intervention should be included in the care plan for a 6-month-old with deficient fluid volume related to excessive GI losses in stool and emesis? 1. Oral electrolyte replacement solutions, breast milk, or lactose-free formula 2. I.V. fluid replacement therapy 3. Clear fluids, such as fruit juices, carbonated soft drinks, and gelatin 4. Delayed introduction of food for several days followed by the BRAT (bananas, rice, apples, and toast or tea) diet

1. Oral electrolyte replacement solutions, breast milk, or lactose-free formula Oral electrolyte replacement solutions given in small amounts can replace fluid and electrolyte losses in an infant with mild diarrhea and vomiting. I.V. fluids are usually reserved for clients experiencing severe vomiting and dehydration. Fruit juices, carbonated soft drinks, and the BRAT diet, which are high in carbohydrates and low in electrolytes, aren't recommended.

Assessment findings for tonsillitis

1. Persistent or recurrent sore throat 2. Enlarged, bright red tonsils that may be covered with white exudate 3. Difficulty in swallowing 4. Mouth breathing and an unpleasant mouth odor 5. Fever 6. Cough 7. Enlarged adenoids may cause nasal quality of speech, mouth breathing, hearing difficulty, snoring, or obstructive sleep apnea.

A nurse is preparing for the admission of an infant with a diagnosis of bronchiolitis caused by respiratory syncytial virus (RSV). Which interventions would the nurse include in the plan of care? Select all that apply. 1. Place the infant in a private room. 2. Ensure that the infant's head is in a flexed position. 3. Wear a mask at all times when in contact with the infant. 4. Place the infant in a tent that delivers warm humidified air. 5. Position the infant side-lying, with the head lower than the chest. 6. Ensure that nurses caring for the infant with RSV do not care for other high-risk children.

1. Place the infant in a private room. 6. Ensure that nurses caring for the infant with RSV do not care for other high-risk children. Respiratory syncytial virus (RSV) is a highly communicable disorder and is not transmitted via the airborne route. The virus usually is transferred by the hands. Use of contact and standard precautions during care (wearing gloves and a gown) reduce nosocomial transmission of RSV. A mask is unnecessary. Additionally, it is important to ensure that nurses caring for a child with RSV do not care for other high-risk children to prevent the transmission of the infection. An infant with RSV should be isolated in a private room or in a room with another infant with RSV infection. The infant should be positioned with the head and chest at a 30- to 40-degree angle and the neck slightly extended to maintain an open airway and decrease pressure on the diaphragm . Cool humidified oxygen is delivered to relieve dyspnea, hypoxemia, and insensible water loss from tachypnea.

PostOp for tonsillectomy

1. Position the child prone or side-lying to facilitate drainage. 2. Have suction equipment available, but do not suction unless there is an airway obstruction. 3. Monitor for signs of bleeding (frequent swallowing may indicate bleeding); if bleeding occurs, turn the child to the side and notify the physician. 4. Discourage coughing, clearing the throat, or nose blowing to prevent bleeding. 5. Provide an ice collar or analgesics (rectally or intravenously) for discomfort. 6. Administer antiemetics to prevent vomiting if prescribed. 7. Provide clear, cool, noncitrus and noncarbonated fluids (crushed ice, ice pops). 8. Avoid red, purple, or brown liquids, which simulate the appearance of blood if the child vomits. 9. Avoid milk products such as milk, ice cream, and pudding initially because they coat the throat causing the child to cough to clear the throat. 10. Soft foods may be prescribed 1 to 2 days postoperatively. 11. Do not give the child any straws, forks, or sharp objects that can be put into the mouth. 12. Mouth odor, slight ear pain, and a low-grade fever may occur for a few days postoperatively, but the parents should be instructed to notify the physician if bleeding, persistent earache, or fever occurs. 13. Instruct the parents to keep the child away from crowds until healing has occurred; usually the child is able to resume normal activities after 1 to 2 weeks postoperatively.

A clinic nurse reads the results of a Mantoux test on a 3-year-old child. The results indicate an area of induration measuring 10 mm. The nurse would interpret these results as: 1. Positive 2. Negative 3. Inconclusive 4. Definitive and requiring a repeat test

1. Positive Induration measuring 10 mm or more is considered to be a positive result in children younger than 4 years of age and in children with chronic illness or at high risk for exposure to tuberculosis. A reaction of 5 mm or more is considered to be a positive result for the highest risk groups, such as a child with an immunosuppressive condition or a child with human immunodeficiency virus (HIV) infection. A reaction of 15 mm or more is positive in children 4 years or older without any risk factors.

Encouraging fantasy play and participation by children in their own care is a useful developmental approach for which pediatric age-group? 1. Preschool age (3 to 5 years) 2. Adolescence (10 to 19 years) 3. School age (5 to 10 years) 4. Toddler (1 to 3 years)

1. Preschool age (3 to 5 years) Children in the preschool age-group have a rich fantasy life and think about magic. Combined with their strong concept of self, fantasy play and participation in care can minimize the trauma of being hospitalized. Adolescents should be allowed choices and control. School-age children are modest and need to have their privacy respected. Procedures should be explained to them. Toddlers should be examined in the presence of their parents because they fear separation. Allow choices when possible.

A neonate born 18 hours ago with myelomeningocele over the lumbosacral region is scheduled for corrective surgery. Preoperatively, what is the most important nursing goal? 1. Preventing infection 2. Ensuring adequate hydration 3. Providing adequate nutrition 4. Preventing contracture deformity

1. Preventing infection Preventing infection is the nurse's primary preoperative goal for a neonate with myelomeningocele. Although the other options are relevant for this neonate, they're secondary to preventing infection.

A toddler is admitted to the facility with nephrotic syndrome. The nurse carefully monitors the toddler's fluid intake and output and checks urine specimens regularly with a reagent strip (Labstix). Which finding should the nurse report? 1. Proteinuria 2. Glycosuria 3. Ketonuria 4. Polyuria

1. Proteinuria In nephrotic syndrome, the glomerular membrane of the kidneys becomes permeable to proteins. This results in massive proteinuria, which the nurse can detect with a reagent strip. Nephrotic syndrome typically doesn't cause glycosuria or ketonuria. Because the syndrome causes fluids to shift from plasma to interstitial spaces, it's more likely to decrease urine output than to cause polyuria (excessive urine output).

The nurse should expect a 3-year-old child to be able to perform which action? 1. Ride a tricycle 2. Tie the shoelaces 3. Roller-skate 4. Jump rope

1. Ride a tricycle At age 3, gross motor development and refinement in eye-hand coordination enable a child to ride a tricycle. The fine motor skills required to tie shoelaces and the gross motor skills required for roller-skating and jumping rope develop around age 5.

A child has been diagnosed with acute otitis media of the right ear. Which interventions should the nurse include in the plan of care? Select all that apply. 1. Provide a soft diet. 2. Position the child on the left side. 3. Administer an antihistamine twice daily. 4. Irrigate the right ear with normal saline every 8 hours. 5. Administer acetaminophen (Tylenol) for fever every 4 hours as prescribed and as needed. 6. Instruct the parents about the need to administer the antibiotics for the full course of prescribed therapy.

1. Provide a soft diet. 5. Administer acetaminophen (Tylenol) for fever every 4 hours as prescribed and as needed. 6. Instruct the parents about the need to administer the antibiotics for the full course of prescribed therapy. Acute otitis media is an inflammatory disorder caused by an infection of the middle ear. The child often has fever, pain, loss of appetite, and possible ear drainage. The child also is irritable and lethargic and may roll the head or pull on or rub the affected ear. Otoscopic examination may reveal a red, opaque, bulging, and immobile tympanic membrane. Hearing loss may be noted particularly in chronic otitis media. The child's fever should be treated with acetaminophen (Tylenol) or ibuprofen (Motrin). The child is positioned on his or her affected side to facilitate drainage. A soft diet is recommended during the acute stage to avoid pain that can occur with chewing. Antibiotics are prescribed to treat the bacterial infection and should be administered for the full prescribed course. The ear should not be irrigated with normal saline because it can exacerbate the inflammation further. Antihistamines are not usually recommended as a part of therapy.

After gathering all necessary equipment and setting up the supplies, which of the following would be the first step in performing endotracheal (ET) or tracheal suctioning in an infant? 1. Provide extra oxygen by using a ventilator or through manual bagging. 2. Insert a suction catheter to the appropriate measured length. 3. Insert a few drops of sterile saline solution. 4. Put on clean gloves.

1. Provide extra oxygen by using a ventilator or through manual bagging. Providing extra oxygen prior to suctioning helps prevent hypoxemia. Insertion of a suction catheter is performed after preoxygenation. Instilling a few drops of sterile saline solution is no longer part of routine suctioning. ET and tracheal suctioning require sterile technique and sterile gloves, not just clean gloves.

Which interventions are appropriate for the care of an infant? Select all that apply. 1. Provide swaddling. 2. Talk in a loud voice. 3. Provide the infant with a bottle of juice at nap time. 4. Hang mobiles with black and white contrast designs. 5. Caress the infant while bathing or during diaper changes. 6. Allow the infant to cry for at least 10 minutes before responding.

1. Provide swaddling. 4. Hang mobiles with black and white contrast designs. 5. Caress the infant while bathing or during diaper changes. Holding, caressing, and swaddling provide warmth and tactile stimulation for the infant. To provide auditory stimulation, the nurse should talk to the infant in a soft voice and should instruct the mother to do so also. Additional interventions include playing a music box, radio, or television, or having a ticking clock or metronome nearby. Hanging a bright shiny object in midline within 20 to 25 cm of the infant's face and hanging mobiles with contrasting colors, such as black and white, provide visual stimulation. Crying is an infant's way of communicating; therefore the nurse would respond to the infant's crying. The mother is taught to do so also. An infant or child should never be allowed to fall asleep with a bottle containing milk, juice, soda pop, sweetened water, or another sweet liquid because of the risk of nursing (bottle-mouth) caries.

A preschool-age child with sickle cell anemia is admitted to the health care facility in vaso-occlusive crisis after developing a fever and joint pain. What is the nurse's highest priority when caring for this child? 1. Providing fluids 2. Maintaining protective isolation 3. Applying cool compresses to affected joints 4. Administering antipyretics as prescribed

1. Providing fluids During a vaso-occlusive crisis, sickle-shaped red blood cells (RBCs) clump together and obstruct blood vessels, causing ischemia and tissue damage. Providing I.V. and oral fluids promotes hemodilution, which aids the free flow of RBCs through blood vessels. The client must be kept away from known infection sources but doesn't require protective isolation. Warm compresses may be applied to painful joints to promote comfort; cool compresses would cause vasoconstriction, which exacerbates sickling. Antipyretics may be administered to reduce fever but don't play a crucial role in resolving the crisis.

Diagnostic tests for CF

1. Quantitative sweat chloride test is positive. 2. Newborn screening may be done in some states and may consist of immunoreactive trypsinogen analysis and direct DNA analysis for mutant genes. 3. Chest x-ray reveals atelectasis and obstructive emphysema. 4. Pulmonary function tests provide evidence of abnormal small airway function. 5. Stool, fat, enzyme analysis: A 72-hour stool sample is collected to check the fat or enzyme (trypsin) content, or both (food intake is recorded during the collection).

The nurse is interviewing the mother of a 7-year-old child. Which symptom reported by the mother would most lead the nurse to suspect that the child has type 1 diabetes? 1. Recent bed-wetting 2. Poor appetite 3. Weight gain 4. Boundless energy

1. Recent bed-wetting Polyuria is a hallmark of type 1 diabetes mellitus. Parents often notice this symptom as bed-wetting in a child previously toilet-trained. Polyphagia is also a hallmark of type 1 diabetes mellitus. A parent is likely to report that a child eats excessively but seems to be losing weight. The child with type 1 diabetes mellitus may also complain of fatigue.

After an infant undergoes surgical repair of a cleft lip, the physician orders elbow restraints. For this infant, the postoperative care plan should include which nursing action? 1. Removing the restraints every 2 hours 2. Removing the restraints while the infant is asleep 3. Keeping the restraints on both arms only while the child is awake 4. Using the restraints until the infant recovers fully from anesthesia

1. Removing the restraints every 2 hours The nurse should remove one elbow restraint at a time every 2 hours for about 5 minutes, to allow exercise of the arms and to inspect for skin irritation. To prevent the infant from touching and disrupting the suture line, the nurse should use the restraints when the infant is asleep and awake. The nurse should maintain the elbow restraints from the time the infant recovers from anesthesia until the suture line is healed.

Which nursing diagnosis takes highest priority for a child in the early stages of burn recovery? 1. Risk for infection 2. Impaired physical mobility 3. Disturbed body image 4. Constipation

1. Risk for infection Because infection is a serious risk for a client in the early stages of burn recovery, a diagnosis of Risk for infection takes highest priority. Diagnoses of Impaired physical mobility, Disturbed body image, and Constipation may be relevant but take lower priority at this time.

For children from infancy through the preschool years, what is the major stressor posed by hospitalization? 1. Separation from the family 2. Fear of bodily injury 3. Loss of control 4. Fear of pain

1. Separation from the family For infants through preschoolers, separation from the family is the major stressor posed by hospitalization. To minimize the effects of separation, the nurse may suggest that a family member stay with the child as much as possible. Reducing this stressor may help a young child withstand other stressors of hospitalization, such as fear of bodily injury, loss of control, and fear of pain.

A mother of a 3-year-old tells a clinic nurse that the child is rebelling constantly and having temper tantrums . Using Erikson's psychosocial development theory, the nurse tells the mother to: 1. Set limits on the child's behavior. 2. Ignore the child when this behavior occurs. 3. Allow the behavior, because this is normal at this age period. 4. Punish the child every time the child says "no" to change the behavior.

1. Set limits on the child's behavior. According to Erikson, the child focuses on gaining some basic control over self and the environment and independence between ages 1 and 3 years. Gaining independence often means that the child has to rebel against the parents' wishes. Saying things like "no" or "mine" and having temper tantrums are common during this period of development. Being consistent and setting limits on the child's behavior are necessary elements. Options 2 and 3 do not address the child's behavior. Option 4 is likely to produce a negative response during this normal developmental pattern.

A child, age 3, is brought to the emergency department in respiratory distress caused by acute epiglottitis. Which clinical manifestations should the nurse expect to assess? 1. Severe sore throat, drooling, and leaning forward to breathe 2. Low-grade fever, stridor, and a barking cough 3. Pulmonary congestion, a productive cough, and a fever 4. Sore throat, a fever, and general malaise

1. Severe sore throat, drooling, and leaning forward to breathe A child with acute epiglottitis appears acutely ill and clinical manifestations may include drooling (because of difficulty swallowing), severe sore throat, hoarseness, leaning forward with the neck hyperextended, a high fever, and severe inspiratory stridor. A low-grade fever, stridor, and barking cough that worsens at night are suggestive of croup. Pulmonary congestion, productive cough, and fever along with nasal flaring, retractions, chest pain, dyspnea, decreased breath sounds, and crackles are indicative of pneumococcal pneumonia. A sore throat, fever, and general malaise point to viral pharyngitis.

In the family of a 7-year-old child with a chronic illness, which of the following family members feel jealousy, resentment, embarrassment, shame, fear of becoming ill, and guilt at causing the illness? 1. Siblings 2. Parents 3. Child with the illness 4. Grandparents

1. Siblings When a brother or sister is ill, siblings frequently experience jealousy and resentment of the increased attention given to the ill child, embarrassment and shame, fear of becoming ill, and guilt at causing the illness. Parents may experience grieving, denial, overprotectiveness, rejection, and overcompensation. The ill child may regress to a previous developmental stage and feel anxiety, depression, and anger. Both the child's and the siblings' reactions are influenced by the parents' response. Grandparents may experience ambivalence, disappointment, and grief.

The nurse is assessing an 8-month-old during a wellness checkup. Which action is a normal developmental task for an infant this age? 1. Sitting without support 2. Saying two words 3. Feeding himself with a spoon 4. Playing patty-cake

1. Sitting without support According to the Denver Developmental Screening Test, most infants should be able to sit unsupported by age 7 months. A 15-month-old child should be able to say two words. By 17 months, the toddler should be able to feed himself with a spoon. A 10-month-old should be able to play patty-cake.

As an adolescent is receiving care, he's inadvertently injured with a warm compress. The nurse completes an incident report based on the knowledge that identification of which of the following is not a goal of the report? 1. Staff involved so they're reprimanded for their actions 2. Learning needs of staff to prevent recurrence of incidents 3. Patterns of client care problems 4. Facts surrounding each incident

1. Staff involved so they're reprimanded for their actions The main goal of an incident report following an adventitious event isn't punishment for those involved in the incident. The purpose of an incident report is threefold: to identify ways to prevent recurrences of incidents, to identify patterns of care problems, and to identify facts surrounding each incident.

The nurse is caring for a 10-year-old child with rheumatic fever. While obtaining the child's health history from the mother, the nurse should ask if the child recently had which illness? 1. Strep throat 2. Influenza 3. Chickenpox 4. Mononucleosis

1. Strep throat Rheumatic fever often follows an infection with group A beta-hemolytic streptococci, as in strep throat, impetigo, scarlet fever, or pharyngitis. Influenza, chickenpox, and mononucleosis are caused by viruses and don't lead to rheumatic fever.

A 2-year-old child with a tracheostomy suddenly becomes diaphoretic and has an increased heart rate, an increased work of breath, and a decreased oxygen saturation level. Which of the following should be the nurse's first action? 1. Suction the tracheostomy. 2. Turn the child to a side-lying position. 3. Administer pain medication. 4. Perform chest physiotherapy.

1. Suction the tracheostomy. Diaphoresis, increased heart rate, increased respiratory effort, and decreased oxygen saturation are signs that mucus is partially occluding the airway. The child needs suctioning immediately to prevent full occlusion. Turning the child to a side-lying position won't remove mucus from the airway. The child may require pain medication after his airway has been cleared if his condition warrants it. Chest physiotherapy will help drain excess mucus from the lungs but not from a tracheostomy.

The nurse is preparing to administer the first dose of tobramycin (Nebcin) to an adolescent with cystic fibrosis. The order is for 3 mg/kg I.V. daily in three divided doses. The client weighs 95 lb. How many milligrams should the nurse administer per dose?

43.2mg

Respiratory signs and symptoms of CF

1. Symptoms are produced by the stagnation of mucus in the airway, leading to bacterial colonization and destruction of lung tissue. 2. Emphysema and atelectasis occur as the airways become increasingly obstructed. 3. Chronic hypoxemia causes contraction and hypertrophy of the muscle fibers in pulmonary arteries and arterioles, leading to pulmonary hypertension and eventual cor pulmonale. 4. Pneumothorax from ruptured bullae and hemoptysis from erosion of the bronchial wall occur as the disease progresses. 5. Wheezing and cough; Dyspnea; Cyanosis; Clubbing of the fingers and toes; Barrel chest; Repeated episodes of bronchitis and pneumonia

Assessment findings CHF (peds)

1. Tachycardia, especially during rest and slight exertion 2. Tachypnea 3. Profuse scalp diaphoresis, especially in infants 4. Fatigue and irritability 5. Sudden weight gain 6. Respiratory distress

A 10-year-old child diagnosed with acute glomerulonephritis is admitted to the pediatric unit. The nurse should ensure that which of the following is a part of the child's care? 1. Taking vital signs every 4 hours and obtaining daily weight 2. Obtaining a blood sample for electrolyte analysis every morning 3. Checking every urine specimen for protein and specific gravity 4. Ensuring that the child has accurate intake and output and eats a high-protein diet

1. Taking vital signs every 4 hours and obtaining daily weight Because major complications — such as hypertensive encephalopathy, acute renal failure, and cardiac decompensation — can occur, monitoring vital signs (including blood pressure) is an important measure for a child with acute glomerulonephritis. Obtaining daily weight and monitoring intake and output also provide evidence of the child's fluid balance status. Sodium and water restrictions may be ordered depending on the severity of the edema and the extent of impaired renal function. Typically, protein intake remains normal for the child's age and is only increased if the child is losing large amounts of protein in the urine. Checking urine specimens for protein and specific gravity and daily monitoring of serum electrolyte levels may be done, but their frequency is determined by the child's status. These are less important nursing measures in this situation.

A toddler is being prepared for surgery. Who is responsible for obtaining informed consent? 1. The attending physician 2. The floor nurse 3. The operating room nurse 4. The nursing student

1. The attending physician The child's physician is legally responsible for obtaining consent and making sure the parents are well informed. This step includes telling the parents why the child needs the procedure, providing accurate information about the procedure, and explaining the risks involved. The floor nurse may serve as a witness to the parent's signature, and is obligated to inform the physician if the parent doesn't seem informed. The operating room nurse must make sure that the informed consent form has been signed; however, it isn't her responsibility to obtain the consent. Nursing students aren't legally allowed to obtain consent, nor should they act as witnesses.

A mother asks the nurse why her 12-month-old baby gets otitis media more frequently than her 10-year-old son. What should the nurse tell her? 1. The baby's eustachian tubes are shorter and lie more horizontally. 2. The baby is too young to blow his nose when he has a cold. 3. The baby spends more time lying down than his older brother; therefore, more dirt gets in the baby's ear. 4. The baby puts dirty toys in his mouth.

1. The baby's eustachian tubes are shorter and lie more horizontally. Infants and young children are more prone to otitis media because their eustachian tubes are shorter and lie more horizontally. Pathogens from the nasopharynx can more readily enter the eustachian tube of the middle ear. The inability to clear nasal passages by blowing the nose, lying down on the floor, and putting dirty toys in the mouth don't increase the tendency toward otitis media.

A school-age child is being discharged with a diagnosis of rheumatic fever. Which of the following should be included in the teaching plan for the family? 1. The child should stay on penicillin and return for a follow-up appointment. 2. At home, be sure to keep the child on bed rest. 3. All children with rheumatic fever need monthly blood tests. 4. The child should stay out of school until the source of the infection is determined.

1. The child should stay on penicillin and return for a follow-up appointment. A child with rheumatic fever, which is caused by group A beta-hemolytic streptococci, should stay on penicillin — either oral daily or an injection monthly — to prevent a recurrence. A follow-up appointment is needed to determine how the child is responding to treatment. Neither bed rest nor monthly blood tests will be prescribed for all children. Rheumatic fever is caused by group A beta-hemolytic streptococci, so the source of the infection is already known.

A child who was hospitalized for sickle cell crisis is being discharged. Which client outcome demonstrates effective teaching regarding prevention of future crises? 1. The client verbalizes the need to stay away from persons with known infections. 2. The client verbalizes appropriate dietary restrictions. 3. The client verbalizes the need to restrict fluid intake. 4. The client participates in an aerobic exercise program.

1. The client verbalizes the need to stay away from persons with known infections. Preventing infections through proper hand washing and staying away from persons with known infections is an important measure in preventing sickle cell crises. Dietary restrictions aren't significant in preventing these crises. The client should maintain adequate hydration, not restrict fluid intake, and should avoid strenuous activity such as aerobics.

An infant who has been in foster care since birth requires a blood transfusion. Who is authorized to give written, informed consent for the procedure? 1. The foster mother 2. The social worker who placed the infant in the foster home 3. The registered nurse caring for the infant 4. The nurse manager

1. The foster mother When children are minors and aren't emancipated, their parents or designated legal guardians are responsible for providing consent for medical procedures. Therefore, the foster mother is authorized to give consent for the blood transfusion. The social worker, the nurse, and the nurse manager have no legal rights to give consent in this scenario.

A nurse is caring for a 5-year-old child who is in the terminal stages of cancer. Which statements are true? 1. The parents may be at different stages in dealing with the child's death. 2. The child is thinking about the future and knows he may not be able to participate. 3. The dying child may become clingy and act like a toddler. 4. Whispering in the child's room will help the child to cope. 5. The death of a child may have long-term disruptive effects on the family. 6. The child doesn't fully understand the concept of death.

1. The parents may be at different stages in dealing with the child's death. 3. The dying child may become clingy and act like a toddler. 5. The death of a child may have long-term disruptive effects on the family. 6. The child doesn't fully understand the concept of death.

A 13-year-old child may have appendicitis. Which symptoms would help determine the child's condition? 1. The severity, location, and movement of pain 2. The degree of fever 3. A history of vomiting and diarrhea, if present 4. A history of irritability and lethargy

1. The severity, location, and movement of pain The pattern of pain is a reliable indicator of acute appendicitis. It begins with a severe colicky abdominal pain that gets progressively worse. The pain starts in the midabdominal (periumbilical) region and moves to the right lower quadrant after 6 to 12 hours. The degree of fever, a history of vomiting and diarrhea, and a history of irritability and lethargy are all clinical manifestations of acute appendicitis; however, these conditions can also be present in a number of other childhood illnesses.

A nursing instructor asks a nursing student to present a clinical conference to peers regarding Freud's psychosexual stages of development, specifically the anal stage. The student plans the conference, knowing that which of the following most appropriately relates to this stage of development? 1. This stage is associated with toilet training. 2. This stage is characterized by the gratification of self. 3. This stage is characterized by a tapering off of conscious biological and sexual urges. 4. This stage is associated with pleasurable and conflicting feelings about the genital organs.

1. This stage is associated with toilet training. Generally, toilet training occurs during the anal stage. According to Freud, the child gains pleasure from the elimination of feces and from their retention. Option 2 relates to the oral stage. Option 3 relates to the latency period. Option 4 relates to the phallic stage.

Which of the following uses of restraints would be contraindicated in a school-age child? 1. To substitute for observation 2. To ensure the child's comfort or safety 3. To facilitate examination 4. To aid in carrying out procedures

1. To substitute for observation Restraints should never be used as a punishment or as a substitute for observation. If a child is at risk for harming himself when left alone, the child requires one-on-one observation. Ensuring the child's comfort or safety (restraining him to keep an I.V., drainage tube, or orthopedic device in place), facilitating examination, and carrying out procedures are all valid reasons for restraint. Restraining devices aren't without risk and must be checked and documented every 1 to 2 hours.

Before performing an otoscopic examination on a child, where should the nurse palpate for tenderness? 1. Tragus, mastoid process, and helix 2. Helix, umbo, and tragus 3. Tragus, cochlea, and lobule 4. Mastoid process, incus, and malleus

1. Tragus, mastoid process, and helix Before inserting the otoscope, the nurse should palpate the child's external ear, especially the tragus and mastoid process, and should pull the helix backward to determine the presence of pain or tenderness. The umbo, incus, and malleus (parts of the middle ear) and the cochlea (part of the inner ear) aren't palpable.

A physician ordered an X-ray for an adolescent client in the pediatric unit. With whom should the nurse collaborate to carry out this order? 1. Transport personnel 2. Physician 3. Pharmacist 4. Circulating nurse

1. Transport personnel Transport personnel are responsible for escorting clients throughout the hospital, including various test locations. The physician isn't required to transport this client to the radiology department. The pharmacist is responsible for anything related to medications. The circulating nurse assists with surgical procedures in the operating room; she doesn't help transport clients to the X-ray department.

Which situation violates a hospitalized adolescent client's right to confidentiality? 1. Two nurses talk about the adolescent on an elevator on their way to lunch. 2. The adolescent talks about his disease to another client in the hallway. 3. The physician discusses treatment plans with the adolescent in the presence of his mother. 4. The physician discusses a new medication for the adolescent while on the phone with the pharmacist.

1. Two nurses talk about the adolescent on an elevator on their way to lunch. The elevator isn't a secure area in which to talk about clients; anyone could overhear the nurses' conversation. The client isn't breaching his own confidentiality if he volunteers the information. When the client is present for the conversation, he can object at any time to the content of the conversation. Physicians and other health care providers are expected to discuss clients and cases, as long as it's within the context of a professional relationship and is necessary for the course of treatment.

Which I.M. injection site might the nurse use for a 2-year-old child? 1. Ventrogluteal muscle 2. Pectoral muscle 3. Femoral muscle 4. Deltoid muscle

1. Ventrogluteal muscle When administering an I.M. injection to a 2-year-old child, the nurse might select the ventrogluteal muscle if the muscle is well developed. However, the preferred site is the vastus lateralis. The pectoral, femoral, and deltoid muscles aren't appropriate injection sites for a child.

When teaching school-age children important injury prevention strategies, the nurse must use creativity to gain cooperation because children tend not to comply with which of the following? 1. Wearing safety apparel (helmets, knee pads, elbow pads) 2. Learning to swim 3. Saying "no" when offered illegal or dangerous drugs 4. Learning "stranger danger"

1. Wearing safety apparel (helmets, knee pads, elbow pads) School-age children are subject to peer pressure, and they would rather not participate in a sport if they must wear safety apparel that provokes taunts from peers. Therefore, the nurse should discuss stylishness, comfort, and social acceptance because these are major determinants of compliance. School-age children like to swim and may work hard to perfect that skill. This age-group will usually listen to reasons for not taking illegal drugs and will adhere to group rules for not tolerating drug use. Regarding stranger danger, this age-group simply needs to be reminded of potential dangers.

An infant with congestive heart failure is receiving diuretic therapy, and a nurse is closely monitoring the intake and output. The nurse uses which most appropriate method to assess the urine output? 1. Weighing the diapers 2. Inserting a Foley catheter 3. Comparing intake with output 4. Measuring the amount of water added to formula

1. Weighing the diapers Congestive heart failure is the inability of the heart to pump a sufficient amount of oxygen to meet the metabolic needs of the body. The most appropriate method for assessing urine output in an infant receiving diuretic therapy is to weigh the diapers. Comparing intake with output would not provide an accurate measure of urine output. Measuring the amount of water added to formula is unrelated to the amount of output. Although Foley catheter drainage is most accurate in determining output, it is not the most appropriate method in an infant and places the infant at risk for infection.

A toddler is brought to the emergency department with sudden onset of abdominal pain, vomiting, and stools that look like red currant jelly. To confirm intussusception, the suspected cause of these findings, the nurse expects the physician to order: 1. a barium enema. 2. suprapubic aspiration. 3. nasogastric (NG) tube insertion. 4. indwelling urinary catheter insertion.

1. a barium enema. A barium enema commonly is used to confirm and correct intussusception. Performing a suprapubic aspiration or inserting an NG tube or an indwelling urinary catheter wouldn't help diagnose or treat this disorder.

A 2-year-old child is brought to the emergency department with a history of upper airway infection that has worsened over the last 2 days. The nurse suspects the child has croup. Signs of croup include a hoarse voice, inspiratory stridor, and: 1. a barking cough. 2. a high fever. 3. sudden onset. 4. dysphagia.

1. a barking cough. Croup is an acute viral respiratory illness characterized by a barking cough. Fever is usually low grade. Croup has a gradual onset, and dysphagia isn't a symptom.

The nurse on the adolescent unit delegates a task to the nursing assistant. After delegating the task, the nurse should: 1. allow adequate time for the task to be completed, then follow-up with the nursing assistant. 2. document in the chart that the task has been completed. 3. keep asking the nursing assistant if the task has been completed. 4. assume the task has been completed to her satisfaction.

1. allow adequate time for the task to be completed, then follow-up with the nursing assistant. The nurse remains accountable for all of the client's care, including that which has been delegated to the nursing assistant. The nurse should allow the nursing assistant ample time to complete the task, then follow-up with the assistant to make sure the task was completed. Documentation occurs after the task has been completed satisfactorily. Once the task has been delegated, it's important to allow your team members the authority to complete the assigned task. However, the nurse should follow-up with the nursing assistant to make sure the task was completed satisfactorily; the nurse can't assume that it was.

A toddler with a ventricular septal defect (VSD) is receiving digoxin (Lanoxin) to treat heart failure. The nurse should monitor the child for early signs of digoxin toxicity, which include: 1. bradycardia. 2. tachycardia. 3. hypertension. 4. hyperactivity.

1. bradycardia. Digoxin enhances cardiac efficiency by increasing the force of contraction and decreasing the heart rate. An early sign of digoxin toxicity is bradycardia (an abnormally slow heart rate). To help detect digoxin toxicity, the nurse always should measure the apical heart rate before administering each digoxin dose. Other signs and symptoms of digoxin toxicity include arrhythmias, vomiting, hypotension, fatigue, drowsiness, and visual halos around objects. The other options aren't associated with digoxin toxicity.

When performing a physical assessment on a girl, age 10, the nurse keeps in mind that the first sign of sexual maturity in girls is: 1. breast bud development. 2. pubic hair. 3. axillary hair. 4. menarche.

1. breast bud development. Breast bud development — elevation of the nipple and areola to form a breast bud — is the first sign of sexual maturity in girls. Sexual maturation continues with the appearance of pubic hair, axillary hair, and menarche, consecutively.

A nurse on the pediatric floor is caring for a toddler. The nurse should keep in mind that toddlers: 1. express negativism. 2. have reliable verbal responses to pain. 3. have a good concept of danger. 4. have little fear.

1. express negativism. A toddler's increasing autonomy is commonly expressed by negativism. They're unreliable in expressing pain — they respond just as strongly to painless procedures as they do to painful ones. They have little concept of danger and have common fears.

The nurse discovers a 5-year-old child who's unresponsive, apneic, and pulseless. The correct sequence of events that should follow is: 1. call for help, open the airway, provide two rescue breaths, and begin compressions at a rate of 100 per minute. 2. open the airway, provide two rescue breaths, and begin compressions at a rate of 100 per minute. 3. call for help, open the airway, provide two rescue breaths, and begin compressions at a rate of 80 per minute. 4. call for help, continue to attempt to arouse, and assess for breathlessness and lack of pulse until a second rescuer arrives.

1. call for help, open the airway, provide two rescue breaths, and begin compressions at a rate of 100 per minute. The nurse should call for help, open the airway, provide two rescue breaths, begin compressions at a rate of 100 per minute, give one breath for every five compressions, continue for approximately 1 minute, and reassess. This is the accepted sequence defined by the American Heart Association for one-rescuer child cardiopulmonary resuscitation (CPR). Calling for help should be the first action to ensure that assistance arrives quickly. The accepted sequence of events for one-rescuer adult CPR is to call for help, open the airway, provide two rescue breaths, begin compressions at a rate of 80 to 100 per minute, give two breaths for every 15 compressions, continue for approximately 1 minute, and reassess. As soon as unresponsiveness, breathlessness, or lack of pulse has been established, CPR should begin immediately.

An otherwise-healthy adolescent has meningitis and is receiving I.V. and oral fluids. The nurse should monitor this client's fluid intake because fluid overload may cause: 1. cerebral edema. 2. dehydration. 3. heart failure. 4. hypovolemic shock.

1. cerebral edema. Because of the inflammation of the meninges, the client is vulnerable to developing cerebral edema and increased intracranial pressure. Fluid overload won't cause dehydration. It would be unusual for an adolescent to develop heart failure unless the overhydration was extreme. Hypovolemic shock would occur with an extreme loss of fluid or blood.

A 12-month-old child fell down the stairs and a basilar skull fracture is suspected. The nurse should look for: 1. cerebrospinal fluid otorrhea. 2. deafness. 3. raccoon eyes. 4. Battle's sign.

1. cerebrospinal fluid otorrhea. Basilar skull fracture is a fracture in any bone of the base of the skull — frontal, ethmoid, sphenoid, temporal, or occipital. Otorrhea would be observed. Deafness doesn't commonly occur as a result of skull fracture. Battle's sign and raccoon eyes occur primarily in orbital fractures.

The physician prescribes corticosteroids for a child with systemic lupus erythematosus (SLE). The nurse knows that the purpose of corticosteroid therapy for this child is to: 1. combat inflammation. 2. prevent infection. 3. prevent platelet aggregation. 4. promote diuresis.

1. combat inflammation. Corticosteroids are used to combat inflammation in a child with SLE. To prevent infection, the physician would prescribe antibiotics. Aspirin is used to prevent platelet aggregation. Diuretics aren't indicated in SLE.

A mist tent contains a nebulizer that creates a cool, moist environment for an infant with an upper respiratory tract infection. The cool humidity helps the infant breathe by: 1. decreasing respiratory tract edema. 2. avoiding anxiety. 3. drying secretions. 4. increasing fluid intake.

1. decreasing respiratory tract edema. The mist tent decreases respiratory tract edema, which causes croup. However, the child needs to be prepared because the confinement can cause high anxiety. The tent liquefies secretions, rather than drying them, and it doesn't increase the infant's fluid intake.

A child, age 6, is anxious and upset before a scheduled bone marrow aspiration. During client preparation, the nurse should keep in mind that: 1. describing what the child will hear, see, smell, and feel will help the child cope with the procedure. 2. the child's anxiety will decrease with each successive procedure. 3. no small detail about the procedure should go unexplained. 4. explaining bone marrow function will help the child understand the reason for the procedure.

1. describing what the child will hear, see, smell, and feel will help the child cope with the procedure. Children cope with situations better when they can anticipate sensations rather than just trying to comprehend technical explanations. Commonly, a child's anxiety increases rather than decreases with each successive procedure. A school-age child can't assimilate every detail. A 6-year-old can't understand an explanation of bone marrow function; also, such an explanation would be irrelevant.

A mother tells the nurse that she wants to begin toilet training her 22-month-old child. The most important factor regarding toilet training that the nurse should stress to her is: 1. developmental readiness of the child. 2. consistency in approach. 3. the mother's positive attitude. 4. developmental level of the child's peers.

1. developmental readiness of the child. If the child isn't developmentally ready, both child and parent will become frustrated. Consistency is important when toilet training is started. The mother's positive attitude is important when the child is determined to be ready. Developmental levels of children are individualized and comparison to peers isn't useful.

After a child has a cardiopulmonary arrest, which drug would the nurse expect to administer during the initial phase of treatment? 1. dopamine 2. epinephrine 3. sodium bicarbonate 4. atropine

1. dopamine After successful resuscitation, dopamine would be given as an infusion to increase cardiac output and maintain blood pressure. Epinephrine, sodium bicarbonate, and atropine are first-round drugs in a cardiopulmonary arrest.

An adolescent with well-controlled type 1 diabetes has assumed complete management of the disease and wants to participate in gymnastics after school. To ensure safe participation, the nurse should instruct the child to adjust the therapeutic regimen by: 1. eating a snack before each gymnastics practice. 2. measuring the urine glucose level before each gymnastics practice. 3. measuring the blood glucose level after each gymnastics practice. 4. increasing the morning dosage of intermediate-acting insulin.

1. eating a snack before each gymnastics practice. Because exercise decreases the blood glucose level, the nurse should instruct the child to eat a snack before engaging in physical activity to prevent a hypoglycemic episode. Option 2 is incorrect because the urine glucose level doesn't reflect the current blood glucose level. To prevent hypoglycemia, the blood glucose level should be measured before the activity, making option 3 incorrect. Option 4 may lead to hypoglycemia during gymnastics practice; to avoid this condition, the child may need to decrease, not increase, the morning dosage of intermediate-acting insulin.

A child, age 3, is hospitalized for treatment of Kawasaki disease. During the acute phase of this disease, the nurse must assess the child frequently for: 1. heart failure. 2. kidney failure. 3. desquamation of the hands and feet. 4. hepatitis.

1. heart failure. Kawasaki disease, which affects young children, is characterized by acute systemic vasculitis. Myocarditis, a major complication of this disease, commonly causes left-sided heart failure. Therefore, the nurse must monitor the client's weight, fluid intake and output, and vital signs closely for evidence of heart failure. Kidney failure and hepatitis aren't associated with Kawasaki disease. Desquamation of the hands and feet typically occurs after the acute disease phase ends and doesn't pose a danger.

A child, age 8, is immobilized with a hip spica cast. To minimize the child's feelings of isolation, the nurse should: 1. let the child visit the playroom daily. 2. sit with the child for an hour in the room. 3. place a telephone in the child's room. 4. arrange a visit by a cooperative child from the same unit.

1. let the child visit the playroom daily. School-age children need peer interaction and thrive on peer approval and acceptance. Allowing the child to visit the playroom daily provides a nonthreatening atmosphere for peer interaction and helps the child feel less isolated. Sitting with the child for an hour wouldn't foster the necessary peer interaction. Placing a telephone in the child's room would allow the child to communicate with family and friends but could reinforce feelings of isolation. Having another child visit would be appropriate only if the child is of the same age-group.

While examining a 2-year-old child, the nurse sees that the anterior fontanel is open. The nurse should: 1. notify the physician. 2. look for other signs of abuse. 3. recognize this as a normal finding. 4. ask about a family history of Tay-Sachs disease.

1. notify the physician. Because the anterior fontanel normally closes between ages 12 and 18 months, the nurse should notify the physician promptly of this abnormal finding. An open fontanel doesn't indicate abuse and isn't associated with Tay-Sachs disease.

A healthy, 6-month-old infant is brought to the well-baby clinic for a checkup. When assessing the infant's anterior fontanel, the nurse expects it to be: 1. open. 2. sunken. 3. closed. 4. bulging.

1. open. The anterior fontanel is open in a healthy, 6-month-old infant. Normally, it closes between ages 9 and 18 months. It should feel flat and firm. A sunken fontanel indicates dehydration. Although coughing or crying may cause temporary bulging, persistent bulging and tenseness of the fontanel signal increased intracranial pressure.

A 7-year-old boy is hospitalized with cystic fibrosis. To help him manage secretions and avoid respiratory distress, the nurse should: 1. perform chest physiotherapy every 4 hours. 2. give pancreatic enzymes as ordered. 3. place the child in an oxygen tent and have oxygen administered continuously. 4. serve a high-calorie diet.

1. perform chest physiotherapy every 4 hours. Chest physiotherapy aids in loosening secretions in the entire respiratory tract. Pancreatic enzymes aid in the absorption of necessary nutrients — not in managing secretions. Oxygen therapy doesn't aid in loosening secretions and can cause carbon dioxide retention and respiratory distress in children with cystic fibrosis. A high-calorie diet is appropriate but doesn't facilitate respiratory effort.

A child is admitted to the pediatric unit with a fracture of the hip. The physician orders Russell traction. This type of traction is: 1. skin traction applied to a lower extremity, with the extremity suspended above the bed. 2. skeletal traction applied to a lower extremity. 3. skin traction applied to an extended lower extremity. 4. skin traction applied bilaterally to the lower extremities.

1. skin traction applied to a lower extremity, with the extremity suspended above the bed. Russell traction is skin traction applied to a lower extremity, with the extremity suspended above the bed and a sling placed under the knee. Skeletal traction applied to a lower extremity is called 90-90 traction. Skin traction applied to an extended lower extremity is called Buck's extension traction. Skin traction applied bilaterally to the lower extremities is called Bryant's traction.

Several children in a kindergarten class have been treated for pinworm. To prevent the spread of pinworm, the school nurse meets with the parents and explains that they should: 1. tell the children not to bite their fingernails. 2. not let children share hairbrushes. 3. tell the children to cover their mouths and noses when they cough or sneeze. 4. have their children immunized.

1. tell the children not to bite their fingernails. Pinworms come out of the intestine through the anus at night to lay eggs, causing perianal itching. The child wakes up and may begin scratching. Eggs under the fingernails are carried to the mouth if the child chews on his nails, and the life cycle of the pinworm continues. In addition to teaching children not to bite their fingernails, parents should keep the nails short and encourage hand washing before food preparation and eating. Sharing hairbrushes contributes to the spread of head lice, not pinworms. Although covering the mouth and nose are hygienic practices to reduce the spread of infections from respiratory droplets, doing so doesn't affect the spread of pinworms. There are no immunizations to protect against pinworms.

A parent calls the pediatric clinic to express concern over her child's eating habits. She says the child eats very little and consumes only a single type of food for weeks on end. The nurse knows that this behavior is characteristic of: 1. toddlers. 2. preschool-age children. 3. school-age children. 4. adolescents.

1. toddlers. Erratic eating is typical of toddlers. The physiologic need for food decreases at about age 18 months as growth declines from the rapid rate of infancy. The toddler also develops strong food and taste preferences, sometimes eating just one type of food for days or weeks and then switching to another. The child shouldn't be forced to eat. Typically, the child switches to another food spontaneously after a while, correcting any nutritional imbalances. Parents may encourage the child to eat other foods by offering items from the various food groups at each meal.

If an infant's I.V. access site is in an extremity, the nurse should: 1. use a padded board to secure the extremity. 2. restrain all four extremities. 3. restrain the extremity to the bed's side rail. 4. allow the extremity to be loose.

1. use a padded board to secure the extremity. A padded board is adequate to secure the extremity. Restraining all four extremities can be harmful and uncomfortable for the child. Restraining the extremity to the bed's side rail limits the child's movement; the child may bang against the rail and cause injury. Allowing the extremity to be loose increases the risk that the I.V will infiltrate or be dislodged by the infant.

When telling a 4-year-old child about an upcoming procedure, the nurse's most important consideration is to: 1. use simple terms. 2. speak loudly and clearly. 3. offer a toy to keep the child happy. 4. include every detail.

1. use simple terms. When explaining a procedure to a 4-year-old child, the nurse must use simple terms that the child can understand. Speaking loudly may provoke anxiety. Distracting the child with a toy is more appropriate during the procedure than before it. Because preschoolers have a limited attention span, the nurse should provide only the necessary basic facts to prevent anxiety.

A 4-year-old child is receiving dextrose 5% in water and half-normal saline solution at 100 ml/hour. The nurse should suspect that the child's I.V. fluid intake is excessive if assessment reveals: 1. worsening dyspnea. 2. gastric distention. 3. nausea and vomiting. 4. a temperature of 102° F (38.9° C).

1. worsening dyspnea. Dyspnea and other signs of respiratory distress signify fluid volume overload, which can occur quickly in a child as fluid shifts rapidly between the intracellular and extracellular compartments. Gastric distention suggests excessive oral fluid intake or infection. Nausea and vomiting or an elevated temperature may indicate a fluid volume deficit.

A school-age child experiences symptoms of excessive polyphagia, polyuria, and weight loss. The physician diagnoses type I diabetes mellitus and admits the child to the facility for insulin regulation. The physician prescribes an insulin regimen of insulin (Humulin R) and isophane insulin (Humulin N) administered subcutaneously. How soon after administration can the nurse expect the regular insulin to begin to act? 1. ½ to 1 hour 2. 1 to 2 hours 3. 4 to 8 hours 4. 8 to 10 hours

1. ½ to 1 hour Regular insulin, a rapid-acting insulin, begins to act in ½ to 1 hour, reaches peak concentration levels in 2 to 10 hours, and has a duration of action of 5 to 15 hours.

Piaget - Formal operations stage

11 years to adulthood The person is able to think abstractly and logically. Logical thinking is expanded to include solving abstract and concrete problems.

Erikson - adolescence

12 - 20 years Identity vs. role confusion Task: Developing sense of identity Successful: Sense of personal identity Unsuccessful: Confusion about who one is; identity submerged in relationships or group memberships

The nurse expects to observe an infant transferring an object from one hand to another at which age? 1. 4 months 2. 6 months 3. 9 months 4. 12 months

2. 6 months An infant typically transfers objects from one hand to another between ages 6 and 7 months. The infant can grasp a rattle in the hands at age 4 months, bang objects together between ages 9 and 10 months, and place objects in a container by age 12 months.

Erikson - Early childhood

18mo - 3 years Autonomy vs. shame and doubt Task: Gaining some basic control over self and environment Successful: Sense of self-control and adequacy; will power Unsuccessful: Independence-fear conflict; sever feelings of self doubt

Piaget - Preoperational stage

2 to 7 years The child learns to think in terms of past, present, and future. The child moves from knowing the world through sensation and movement to prelogical thinking and finding solutions to problems. The child is egocentric. The child is unable to conceptualize and requires concrete examples.

What is a normal systolic blood pressure for a 3-year-old child? 1. 100 mm Hg 2. 93 mm Hg 3. 120 mm Hg 4. 60 mm Hg

2. 93 mm Hg The formula for normal systolic blood pressure in children ages 1 to 7 is the child's age in years plus 90; therefore, normal systolic blood pressure for a 3-year-old child is 93 mm Hg.

A mother is concerned that she might be spoiling her 2-month-old daughter by picking her up each time she cries. Which suggestion should the nurse offer? 1. "If the baby's diaper is dry when she's crying, leave her alone and she'll fall asleep." 2. "Continue to pick her up when she cries because young infants need cuddling and holding to meet their needs." 3. "Leave your baby alone for 10 minutes. If she hasn't stopped crying by then, pick her up." 4. "Crying at this age indicates hunger. Try feeding her when she cries."

2. "Continue to pick her up when she cries because young infants need cuddling and holding to meet their needs." The nurse should advise the mother to continue to pick the infant up when she cries because a young infant needs to be cuddled and held when crying. Because the infant's cognitive development isn't advanced enough for her to associate crying with getting attention, it would be difficult to spoil her at this age. Even if her diaper is dry, a gentle touch may be necessary until she falls asleep. Crying for 10 minutes wears an infant out; ignoring crying can make the infant mistrust caregivers and the environment. Infants cry for many reasons, not just when hungry.

A 10-month-old child with recurrent otitis media (middle ear inflammation) is brought to the clinic for evaluation. To help determine the cause of the child's condition, the nurse should ask the parents: 1. "Does water ever get into the baby's ears during shampooing?" 2. "Do you give the baby a bottle to take to bed?" 3. "Have you noticed a lot of wax in the baby's ears?" 4. "Can the baby combine two words when speaking?"

2. "Do you give the baby a bottle to take to bed?" In a young child, the eustachian tube is relatively short, wide, and horizontal, promoting drainage of secretions from the nasopharynx into the middle ear. If the child takes a bottle to bed and drinks while lying down, fluids may pool in the pharyngeal cavity, increasing the risk of otitis media. Cerumen in the external ear canal doesn't promote the development of otitis media. However, during shampooing, water may become trapped in the external ear canal by large amounts of cerumen, possibly causing otitis externa (external ear inflammation). Persistent fluid in the middle ear may impair language development and hearing; however, a 10-month-old child isn't expected to combine two words when speaking.

A toddler is diagnosed with iron deficiency anemia. When teaching the parents about using supplemental iron elixir, the nurse should provide which instruction? 1. "Give the iron preparation with milk." 2. "Give the elixir with water or juice." 3. "Monitor the child for episodes of diarrhea." 4. "Give the iron preparation before meals."

2. "Give the elixir with water or juice." Because iron preparations may stain the teeth, the nurse should instruct the parents to give the elixir with water or juice. The iron preparation shouldn't be given with milk because milk impedes iron absorption. This preparation may darken the stools and cause constipation; parental instruction regarding increased fluid intake and fiber intake can relieve constipation. To prevent GI upset, the nurse should instruct the parents to mix the iron preparation with water or fruit juice and have the child take it with meals. (Giving it with fruit juice may be preferable because vitamin C enhances iron solubility and absorption.)

A school-age child has a fever, joint inflammation, and a nonpruritic rash. Knowing that these are signs of rheumatic fever, the nurse should ask the parents: 1. "Has your child recently been exposed to other children with rheumatic fever?" 2. "Has your child had strep throat recently?" 3. "Does your child have a congenital heart defect?" 4. "Is your child's Haemophilus influenzae vaccine up to date?"

2. "Has your child had strep throat recently?" Group A streptococcal infection typically precedes rheumatic fever. An inflammatory disease, rheumatic fever affects the heart, joints, and central nervous system. It isn't infectious and can't be transmitted from one person to another. Congenital heart defects don't play a role in the development of rheumatic fever. H. influenzae vaccine doesn't prevent streptococcal infection or rheumatic fever.

A nurse has provided home care instructions to the mother of a child who is being discharged after cardiac surgery. Which statement made by the mother indicates a need for further instructions? 1. "A balance of rest and exercise is important." 2. "I can apply lotion or powder to the incision if it is itchy." 3. "Activities in which my child could fall need to be avoided for 2 to 4 weeks." 4. "Large crowds of people need to be avoided for at least 2 weeks after surgery."

2. "I can apply lotion or powder to the incision if it is itchy." The mother should be instructed that lotions and powders should not be applied to the incision site after cardiac surgery. Lotions and powders can irritate the surrounding skin, which could lead to skin breakdown and subsequent infection of the incision site. Options 1, 3, and 4 are accurate instructions regarding home care after cardiac surgery.

The nurse is teaching the mother of a 5-month-old infant diagnosed with bronchiolitis. Which statement by the mother indicates that teaching has been effective? 1. "I hope my baby will come home from the hospital." 2. "I know that this disease is serious and can lead to asthma." 3. "My baby needs to be cured this time so it won't happen again." 4. "My baby has been sick. This machine helps him breathe."

2. "I know that this disease is serious and can lead to asthma." Bronchiolitis places the child at risk for developing asthma. If diagnosed and treated promptly, most infants recover from the illness and return home. Infants typically don't have recurrences of bronchiolitis. Infants diagnosed with bronchiolitis rarely require mechanical ventilation.

During chemotherapy for lymphoma, a child, age 15, is at risk for stomatitis. Which statement by the child supports a nursing diagnosis of Deficient knowledge related to mouth care? 1. "I use a soft toothbrush to clean my teeth." 2. "I remove white patches on my tongue and cheeks with my toothbrush." 3. "I rinse my mouth every 2 to 4 hours with a solution of baking soda and water." 4. "I don't use bottled mouthwashes."

2. "I remove white patches on my tongue and cheeks with my toothbrush." White patches on the tongue and oral mucosa indicate infection; the client should report, not remove, them. The child should use a soft toothbrush to prevent injury to the fragile oral mucosa. To prevent stomatitis, the child should rinse the mouth every 2 to 4 hours with a nonirritating solution, such as baking soda and water or normal saline solution, and should avoid commercial mouthwashes containing alcohol, which may dry the oral mucosa.

The mother of several young children calls the nurse when her school-age child comes down with chickenpox. The nurse provides instruction on communicability and home management of this disease. Which response by the mother indicates effective teaching? 1. "I should keep my child at home until the fever is gone." 2. "I should have my child soak in oatmeal baths twice daily." 3. "I should give my child aspirin every 4 hours until the fever is gone." 4. "I should start checking my other children for lesions in about 4 weeks."

2. "I should have my child soak in oatmeal baths twice daily." Chickenpox is characterized by pruritic lesions; colloidal oatmeal baths may soothe the skin and relieve itching. A fever is common during the first 24 hours. However, the communicable period extends beyond the febrile stage and a normal temperature shouldn't be used as the basis for letting the child leave home. Chickenpox is communicable from 1 day before the lesions erupt until they dry — approximately 1 week. The child should stay home during this time to prevent disease transmission. Aspirin isn't recommended because it's associated with Reye's syndrome; acetaminophen is a suitable substitute. The incubation period for chickenpox is 2 to 3 weeks; the mother should begin to check the other children for lesions 2 weeks after exposure to the infected child.

A teenage mother brings her 1-year-old child to the pediatrician's office for a well-baby checkup. She says that her baby can't sit alone or roll over. An appropriate response by the nurse would be: 1. "This is very abnormal. Your child must be sick." 2. "Let's see about further developmental testing." 3. "Don't worry, this is normal for her age." 4. "Maybe you just haven't seen her do it."

2. "Let's see about further developmental testing." At age 12 months a child should be sitting up and rolling over. Therefore, this child may have developmental problems. Options 1 and 4 aren't therapeutic and can cut off communication with the mother. Option 3 misleads the mother with false reassurance.

After being hospitalized for status asthmaticus, a child, age 5, is discharged with prednisone (Deltasone) and other oral medications. Two weeks later, when the child comes to the clinic for a checkup, the nurse instructs the mother to gradually decrease the dosage of prednisone, which will be discontinued. The mother asks why prednisone must be discontinued. How should the nurse respond? 1. "Steroids increase the appetite, leading to obesity with prolonged use." 2. "Long-term steroid therapy may interfere with a child's growth." 3. "The child may develop a hypersensitivity to steroids with continued use." 4. "Prolonged steroid use may cause depression."

2. "Long-term steroid therapy may interfere with a child's growth." Steroids suppress release of adrenocorticotropic hormone from the pituitary gland, stopping production of endogenous hormones by the adrenal cortex. Because prolonged adrenal suppression may cause growth retardation in a child, the duration and dosage of steroid therapy must be kept to a minimum. Steroids also may cause central nervous system effects, such as euphoria, insomnia, and mood swings. Although they increase the appetite, this isn't the reason for limiting their use in children. Steroids are present in the body, so hypersensitivity isn't a problem. They're likely to cause euphoria, not depression.

A boy, age 4, begins to use curse words. Concerned about this behavior, his parents ask the nurse how to curtail it. Which advice should the nurse offer? 1. "Just ignore it. He'll grow out of it." 2. "Tell him it isn't acceptable and he'll be disciplined if he continues to do it." 3. "Tell him that good little boys don't use curse words." 4. "Tell him that his behavior makes you angry."

2. "Tell him it isn't acceptable and he'll be disciplined if he continues to do it." By explaining their objections and expectations, the parents teach the child why the behavior is unacceptable and help him understand that he must stop it. Options 1 and 4 wouldn't teach the child that his behavior is inappropriate. Option 3 would reinforce the impression that the child is "bad," diminishing his self-image while doing little to change the objectionable behavior.

The mother of a child with sickle cell anemia confides in the nurse that she feels guilty about letting the child run and play with the neighborhood children and that if she had been a better mother, the child wouldn't have suffered a sickle cell crisis. Which response would be most appropriate? 1. "She's just fine now. Don't worry." 2. "Tell me more about how you feel." 3. "But you know that children with sickle cell anemia often have crises." 4. "You shouldn't be so protective of her."

2. "Tell me more about how you feel." Many parents feel guilty when their child is sick. Encouraging parents to talk more about their feelings provides support and helps to develop a therapeutic relationship. Giving a stereotyped answer, such as "Don't worry," shows a lack of interest in what the parent is feeling. Commenting on the course of the disease doesn't address the parent's feelings. Being judgmental or offering an opinion can also block therapeutic communication by inhibiting the parent from discussing her feelings and developing solutions.

A 15-year-old girl with a urinary tract infection is admitted to the facility. She tells the nurse she hopes that she's pregnant. Which of the following would be the best response by the nurse? 1. "Does your mother know about this?" 2. "Tell me what pregnancy would mean to you." 3. "Congratulations. Does the baby's father know?" 4. "I hope you aren't pregnant; you're too young."

2. "Tell me what pregnancy would mean to you." When talking with adolescents, it's best to get their viewpoints and thoughts first. Doing so promotes therapeutic communication. Asking whether the mother knows or about the baby's father focuses the attention away from the adolescent. Making a statement about her being too young to be pregnant is a value judgment and inappropriate.

A child, age 3, is admitted to the pediatric unit with dehydration after 2 days of nausea and vomiting. The mother tells the nurse that her child's illness "is all my fault." How should the nurse respond? 1. "Maybe next time you'll bring the baby in sooner." 2. "Tell me why you think this is your fault." 3. "Try not to cry in front of the child. It'll only upset her." 4. "Don't be so upset. Your child will be fine."

2. "Tell me why you think this is your fault." Many parents feel responsible for their child's illness and may need instruction about the actual cause of the illness. Option 1 could increase the mother's feelings of guilt. Options 3 and 4 ignore her feelings.

During a well-baby visit, a 2-month-old infant receives diphtheria pertussis tetanus (DPT) vaccine, inactivated poliovirus vaccine, hepatitis B vaccine, pneumococcal vaccine, and Haemophilus influenzae b (Hib) vaccine. The parents state that the child's older brother has never received the Hib vaccine and ask why the baby must have it. How should the nurse respond? 1. "This vaccine prevents infection by various strains of the influenza virus." 2. "This vaccine protects against serious bacterial infections, such as meningitis and bacterial pneumonia." 3. "This vaccine prevents infection by the hepatitis B virus." 4. "This vaccine prevents chickenpox."

2. "This vaccine protects against serious bacterial infections, such as meningitis and bacterial pneumonia." The Hib vaccine provides protection against serious childhood infections caused by H. influenzae type B virus, such as meningitis and bacterial pneumonia. The influenza virus vaccine provides immunity to various strains of the influenza virus. The Heptavax vaccine prevents infection by the hepatitis B virus. Chickenpox is caused by the varicella virus; a chickenpox vaccine is now available.

A child, age 6, is about to be discharged after treatment for acute rheumatic fever. Which statement by the parents indicates effective discharge teaching? 1. "We will keep our child in bed for at least a week." 2. "We will give our child penicillin twice per day for 5 years." 3. "We will measure our child's blood pressure every day." 4. "We will keep giving our child corticosteroids."

2. "We will give our child penicillin twice per day for 5 years." A child recovering from acute rheumatic fever must receive prophylactic penicillin for at least 5 years. Bed rest isn't indicated once the acute disease phase ends. Rheumatic fever doesn't call for blood pressure monitoring or corticosteroid therapy.

A 16-year-old client sustains a severe head injury in a motor vehicle accident. He's admitted to the neurologic unit and subsequently develops neurogenic diabetes insipidus. The physician prescribes vasopressin (Pitressin), 5 units subcutaneously (S.C.) twice per day. When vasopressin is given S.C., it begins to act within: 1. 5 minutes. 2. 1 hour. 3. 2 hours. 4. 4 hours.

2. 1 hour. When vasopressin is given S.C., it begins to act within 1 hour. Its duration of action is 2 to 8 hours.

A child has just been admitted to the facility and is displaying fear related to separation from his parents, the room being too dark, being hurt while in the hospital, and having many different staff members come into the room. Based on the nurse's knowledge of growth and development, the child is: 1. 7 to 12 months old (an infant). 2. 1 to 3 years old (a toddler). 3. 6 to 12 years old (a school-age child). 4. 12 to 18 years old (an adolescent).

2. 1 to 3 years old (a toddler). Toddlers show fear of separation from their parents, the dark, loud or sudden noises, injury, strangers, certain persons, certain situations, animals, large objects or machines, and change in environment. Infants show fear of strangers, the sudden appearance of unexpected and looming objects (including people), animals, and heights. School-age children show fear of supernatural beings, injury, storms, the dark, staying alone, separation from parents, things seen on television and in the movies, injury, tests and failure in school, consequences related to unattractive physical appearance, and death. Adolescents show fear of inept social performance, social isolation, sexuality, drugs, war, divorce, crowds, gossip, public speaking, plane and car crashes, and death.

Sudden infant death syndrome (SIDS) is one of the most common causes of death in infants. At what age is the diagnosis of SIDS most likely? 1. 1 to 2 years 2. 1 week to 1 year, peaking at 2 to 4 months 3. 6 months to 1 year, peaking at 10 months 4. 6 to 8 weeks

2. 1 week to 1 year, peaking at 2 to 4 months SIDS can occur anytime between ages 1 week and 1 year. The incidence peaks at ages 2 to 4 months.

When administering total parenteral nutrition (TPN) through a peripheral I.V. line to a school-age child, what is the lowest amount of glucose that is considered safe and not caustic to small veins that will also provide adequate TPN? 1. 5% glucose 2. 10% glucose 3. 15% glucose 4. 17% glucose

2. 10% glucose The amount of glucose that is considered safe for peripheral veins while still providing adequate parenteral nutrition is 10%. A glucose amount of 5% isn't sufficient nutritional replacement, although it's safe for peripheral veins. Any amount above 10% glucose, such as 15% and 17%, must be administered via central venous access.

At what age should boys be taught how to do a monthly testicular self-examination? 1. 8 years old 2. 12 years old 3. 16 years old 4. When they become sexually active

2. 12 years old Testicular cancer occurs most frequently between the ages of 15 and 34; therefore, boys should begin doing testicular self-examinations at age 12, which will help them become familiar with the normal contours and consistency of their genital structures.

The parents of a child with cystic fibrosis, an autosomal recessive disorder, are considering having a second child. Each parent is heterozygous for the cystic fibrosis trait. What is the chance that their second child will manifest the disorder? 1. 0% 2. 25% 3. 50% 4. 100%

2. 25% To manifest, or express, an autosomal recessive disorder, a child must inherit the trait from both parents. A heterozygous person carries one normal gene and one affected gene and doesn't express the disorder. A child of two heterozygous parents has a one-in-four (25%) chance of manifesting an autosomal recessive disorder. Outcomes of previous pregnancies don't influence the probability of subsequent offspring expressing the genetic disorder.

A child with osteomyelitis is to receive nafcillin (Nafcil) I.V. every 6 hours. Before administering the drug, the nurse calculates the appropriate dosage. The recommended dosage is 50 to 100 mg/kg daily; the child weighs 22 lb (10 kg). Which dosage is acceptable? 1. 500 mg every 6 hours 2. 250 mg every 6 hours 3. 100 mg every 6 hours 4. 50 mg every 6 hours

2. 250 mg every 6 hours First, the nurse determines the minimum dose: 50 mg × 10 kg = 500 mg/day 500 mg/4 doses (for administration every 6 hours) = 125 mg/dose. Next, the nurse determines the maximum dose: 100 mg × 10 kg = 1,000 mg/day 1,000 mg/4 doses = 250 mg/dose. Thus, the acceptable dosage range for this client is 125 to 250 mg every 6 hours. Only option 2 falls within this range.

The charge nurse on the pediatric unit informs the staff nurse that four of her clients require attention. Which client should the nurse see first? 1. An 8-year-old client admitted from the postanesthesia care unit who's complaining of pain 2. A 10-year-old client with asthma whose oxygen saturation levels are dropping 3. A 7-year-old client whose mother is waiting for discharge instructions 4. A 9-year-old client with a broken leg who wants help moving from the bed to the chair

2. A 10-year-old client with asthma whose oxygen saturation levels are dropping Decreasing oxygen saturation levels indicate difficulty breathing and increased work of breathing. Airway, breathing, and circulation always take priority. Administration of pain medication and reviewing discharge instructions can be delegated to another registered nurse. Moving a patient from the bed to the chair can be delegated to a nursing assistant.

The nurse is teaching the parents of a 6-month-old infant about usual growth and development. Which statements are true regarding infant development? 1. A 6-month-old infant has difficulty holding objects. 2. A 6-month-old infant can usually roll from prone to supine and supine to prone positions. 3. A teething ring is appropriate for a 6-month-old infant. 4. Stranger anxiety usually peaks at age 12 to 18 months. 5. Head lag is commonly noted in infants at age 6 months. 6. Lack of visual coordination usually resolves by age 6 months.

2. A 6-month-old infant can usually roll from prone to supine and supine to prone positions. 3. A teething ring is appropriate for a 6-month-old infant. 6. Lack of visual coordination usually resolves by age 6 months. Gross motor skills of the 6-month-old infant include rolling from front to back and back to front. Teething usually begins around age 6 months; therefore, a teething ring is appropriate. Visual coordination is usually resolved by age 6 months. At age 6 months, fine motor skills include purposeful grasps. Stranger anxiety normally peaks at 8 months of age. The 6-month-old infant also should have good head control and no longer display head lag when pulled up to a sitting position.

When assessing a child, age 3 months, who has been diagnosed with heart failure, the nurse expects which finding? 1. Bounding peripheral pulses 2. A gallop heart rhythm 3. Widened pulse pressure 4. Bradycardia

2. A gallop heart rhythm Heart failure may cause a gallop rhythm in a child. Bounding peripheral pulses, widened pulse pressure, and bradycardia aren't associated with heart failure.

The nurse is caring for a neonate with congenital clubfoot. The child has a cast to correct the defect. Before discharge, what should the nurse tell the parents? 1. The cast will be removed in 6 weeks. 2. A new cast is needed every 1 to 2 weeks. 3. A short leg cast is applied when the baby is ready to walk. 4. The cast will be removed when the baby begins to crawl.

2. A new cast is needed every 1 to 2 weeks. Because a neonate grows so quickly, the cast may need to be changed as often as every 1 to 2 weeks. A cast for congenital clubfoot isn't left on for 6 weeks because of the rapid rate of the infant's growth. By the time a baby is crawling or ready to walk, the final cast has long since been removed. After the cast is permanently removed, the baby may wear a Denis Browne splint until he's 1 year old.

Which toy would be most appropriate for a 3-year-old? 1. A bicycle 2. A puzzle with large pieces 3. A pull toy 4. A computer game

2. A puzzle with large pieces At age 3, children like to color, draw, and put together puzzles. A bicycle is appropriate for a 5- or 6-year-old child; a pull toy, for a toddler; and a computer game, for a school-age child.

A 4-year-old child has recently been diagnosed with acute lymphocytic leukemia (ALL). What information about ALL should the nurse provide when educating the client's parents? 1. Leukemia is a rare form of childhood cancer. 2. ALL affects all blood-forming organs and systems throughout the body. 3. Because of the increased risk of bleeding, the child shouldn't brush his teeth. 4. Adverse effects of treatment include sleepiness, alopecia, and stomatitis. 5. There's a 95% chance of obtaining remission with treatment. 6. The child shouldn't be disciplined during this difficult time.

2. ALL affects all blood-forming organs and systems throughout the body. 4. Adverse effects of treatment include sleepiness, alopecia, and stomatitis. 6. The child shouldn't be disciplined during this difficult time. In ALL, abnormal white blood cells (WBCs) proliferate, but they don't mature past the blast phase. These blast cells crowd out the healthy WBCs, red blood cells, and platelets in the bone marrow, leading to bone marrow depression. The blast cells also infiltrate the liver, spleen, kidneys, and lymph tissue. Common adverse effects of chemotherapy and radiation include nausea, vomiting, diarrhea, sleepiness, alopecia, anemia, stomatitis, mucositis, pain, reddened skin, and increased susceptibility to infection. There's a 95% chance of obtaining remission with treatment. Leukemia is the most common form of childhood cancer. The child still needs appropriate discipline and limits. A lack of consistent parenting may lead to negative behaviors and fear.

The physician diagnoses leukemia in a child, age 4, who complains of being tired and sleeps most of the day. Which nursing diagnosis reflects the nurse's understanding of the physiologic effects of leukemia? 1. Ineffective airway clearance related to fatigue 2. Activity intolerance related to anemia 3. Imbalanced nutrition: More than body requirements related to lack of activity 4. Ineffective cerebral tissue perfusion related to central nervous system infiltration by leukemic cells

2. Activity intolerance related to anemia A child with leukemia may experience anemia from bone marrow depression, such as from chemotherapy or replacement of normal bone marrow elements by immature white blood cells. Anemia results in fatigue, lack of energy, and activity intolerance. The information given in the question doesn't support the other diagnoses.

An adolescent with type 1 diabetes mellitus is experiencing a growth spurt. Which treatment approach would be most effective for this client? 1. Administering insulin once a day 2. Administering multiple doses of insulin 3. Limiting dietary fat intake 4. Substituting an oral antidiabetic agent for insulin

2. Administering multiple doses of insulin During an adolescent growth spurt, a regimen of multiple insulin doses achieves better control of the blood glucose level because it more closely simulates endogenous insulin release. A single daily dose of insulin wouldn't control this client's blood glucose level as effectively. Limiting dietary fat intake wouldn't help the body use glucose at the cellular level. A child with type 1 diabetes mellitus doesn't produce insulin and therefore can't receive an oral antidiabetic agent instead of insulin.

The nurse is preparing to teach a 13-year-old client with asthma to administer his own breathing treatments. Which principle should the nurse keep in mind when planning the teaching session? 1. Adolescents are unable to follow detailed instructions. 2. Adolescents are worried about appearing different from their peers. 3. Adolescents' fine motor coordination isn't sufficiently developed to administer treatments. 4. Adolescents have a well-developed sense of self-identity.

2. Adolescents are worried about appearing different from their peers. Adolescents have a strong need to belong, and they seek social approval from their peers. Knowing this will help the nurse construct an effective teaching plan. Adolescents are capable of following detailed instructions. According to Piaget, adolescents are at the formal operations stage and are capable of deductive, reflective, and hypothetical reasoning. Fine motor coordination is well developed by adolescence. According to Erikson's stages of psychosocial development, adolescence is the stage of identity versus role confusion. During this stage, the adolescent strives toward establishing a sense of identity.

A 10-year-old client arrives in the emergency department with suspected inhalation anthrax. Which intervention should the nurse perform first? 1. The nurse and other members of the health care team should put on N-95 respirator masks. 2. After obtaining blood cultures, the nurse should insert an I.V. catheter and begin antibiotic and I.V. therapy as ordered. 3. The nurse should move the client to a negative-pressure isolation room. 4. The nurse should prepare to admit the client to a medical-surgical unit.

2. After obtaining blood cultures, the nurse should insert an I.V. catheter and begin antibiotic and I.V. therapy as ordered. Immediate antibiotic administration has been found to lower mortality rates from inhalation anthrax. Supportive care is essential to successful treatment, so the nurse should obtain blood cultures and immediately start an I.V. and antibiotic therapy. Inhalation anthrax is caused by inhalation of aerosolized anthrax spores, and isn't transmitted from human-to-human contact. Although standard precautions should be upheld, the health care team doesn't need special protective equipment, such as an N-95 respirator mask, and the patient doesn't require special isolation, such as a negative-pressure isolation room. Because the client's condition may deteriorate rapidly as anthrax toxins are released into the systemic circulation, he'll most likely require admission to an intensive care unit (not a medical-surgical unit) for monitoring.

The mother of a 3-year-old is concerned because her child still is insisting on a bottle at nap time and at bedtime. Which of the following is the appropriate suggestion to the mother? 1. Allow the bottle if it contains juice. 2. Allow the bottle if it contains water. 3. Do not allow the child to have the bottle. 4. Allow the bottle during naps but not at bedtime.

2. Allow the bottle if it contains water. A toddler should never be allowed to fall asleep with a bottle containing milk , juice, soda pop, sweetened water, or any other sweet liquid because of the risk of nursing (bottle-mouth) caries. If a bottle is allowed at nap time or bedtime, it should contain only water.

The nurse just received the following clients for her shift assignment. Which client should she assess first? 1. A 5-month-old with I.V. fluids infusing 2. An 11-month-old receiving chemotherapy through a central venous catheter 3. An 8-year-old in traction with a femur fracture 4. A 14-year-old postoperative client who has a nasogastric tube and an indwelling urinary catheter

2. An 11-month-old receiving chemotherapy through a central venous catheter The nurse should assess the 11-month-old with a central venous catheter first. This child takes priority because he has an invasive line and is receiving chemotherapy, which may cause toxic effects. Next, the nurse should assess the 5-month-old with an I.V. infusion and then the 14-year-old postoperative client. Because he's the most stable, the nurse can assess the 8-year-old client in traction last.

A 7-year-old child is admitted to the hospital for a course of I.V. antibiotics. What should the nurse do before inserting the peripheral I.V. catheter? 1. Explain the procedure to the child immediately before the procedure. 2. Apply a topical anesthetic to the I.V. site before the procedure. 3. Ask the child which hand he uses for drawing. 4. Explain the procedure to the child using abstract terms. 5. Don't let the child see the equipment to be used in the procedure. 6. Tell the child that the procedure won't hurt.

2. Apply a topical anesthetic to the I.V. site before the procedure. 3. Ask the child which hand he uses for drawing. Topical anesthetics reduce the pain of a venipuncture. The cream should be applied about 1 hour before the procedure and requires a physician's order. Asking which hand the child draws with helps to identify the dominant hand. The I.V. should be inserted into the opposite extremity so that the child can continue to play and do homework with minimal disruption. Younger school-age children don't have the capability for abstract thinking. The procedure should be explained using simple words, and definitions of unfamiliar terms should be provided. The child should have the procedure explained to him well before it takes place so that he has time to ask questions. Although the topical anesthetic will relieve some pain, there's usually some pain or discomfort involved in venipuncture, so the child shouldn't be told otherwise.

In a child recovering from cardiac catheterization, assessment is most likely to reveal which of the following? 1. Rapidly rising blood pressure 2. Arrhythmias 3. Heart failure 4. Hypostatic pneumonia

2. Arrhythmias Arrhythmias may follow cardiac catheterization. Rapidly rising blood pressure, heart failure, and hypostatic pneumonia aren't typical complications of this procedure.

The nurse is caring for an adolescent girl who was admitted to the hospital's medical unit after attempting suicide by ingesting acetaminophen (Tylenol). The nurse should incorporate which interventions into the care plan for this client? 1. Limit care until the client initiates a conversation. 2. Ask the client's parents if they keep firearms in their home. 3. Ask the client if she's currently having suicidal thoughts. 4. Assist the client with bathing and grooming as needed. 5. Inspect the client's mouth after giving oral medications. 6. Assure the client that anything she says will be held in strict confidence.

2. Ask the client's parents if they keep firearms in their home. 3. Ask the client if she's currently having suicidal thoughts. 4. Assist the client with bathing and grooming as needed. 5. Inspect the client's mouth after giving oral medications. Safety is the primary consideration when caring for suicidal clients. Because firearms are the most common method used in suicides, the client's parents should be encouraged to remove firearms from the home, if applicable. Safety also includes assessing for current suicidal ideation. In many cases, suicidal people are depressed and don't have the energy to care for themselves, so the client may need assistance with bathing and grooming. Because depressed and suicidal clients may hide pills in their cheeks, the nurse should inspect the client's mouth after giving oral medications. Rather than limit care, the nurse should try to establish a trusting relationship through nursing interventions and therapeutic communication. The client can't be assured confidentiality when self-destructive behavior is an issue.

Before a routine checkup in the pediatrician's office, an 8-month-old infant sits contentedly on the mother's lap, chewing on a toy. When preparing to examine this infant, what should the nurse plan to do first? 1. Measure the head circumference. 2. Auscultate the heart and lungs. 3. Elicit the pupillary reaction. 4. Weigh the child.

2. Auscultate the heart and lungs. Heart and lung auscultation rarely distresses an infant, so it should be done early in the assessment. Placing a tape measure on the infant's head, shining a light in the eyes, or undressing the infant before weighing may cause distress, making the rest of the examination more difficult.

When assessing a toddler's growth and development, the nurse understands that a child in this age-group displays behavior that fosters which developmental task? 1. Initiative 2. Autonomy 3. Trust 4. Industry

2. Autonomy The toddler's developmental task is to achieve autonomy while overcoming shame and doubt. Developing initiative is the preschooler's task. Developing trust is the infant's task. Developing industry is the task of the school-age child.

Which of the following should the nurse do first when admitting an 11-year-old child in sickle cell crisis? 1. Administer oral pain medication while obtaining the child's history. 2. Begin I.V. fluids after obtaining the child's history. 3. Instruct the parents about what to expect during this hospitalization. 4. Start oxygen therapy as soon as the child's vital signs are taken.

2. Begin I.V. fluids after obtaining the child's history. Fluids are one of the most important components of therapy for sickle cell crisis. Fluids help increase blood volume and prevent sickling and thrombosis. A child experiencing a sickle cell crisis often has severe pain requiring the use of I.V. analgesics such as morphine, which would be administered after fluid therapy has been started. Instructing the parents about what to expect during hospitalization is important but it isn't the first action the nurse should take. Oxygen therapy is used only if the child is hypoxic.

Which of the following car safety devices should be used for a child who is 8 years old and is 4 feet tall? 1. Seat belt 2. Booster seat 3. Rear-facing convertible seat 4. Front-facing convertible seat

2. Booster seat Children should remain in a booster seat until they are 8 to 12 years old and at least 4 feet, 9 inches tall . Infants should ride in a car in a semireclined, rear-facing position in an infant-only seat or a convertible seat until they weigh at least 20 pounds and are at least 1 year of age. The transition point for switching to the forward-facing position is defined by the manufacturer of the convertible car safety seat but is generally at a body weight of 9 kg (20 pounds) and 1 year of age.

A preschool child presents with a history of vomiting and diarrhea for 2 days. Which assessment finding indicates that the child is in the late stages of shock? 1. Tachycardia 2. Bradycardia 3. Irritability 4. Urine output 1 to 2 ml/kg/hour

2. Bradycardia Bradycardia is a sign of late shock in a pediatric client. Cardiovascular dysfunction and impairment of cellular function lead to lowered perfusion pressures, increased precapillary arteriolar resistance, and venous capacitance. Decreased cardiac output occurs in late shock if the circulating volume isn't replaced. Sympathetic nervous innervation has limited compensation mechanisms if the volume isn't replaced. Tachycardia and irritability occur during the early phase of shock as compensatory mechanisms are implemented to increase cardiac output. Normal urine output for a pediatric client is 1 to 2 ml/kg/hour; volumes less than this would indicate a decrease in renal perfusion and activation of the renin-angiotensin-aldosterone system to decrease water and sodium excretion.

The nurse caring for an 8-month-old infant diagnosed with respiratory syncytial virus is unable to read a medication dosage written in the infant's medical record. What is the only ethical and responsible solution for the nurse? 1. Erase the original order and rewrite it more clearly. 2. Call the physician and ask for a verbal order to clarify the dosage. 3. Ask another nurse what she thinks the dosage should be. 4. Ask the mother what dosage the infant takes at home.

2. Call the physician and ask for a verbal order to clarify the dosage. Clarification of written orders must come from the physician or health care provider who wrote the order. A verbal order should be obtained and then entered into the medical chart on a separate line. Assuming or guessing what the writer intended could lead to a medication error. Medical charts are legal documents; information should never be altered or erased. The nurse shouldn't ask the mother; the mother may not be reliable and the physician may have prescribed a different dose during hospitalization.

The nurse is teaching the parents of a 5-year-old child how to respond in case of poisoning. If poisoning occurs, what should the parents' first response be? 1. Administer syrup of ipecac. 2. Call the poison control center. 3. Bring the child to the physician's office. 4. Monitor the child for adverse effects.

2. Call the poison control center. If parents believe that their child has ingested a poison, they should first call the poison control center. Administering syrup of ipecac is no longer recommended. The parents should call the poison control center before transporting the child to a physician. The poison control center may recommend monitoring the child for adverse effects, but parents shouldn't make this decision on their own.

Which method is most reliable for identifying a preschooler before administering a medication? 1. Check the name on the bed. 2. Check the hospital identification bracelet. 3. Ask the child his name. 4. Ask the parents at the bedside.

2. Check the hospital identification bracelet. The only safe method for identifying the child is to check the identification band for the client's name and medical record number and then compare that information with the medication record. Children sometimes exchange beds during play, so checking the name on the bed isn't reliable. Infants are unable to give their names, toddlers or preschoolers may admit to any name, and school-age children may deny their identities in an attempt to avoid the medication. Parents aren't always at the bedside, so they shouldn't be relied on for identification.

When teaching the parent of a school-age child about signs and symptoms of fever that require immediate notification of the physician, which description should the nurse include? 1. Burning or pain with urination 2. Complaints of a stiff neck 3. Fever disappearing for longer than 24 hours, then returning 4. History of febrile seizures

2. Complaints of a stiff neck A child with a fever and a stiff neck should be evaluated immediately for meningitis. All other symptoms should be addressed by the physician but can wait until office hours.

In a pediatric client, what is an early sign of acute renal failure (ARF)? 1. Hypertension 2. Decreased urine output 3. Anemia 4. Hematuria

2. Decreased urine output A decreased urine output (oliguria) is an early sign of ARF. Hypertension and anemia occur later in ARF. Hematuria is rare.

A 10- year-old child with asthma is treated for acute exacerbation in the emergency department. A nurse caring for the child monitors for which of the following, knowing that it indicates a worsening of the condition? 1. Warm, dry skin 2. Decreased wheezing 3. Pulse rate of 90 beats/ min 4. Respirations of 18 breaths/ min

2. Decreased wheezing Asthma is a chronic inflammatory disease of the airways. Decreased wheezing in a child with asthma may be interpreted incorrectly as a positive sign when it may actually signal an inability to move air. A "silent chest" is an ominous sign during an asthma episode. With treatment, increased wheezing actually may signal that the child's condition is improving. Warm, dry skin indicates an improvement in the child's condition because the child is normally diaphoretic during exacerbation. The normal pulse rate in a 10-year-old is 70 to 110 beats/ min. The normal respiratory rate in a 10-year-old is 16 to 20 breaths/ min.

A parent is planning to enroll her 9-month-old infant in a day-care facility. The parent asks the nurse what to look for as indicators that the day-care facility is adhering to good infection-control measures. How should the nurse reply? 1. The facility keeps boxes of gloves in the director's office. 2. Diapers are discarded into covered receptacles. 3. Toys are kept on the floor for the children to share. 4. Disposable papers are used on the diaper-changing surfaces. 5. Facilities for hand washing are located in every classroom. 6. Soiled clothing and cloth diapers are sent home in labeled paper bags.

2. Diapers are discarded into covered receptacles. 4. Disposable papers are used on the diaper-changing surfaces. 5. Facilities for hand washing are located in every classroom. A parent can assess infection-control measures by appraising steps taken by the facility to prevent the spread of potential diseases. Placing diapers in covered receptacles, covering the diaper-changing surfaces with disposable papers, and ensuring that there are available sinks for personnel to wash their hands after activities are all indicators that infection-control measures are being followed. Gloves should be readily available to personnel; therefore, they should be kept in every room — not in an office. Toys typically are shared by numerous children; however, this contributes to the spread of germs and infections. All soiled clothing and cloth diapers should be placed in a sealed plastic bag before the patient is sent home.

A girl, age 13, with anorexia nervosa is admitted to the facility for I.V. fluid therapy and nutritional management. She says she's worried that the I.V. fluids will make her gain weight. Which nursing diagnosis is most appropriate? 1. Noncompliance (dietary regimen) 2. Disturbed body image 3. Dysfunctional grieving 4. Anticipatory grieving

2. Disturbed body image The client with anorexia nervosa has a body image disturbance and views herself as fat despite physical evidence to the contrary. One goal of nursing care is to help her develop realistic perceptions about her body. Although the client has expressed concern about weight gain from I.V. fluids, no information suggests she'll refuse treatment; therefore, a nursing diagnosis of Noncompliance isn't warranted. Likewise, no evidence supports the nursing diagnoses of Dysfunctional grieving and Anticipatory grieving.

When caring for a child, age 12, who's diagnosed with osteomyelitis of the left femur, the nurse should take which action first? 1. Administering I.V. antibiotics as prescribed 2. Drawing blood for cultures as ordered 3. Monitoring hepatic and renal studies 4. Preparing the child for immediate surgery

2. Drawing blood for cultures as ordered Osteomyelitis, an infectious bone disease, typically results from Staphylococcus aureus or Haemophilus influenzae. Before antibiotic therapy begins, blood cultures must be obtained to identify the causative organism and determine its sensitivity to antimicrobial agents. Treatment may include high doses of antibiotics. Hepatic and renal studies are obtained during the course of antibiotic therapy to monitor the child for adverse effects. Later, surgery may be necessary to drain abscesses.

A child with sickle cell anemia is being discharged after treatment for a crisis. Which instructions for avoiding future crises should the nurse provide to the client and his family? 1. Avoid foods high in folic acid. 2. Drink plenty of fluids. 3. Use cold packs to relieve joint pain. 4. Report a sore throat to an adult immediately. 5. Restrict activity to quiet board games. 6. Wash hands before meals and after playing.

2. Drink plenty of fluids. 4. Report a sore throat to an adult immediately. 6. Wash hands before meals and after playing. Fluids should be encouraged to prevent stasis in the bloodstream, which can lead to sickling. Sore throats and other cold symptoms should be promptly reported because they may indicate the presence of an infection, which can precipitate a crisis (red blood cells sickle and obstruct blood flow to tissues). Children with sickle cell anemia should learn appropriate measures to prevent infection, such as proper hand-washing techniques and good nutrition practices. Folic acid intake should be encouraged to help support new cell growth; new cells replace fragile, sickled cells. Warm packs should be applied to provide comfort and relieve pain; cold packs cause vasoconstriction. The child should maintain an active, normal life. When the child experiences a pain crisis, he limits his own activity according to his pain level.

A school-age child begins to have a seizure while walking to the bathroom. What should the nurse do first? 1. Call the physician caring for the child. 2. Ease the child to the floor and turn him on his side. 3. Administer diazepam (Valium) through the I.V. tubing. 4. Notify the parents so they can be with their child.

2. Ease the child to the floor and turn him on his side. Because the child is standing, he should be eased to the floor and turned to the side to prevent aspiration. Notifying the physician wouldn't be the first action the nurse would take because the child's safety is of primary importance. Diazepam would be administered only if it had been ordered. Notifying the parents, although important, isn't the priority. They can be informed after the seizure is over.

The nurse is assessing a 3-year-old child who has ingested toilet bowl cleaner. What finding should the nurse expect? 1. Reddish-colored skin 2. Edematous lips 3. Hypertension 4. Lower abdominal pain

2. Edematous lips The child who has ingested a caustic poison such as lye (found in toilet bowl cleaners) may develop edema, ulcers of the lips and mouth, pain in the mouth and throat, excessive salivation, dysphagia, and burns of the mouth, lips, esophagus, and stomach. Bleeding from burns in the GI tract can lead to pallor, hypotension, tachypnea, and tachycardia. Reddish-colored skin, hypertension, and lower abdominal pain don't commonly occur in caustic poisoning.

The physician suspects tracheoesophageal fistula in a 1-day-old neonate. Which nursing intervention is most appropriate for this child? 1. Avoiding suctioning unless cyanosis occurs 2. Elevating the neonate's head and giving nothing by mouth 3. Elevating the neonate's head for 1 hour after feedings 4. Giving the neonate only glucose water for the first 24 hours

2. Elevating the neonate's head and giving nothing by mouth Because of the risk of aspiration, a neonate with a known or suspected tracheoesophageal fistula should be kept with the head elevated at all times and should receive nothing by mouth (NPO). The nurse should suction the neonate regularly to maintain a patent airway and prevent pooling of secretions. Elevating the neonate's head after feedings or giving glucose water are inappropriate because the neonate must remain on NPO status.

A nurse is caring for a 3-year-old child with viral meningitis. Which signs and symptoms would the nurse expect to find during the initial assessment? 1. Bulging anterior fontanel 2. Fever 3. Nuchal rigidity 4. Petechiae 5. Irritability 6. Photophobia 7. Hypothermia

2. Fever 3. Nuchal rigidity 5. Irritability 6. Photophobia Common signs and symptoms of viral meningitis include fever, nuchal rigidity, irritability, and photophobia. A bulging anterior fontanel is a sign of hydrocephalus, which isn't likely to occur in a toddler because the anterior fontanel typically closes by age 24 months. A petechial, purpuric rash may be seen with bacterial meningitis. Hypothermia is a common sign of bacterial meningitis in an infant younger than age 3 months.

An adolescent female arrives in the emergency department after a physical assault. How could the male nurse best protect the patient's rights during the physical examination? 1. Leave the door open. 2. Have a female health care worker present. 3. Keep the suspected attacker away from the examination room. 4. Keep the client's friends (who are waiting in the lounge area) informed of her medical condition.

2. Have a female health care worker present. A female health care provider should be present to observe the examination when it's performed by a male health care provider. Leaving the door open and informing her friends violates her right to privacy and confidentiality. Although the suspected attacker should be kept away from the examination room, having a female health care worker present during the examination best protects the client's rights.

After surgery to repair a cleft lip, an infant has a Logan bar in place. Which postoperative nursing action is appropriate? 1. Removing the Logan bar during feedings 2. Holding the infant semi-upright during feedings 3. Burping the infant less frequently 4. Placing the infant on the abdomen after feedings

2. Holding the infant semi-upright during feedings Holding the infant semi-upright during feedings helps prevent aspiration. The Logan bar must be kept in place at all times to protect the suture line. The infant should be burped more frequently to prevent regurgitation and aspiration. Placing the infant on the abdomen could lead to disruption of the suture line if the infant rubs the face.

When assessing a child for impetigo, the nurse expects which assessment findings? 1. Small, brown, benign lesions 2. Honey-colored, crusted lesions 3. Linear, threadlike burrows 4. Circular lesions that clear centrally

2. Honey-colored, crusted lesions In impetigo, honey-colored, crusted lesions develop once the pustules rupture. Small, brown, benign lesions are common in children with warts. Linear, threadlike burrows are typical in a child with scabies. Circular lesions that clear centrally characterize tinea corporis.

A 3-year-old child is admitted to the hospital with an acute exacerbation of asthma. The child's history reveals that the child was exposed to chickenpox 1 week ago. When would this child require isolation? 1. Isolation isn't required. 2. Immediate isolation is required. 3. Isolation is required 10 days after exposure. 4. Isolation is required 12 days after exposure.

2. Immediate isolation is required. The incubation period for chickenpox is 2 to 3 weeks. A client is commonly isolated 1 week after exposure to avoid the risk of an outbreak. A person is infectious from 1 day before eruption of lesions to 6 days after the vesicles have formed crusts.

When assessing a family suspected of abusing its 4-year-old child, which of the following is the most important criterion that would suggest abuse? 1. Attempts by the child to defend or verify what the parent states 2. Incompatibility between the history (mechanism) and the injury 3. Responsibility taken by the child for the act 4. A complaint other than the one associated with the signs of abuse

2. Incompatibility between the history (mechanism) and the injury The most important criterion on which to base a decision for reporting suspected abuse is an incompatibility between the history and the injury. A maltreated child will rarely betray his parents by saying he has been abused and will, instead, attempt to defend the parent's action and verify the story. The child may even take responsibility for the act in attempt to vindicate them. A complaint other than the one associated with the signs of abuse (for example, a complaint of being cold when second-degree burns are visible) is a warning sign of abuse.

A child, age 5, is hospitalized for treatment of Kawasaki disease. Which nursing action would best identify potential complications of this disease? 1. Auscultating breath sounds 2. Instituting cardiac monitoring 3. Monitoring blood pressure 4. Assessing the skin daily

2. Instituting cardiac monitoring Kawasaki disease sometimes causes cardiac complications, including arrhythmias. Instituting cardiac monitoring is crucial in detecting such complications. Auscultating breath sounds, monitoring blood pressure, and assessing the skin daily are important but to a lesser degree.

A child is admitted to the pediatric unit with a serum sodium level of 118 mEq/L. Which nursing action takes highest priority at this time? 1. Replacing fluids slowly as ordered 2. Instituting seizure precautions 3. Administering diuretic therapy as prescribed 4. Administering sodium bicarbonate as prescribed

2. Instituting seizure precautions A serum sodium level of 118 mEq/L indicates severe hyponatremia, which places the client at risk for seizures. Therefore, instituting seizure precautions takes highest priority. Fluid and sodium replacement should be done rapidly. Diuretic therapy isn't indicated because it may cause additional sodium loss. In a child with hyperkalemia, administering sodium bicarbonate would be appropriate because it promotes movement of potassium into the intracellular spaces.

For an 8-month-old infant, which toy promotes cognitive development? 1. Finger paint 2. Jack-in-the-box 3. A small rubber ball 4. A play gym strung across the crib

2. Jack-in-the-box According to Piaget's theory of cognitive development, an 8-month-old child will look for an object once it disappears from sight to develop the cognitive skill of object permanence. Finger paint and small balls are potentially dangerous because infants frequently put their fingers or objects in their mouths. Anything strung across a crib, such as a play gym, is a safety hazard — especially to a child who may use it to pull up to a standing position.

When caring for a toddler with epiglottitis, the nurse should first: 1. examine his throat. 2. Keep a tracheotomy tray at the bedside. 3. administer I.V. fluids. 4. administer antibiotics.

2. Keep a tracheotomy tray at the bedside. Acute epiglottitis is an emergency situation that may require tracheotomy or endotracheal intubation. Inflammation of the epiglottis can cause the airway to swell so that it's unable to rise, totally obstructing the airway. Never depress the tongue of a child with a tongue blade to examine the throat if signs or symptoms of epiglottitis are present. This maneuver can cause the swollen epiglottis to completely obstruct the airway. Because the child can't swallow, I.V. fluids are necessary; however, airway concerns are a priority. After a patent airway is secured, antibiotics may be given to treat Haemophilus influenzae, a common cause of acute epiglottitis.

When caring for an adolescent who's at risk for injury related to intracranial pathology, which action would maintain stable intracranial pressure (ICP)? 1. Turn the client's head from side to side frequently. 2. Keep the head in midline position while raising the head of the bed 15 to 30 degrees. 3. Hyperextend the client's head with a blanket roll. 4. Suction frequently to maintain a clear airway.

2. Keep the head in midline position while raising the head of the bed 15 to 30 degrees. Elevating the head of the bed while keeping the client's head in midline position will facilitate venous drainage and avoid jugular compression. Turning the head, hyperextending the neck, and suctioning will increase ICP.

The mother of a preschooler with spina bifida tells the nurse that her daughter sneezes and gets a rash when playing with brightly colored balloons and that recently she had an allergic reaction after eating kiwifruit and bananas. The nurse would suspect that the child may have an allergy to which of the following? 1. Bananas 2. Latex 3. Kiwifruit 4. Color dyes

2. Latex Children with spina bifida often develop an allergy to latex and shouldn't be exposed to it. If a child is sensitive to bananas, kiwifruit, and chestnuts, she's likely to be allergic to latex. Some children are allergic to dyes in foods and other products, but dyes aren't a factor in a latex allergy.

The nurse is teaching parents about accident prevention for a toddler. Which guideline is most appropriate? 1. Always make the toddler wear a seat belt when riding in a car. 2. Make sure all medications are kept in containers with childproof safety caps. 3. Never leave a toddler unattended on a bed. 4. Teach rules of the road for bicycle safety.

2. Make sure all medications are kept in containers with childproof safety caps. All over-the-counter and prescription medications should have childproof safety caps. Poisoning accidents are common in toddlers owing to the toddler's curiosity and his increasing mobility and ability to climb. When riding in a car, a toddler should be strapped into a car seat. Wearing a seat belt is an appropriate guideline for a school-age child. Never leaving a child alone on a bed is an appropriate guideline for parents of infants. Toddlers already have the ability to climb on and off of beds and other furniture by themselves. Note, however, that toddlers should never be left unattended on high surfaces, such as an examining table in a physician's office. Teaching the rules of the road for bicycle safety is an appropriate safety measure for a school-age child. Toddlers shouldn't be allowed in the road unsupervised.

A child is admitted for treatment of bulimia nervosa. When developing the care plan, the nurse anticipates including interventions that address which metabolic disorder? 1. Hypoglycemia 2. Metabolic alkalosis 3. Metabolic acidosis 4. Hyperkalemia

2. Metabolic alkalosis In a client with bulimia nervosa, metabolic alkalosis may occur secondary to hydrogen loss caused by frequent, self-induced vomiting. Typically, the blood glucose level is within normal limits, making hypoglycemia unlikely. In bulimia nervosa, hypokalemia is more common than hyperkalemia and typically results from potassium loss related to frequent vomiting.

A nurse educator is preparing to conduct a session to the nursing staff regarding the theories of growth and development and plans to discuss Kohlberg's theory of moral development. Which of the following should the nurse include in the session? Select all that apply. 1. Individuals move through all six stages in a sequential fashion. 2. Moral development progresses in relationship to cognitive development. 3. A person's ability to make moral judgments develops over a period of time. 4. The theory provides a framework for understanding how individuals determine a moral code to guide their behavior. 5. In stage 1 ( punishment-obedience orientation), children are expected to reason as mature members of society. 6. In stage 2 (instrumental relativist orientation), the child conforms to rules to obtain rewards or have favors returned.

2. Moral development progresses in relationship to cognitive development. 3. A person's ability to make moral judgments develops over a period of time. 4. The theory provides a framework for understanding how individuals determine a moral code to guide their behavior. 6. In stage 2 (instrumental relativist orientation), the child conforms to rules to obtain rewards or have favors returned. Kohlberg's theory states that individuals move through the six stages of development in a sequential fashion but that not everyone reaches stages 5 and 6 in his or her development of personal morality. The theory provides a framework for understanding how individuals determine a moral code to guide their behavior. It states that moral development progresses in relationship to cognitive development and that a person's ability to make moral judgments develops over a period of time. In stage 1, ages 2 to 3 years (punishment-obedience orientation), children cannot reason as mature members of society. In stage 2, ages 4 to 7 years, (instrumental relativist orientation), the child conforms to rules to obtain rewards or have favors returned.

A nurse is caring for an infant with bronchiolitis , and diagnostic tests have confirmed respiratory syncytial virus (RSV). Based on this finding, which of the following is the appropriate nursing action? 1. Initiate strict enteric precautions. 2. Move the infant to a room with another child with RSV. 3. Leave the infant in the present room because RSV is not contagious. 4. Inform the staff that they must wear a mask, gloves, and a gown when caring for the child.

2. Move the infant to a room with another child with RSV. Respiratory syncytial virus (RSV) is a highly communicable disorder and is not transmitted via the airborne route. The virus usually is transferred by the hands. Use of contact and standard precautions during care is necessary. Using good handwashing techniques and wearing gloves and gowns are also necessary. Masks are not required. An infant with RSV is isolated in a single room or placed in a room with another child with RSV. Enteric precautions are unnecessary.

The nurse is caring for a 16-year-old girl who isn't sexually active. The client asks if she needs a Papanicolaou (Pap) test. The nurse should reply: 1. Yes; she should have a Pap test after the onset of menstruation. 2. No; it's not necessary because she isn't sexually active. 3. Yes; she should have a Pap test because she's 16 years old. 4. No; it's not necessary because she isn't 21 years old.

2. No; it's not necessary because she isn't sexually active. A 16-year-old girl who isn't sexually active doesn't need a Pap test. When a client is sexually active or reaches age 18, a Pap test should be performed.

A 2-year-old child with a low blood level of the immunosuppressive drug cyclosporine comes to a liver transplant clinic for her appointment. The mother says the child hasn't been vomiting and hasn't had diarrhea, but she admits that her daughter doesn't like taking the liquid medication. Based on knowledge of the drug, which of the following would the nurse instruct the mother to do? 1. Let your daughter take her medication only when she wants it; it's okay for her to miss some doses. 2. Offer the medication diluted with chocolate milk or orange juice to make it more palatable. 3. Insert a nasogastric (NG) tube and administer the medication using the tube as ordered by the physician. 4. Give the ordered dose a little bit at a time over 2 hours to ensure administration of the medication.

2. Offer the medication diluted with chocolate milk or orange juice to make it more palatable. Liquid cyclosporine has a very unpleasant taste. Diluting it will lessen the strong taste and help the child take the medication as ordered. It isn't acceptable to miss a dose because the drug's effectiveness is based on therapeutic blood levels, and skipping a dose could lower the level. Cyclosporine shouldn't be given by NG tube because it adheres to the plastic tube and, thus, all of the drug may not be administered. Taking the medication over a period of time could negatively affect the blood level.

A neonate was admitted to the pediatric unit with an unexpected congenital defect. What is the best way to involve the parents in the neonate's care? 1. Assume the parents have already been told how to care for their neonate. 2. Offer the parents opportunities to be involved with the neonate's care while they adjust to his unexpected condition. 3. Tell the parents that they'll be shown how to do everything for the neonate once before they take him home. 4. Don't show the parents how to care for the neonate at this time.

2. Offer the parents opportunities to be involved with the neonate's care while they adjust to his unexpected condition. Many new parents need to grieve over the loss of a "normal" child. Adequate time and support should be given for the parents to adjust to the unexpected condition of their child. Never assume that the parents have already been educated about the neonate's care, or that they'll be able to learn everything they need to know after receiving instructions only once. The parents should be involved in the neonate's care during hospitalization; this will help them learn and will instill confidence.

An infant is diagnosed with a congenital hip dislocation. On assessment, the nurse expects to note: 1. symmetrical thigh and gluteal folds. 2. Ortolani's sign. 3. increased hip abduction. 4. femoral lengthening.

2. Ortolani's sign. In a child with a congenital hip dislocation, assessment typically reveals Ortolani's sign, asymmetrical thigh and gluteal folds, limited hip abduction, femoral shortening, and Trendelenburg's sign.

A 4-year-old boy is scheduled for a nephrectomy to remove a Wilms' tumor. Which intervention shouldn't be included in the nursing care plan? 1. Provide preoperative teaching to the child and his parents. 2. Palpate his abdomen to monitor tumor growth. 3. Assess vital signs and report hypertension. 4. Monitor urine for hematuria.

2. Palpate his abdomen to monitor tumor growth. The abdomen of a child with Wilms' tumor should never be palpated because it may increase the risk of metastasis. All children and their parents require preoperative teaching when surgery is planned. The child with Wilms' tumor may be hypertensive as a result of excessive renin production and may have hematuria.

The nurse observes a 2½-year-old child playing with another child of the same age in the playroom on the pediatric unit. What type of play should the nurse expect the children to engage in? 1. Associative play 2. Parallel play 3. Cooperative play 4. Therapeutic play

2. Parallel play Two-year-olds engage in parallel play, in which they play side by side but rarely interact. Associative play is characteristic of preschoolers, in which they are all engaged in a similar activity but there is little organization. School-age children engage in cooperative play, which is organized and goal-directed. Therapeutic play is a technique that can be used to help understand a child's feelings; it consists of energy release, dramatic play, and creative play.

A parent asks the nurse for advice on disciplining a 3-year-old child. During the teaching session, which fact should the nurse emphasize? 1. Children under age 3 or 4 rarely need to be punished. 2. Parental control should be firm and consistent. 3. Withdrawing affection is a highly effective strategy. 4. Parents should enforce rules rigidly.

2. Parental control should be firm and consistent. To deal with misbehavior most successfully, parents should be firm and consistent when taking appropriate disciplinary action. Usually, parents should begin setting limits and implementing discipline around age 1, or when the child begins to crawl and explore the environment. Rigidly enforcing rules wouldn't allow the development of autonomy and could lead to self-doubt. The parent should never be encouraged to withdraw affection as a result of the child's behavior, or any other reason.

The nurse is caring for an infant with congenital clubfoot. After the final cast has been removed, which member of the health care team will most likely help the infant with leg and ankle exercises and provide his parents with a home exercise regimen? 1. Occupational therapist 2. Physical therapist 3. Recreational therapist 4. Nurse

2. Physical therapist After the final cast has been removed, foot and ankle exercises may be necessary to improve range of motion. A physical therapist should work with the child. A physical therapist is trained to help clients restore function and mobility, which will prevent further disability. An occupational therapist, who helps the chronically ill or disabled to perform activities of daily living and adapt to limitations, isn't necessary at this time. A recreational therapist, who uses games and group activities to redirect maladaptive energy into appropriate behavior, also isn't required. The nurse hasn't been trained to design an exercise regimen for a child with congenital clubfoot.

The nurse is teaching accident prevention to the parents of a toddler. Which instruction is appropriate for the nurse to tell the parents? 1. The toddler should wear a helmet when roller blading. 2. Place locks on cabinets containing toxic substances. 3. Teach the toddler water safety. 4. Don't allow the toddler to use pillows when sleeping.

2. Place locks on cabinets containing toxic substances. All household cleaners and poisons should be locked with childproof locks. The toddler's curiosity and the ability to climb and open doors and drawers makes poisoning a concern in this age-group. Roller blading isn't an appropriate activity for toddlers. Toddlers lack the cognitive development to understand water safety. (Note that roller blading and teaching water safety are appropriate for school-age children.) Pillows shouldn't be placed in the crib of an infant to avoid suffocation; however, toddlers may use them.

The nurse is developing a plan to teach a mother how to reduce her baby's risk of developing otitis media. Which direction should the nurse include in the teaching plan? 1. Administer antibiotics whenever the baby has a cold. 2. Place the baby in an upright position when giving a bottle. 3. Avoid getting the ears wet while bathing or swimming. 4. Clean the external ear canal daily.

2. Place the baby in an upright position when giving a bottle. Feeding a baby in an upright position reduces the pooling of formula in the nasopharynx. Formula provides a good medium for the growth of bacteria, which can travel easily through the short, horizontal eustachian tubes. The other interventions don't reduce the risk of a baby developing otitis media.

An 18-month-old child immobilized with traction to the legs has a nursing diagnosis of Deficient diversional activity related to immobility. The nurse should include which diversional activity in the care plan? 1. Playing with tinker toys 2. Playing with a pounding board 3. Playing with a pull toy 4. Playing games

2. Playing with a pounding board Playing with a pounding board is a developmentally appropriate diversional activity for a toddler. Besides promoting physical development, it provides an acceptable energy outlet during immobilization. A child younger than age 3 accidentally may swallow tinker toys and other toys with small parts. A pull toy is appropriate for a toddler but not for one who's immobilized. Playing games is too advanced for a toddler's developmental stage.

The nurse is caring for a 10-year-old child with cystic fibrosis. The child's parents tell the nurse that they're having difficulty coping with their child's disease. Which action wouldn't be appropriate for the nurse to take? 1. Helping the child and family obtain necessary equipment, supplies, and medication 2. Pointing out to the parents ways in which they might have done things differently 3. Providing referrals to local community agencies and the Cystic Fibrosis Foundation 4. Encouraging the parents to allow their child to follow as normal a childhood as possible

2. Pointing out to the parents ways in which they might have done things differently The nurse should avoid being critical when talking with parents about how they have handled their child's disease or condition. The nurse can help this family by assisting them with finding appropriate financial, psychological, and social support. Providing referrals to the local community agencies and the Cystic Fibrosis Foundation is also an appropriate intervention. The child should be treated as much like a normal child as possible.

The mother of a 6-year-old child arrives at a clinic because the child has been experiencing scratchy, red, and swollen eyes. The nurse notes a discharge from the eyes and sends a culture to the laboratory for analysis. Chlamydial conjunctivitis is diagnosed. Based on this diagnosis, the nurse determines that which of the following requires further investigation? 1. Possible trauma 2. Possible sexual abuse 3. Presence of an allergy 4. Presence of a respiratory infection

2. Possible sexual abuse Conjunctivitis is an inflammation of the conjunctiva. A diagnosis of chlamydial conjunctivitis in a child who is not sexually active should signal the health care provider to assess the child for possible sexual abuse. Allergy, infection, and trauma can cause conjunctivitis, but the causative organism is not likely to be chlamydia.

When developing a care plan for a hospitalized child, the nurse knows that children in which age-group are most likely to view illness as a punishment for misdeeds? 1. Infancy 2. Preschool age 3. School age 4. Adolescence

2. Preschool age Preschool-age children are most likely to view illness as a punishment for misdeeds. Separation anxiety, although seen in all age-groups, is most common in older infants. Fear of death is typical of older school-age children and adolescents. Adolescents also fear mutilation.

For a child with hemophilia, what is the most important nursing goal? 1. Enhancing tissue perfusion 2. Preventing bleeding episodes 3. Promoting tissue oxygenation 4. Controlling pain

2. Preventing bleeding episodes A child with hemophilia is prone to bleeding episodes stemming from coagulatory problems. Therefore, the primary nursing goal is to prevent bleeding episodes and possible hemorrhage. A secondary effect of preventing bleeding episodes is maintenance of tissue perfusion and oxygenation. Hemophilia rarely causes pain.

What is the most common assessment finding in a child with ulcerative colitis? 1. Intense abdominal cramps 2. Profuse diarrhea 3. Anal fissures 4. Abdominal distention

2. Profuse diarrhea Ulcerative colitis causes profuse diarrhea. Intense abdominal cramps, anal fissures, and abdominal distention are more common in Crohn's disease.

A nurse is reviewing the laboratory results for a child scheduled for tonsillectomy. The nurse determines that which laboratory value is most significant to review? 1. Creatinine level 2. Prothrombin time 3. Sedimentation rate 4. Blood urea nitrogen level

2. Prothrombin time A tonsillectomy is the surgical removal of the tonsils. Because the tonsillar area is so vascular, postoperative bleeding is a concern. Prothrombin time, partial thromboplastin time, platelet count, hemoglobin and hematocrit, white blood cell count, and urinalysis are performed preoperatively. The prothrombin time results would identify a potential for bleeding. Creatinine level, sedimentation rate, and blood urea nitrogen would not determine the potential for bleeding.

A child, age 14, is hospitalized for nutritional management and drug therapy after experiencing an acute episode of ulcerative colitis. Which nursing intervention would be appropriate? 1. Administering digestive enzymes before meals as prescribed 2. Providing small, frequent meals 3. Administering antibiotics with meals as prescribed 4. Providing high-fiber snacks

2. Providing small, frequent meals Clients with ulcerative colitis, an inflammatory bowel disorder (IBD), tolerate small, frequent meals better than a few large meals daily. Eating large amounts of food may exacerbate the abdominal distention, cramps, and nausea typically caused by IBD. Frequent meals also provide the additional calories needed to restore nutritional balance. This client doesn't lack digestive enzymes and therefore doesn't need enzyme supplementation. Antibiotics are contraindicated because they may interfere with the actions of other prescribed drugs and because ulcerative colitis isn't caused by bacteria. High-fiber foods may irritate the bowel further.

The nurse notes that an infant develops arm movement before fine-motor finger skills and interprets this as an example of which pattern of development? 1. Cephalocaudal 2. Proximodistal 3. Differentiation 4. Mass-to-specific

2. Proximodistal Proximodistal development progresses from the center of the body to the extremities, such as from the arm to the fingers. Cephalocaudal development occurs along the body's long axis; for example, the infant develops control over the head, mouth, and eye movements before the upper body, torso, and legs. Mass-to-specific development, sometimes called differentiation, occurs as the child masters simple operations before complex functions and moves from broad, general patterns of behavior to more refined ones.

The nurse is caring for a 2½-year-old child with tetralogy of Fallot (TOF). Which abnormalities are associated with TOF? 1. Aortic stenosis, atrial septal defect, overriding aorta, and left ventricular hypertrophy 2. Pulmonic stenosis, intraventricular septal defect, overriding aorta, and right ventricular hypertrophy 3. Pulmonic stenosis, patent ductus arteriosus, overriding aorta, and right ventricular hypertrophy 4. Transposition of the great vessels, intraventricular septal defect, right ventricular hypertrophy, and patent ductus arteriosus

2. Pulmonic stenosis, intraventricular septal defect, overriding aorta, and right ventricular hypertrophy TOF consists of four congenital anomalies: pulmonic stenosis, intraventricular septal defect, overriding aorta, and right ventricular hypertrophy. The other combinations of defects aren't characteristic of TOF.

A nurse is conducting an infant nutrition class for parents. Which foods should the nurse tell parents it's okay to introduce during the first year of life? 1. Sliced beef 2. Pureed fruits 3. Whole milk 4. Rice cereal 5. Strained vegetables 6. Fruit juice

2. Pureed fruits 4. Rice cereal 5. Strained vegetables The first food provided to a neonate is breast milk or formula. Between ages 4 and 6 months, rice cereal can be introduced, followed by pureed or strained fruits and vegetables, then strained or ground meat. Meats must be chopped or ground prior to feeding them to an infant to prevent choking. Infants shouldn't be given whole milk until they are at least 1 year old. Fruit drinks provide no nutritional benefit and shouldn't be encouraged.

The nurse is planning care for a 10-year-old child in the acute phase of rheumatic fever. Which activity would be most appropriate for the nurse to schedule in the care plan? 1. Playing ping-pong 2. Reading books 3. Climbing on play equipment in the playroom 4. Unrestricted ambulation

2. Reading books During the acute phase of rheumatic fever, the child should be placed on bed rest to reduce the workload of the heart and prevent heart failure. An appropriate activity for this child would be reading books. The other activities are too strenuous during the acute phase.

To obtain the most accurate measurement of an infant's height (length), the nurse should measure which of the following? 1. Recumbent height with the infant lying on the side 2. Recumbent height with the infant supine 3. Recumbent height with the infant prone 4. Standing height with the infant held upright

2. Recumbent height with the infant supine For the most accurate measurement, the nurse should place the infant in a supine position and then measure recumbent height. Measuring recumbent height with the infant lying on the side would yield an inaccurate result. Measuring recumbent height with the infant prone would yield an inaccurately long result because it includes the length of the foot. Measuring standing height with the infant held upright would yield an inaccurate result until the child no longer needs assistance to stand up straight.

The nurse is teaching the mother of an ill child about childhood immunizations. The nurse should tell the mother that live virus vaccines shouldn't be administered to children with: 1. diabetes. 2. leukemia. 3. asthma. 4. cystic fibrosis.

2. leukemia. Leukemia causes immunosuppression, so inactivated — rather than live — viruses should be administered. Children with the other conditions listed can receive live virus vaccines because they aren't immunosuppressed.

A toddler is hospitalized with multiple injuries. Although the parent states that the child fell down the stairs, the child's history and physical findings suggest abuse as the cause of the injuries. What should the nurse do first? 1. Refer the parent to a support group such as Parents Anonymous. 2. Report the incident to the proper authorities. 3. Prepare the child for foster care placement. 4. Restrict the parent from the child's room.

2. Report the incident to the proper authorities. The nurse is required by law to report all incidents of suspected child abuse. Once the appropriate authorities have been notified, the child can be placed under protective custody. Later, the nurse may need to prepare the child for foster care placement and refer the parent to a support group. After reporting suspected abuse, the nurse should allow the parent to visit and help care for the child; during these visits, the nurse should exhibit and reinforce positive parenting behaviors.

A teenager is brought to the facility by friends after accidentally ingesting gasoline while siphoning it from a car. Based on the nurse's knowledge of petroleum distillates, which system should be the priority assessment? 1. GI system 2. Respiratory system 3. Neurologic system 4. Cardiovascular system

2. Respiratory system The primary concern with petroleum distillate ingestion is its effect on the respiratory system. Aspiration or absorption of petroleum distillates can cause severe chemical pneumonitis and impaired gas exchange. The GI, neurologic, and cardiovascular systems may also be affected if the petroleum contains additives such as pesticides.

A child is admitted with a tentative diagnosis of clinical depression. Which assessment finding is most significant in confirming this diagnosis? 1. Irritability 2. Sadness 3. Weight gain 4. Fatigue

2. Sadness Clinical depression is diagnosed if the child exhibits a depressed mood (sadness) or loss of interest. Irritability isn't diagnostic for depression. Although a depressed child may gain weight and report fatigue, these findings aren't essential to the diagnosis.

The parents of a 5-year-old call the clinic to tell the nurse that they think their child has been abused by her day-care provider. What should the nurse advise them to do? 1. Make an appointment to speak with the day-care provider. 2. Schedule an immediate appointment with their health care provider. 3. Call the child protective services to file a complaint. 4. Talk to their attorney to file charges against the accused.

2. Schedule an immediate appointment with their health care provider. Because more information needs to be obtained from the child and family, an immediate appointment is most appropriate. It's unclear what type of abuse the parents are concerned about. Calling child protective services is appropriate but isn't the first action to take; neither is talking to an attorney or the day-care provider.

A nurse is preparing to care for a child after a tonsillectomy. The nurse documents on the plan of care to place the child in which appropriate position? 1. Supine 2. Side-lying 3. High Fowler's 4. Trendelenburg's

2. Side-lying A tonsillectomy is the surgical removal of the tonsils. The child should be placed in a prone or side-lying position after the surgical procedure to facilitate drainage. Options 1, 3, and 4 would not achieve this goal.

A child is sent to the school nurse because, according to his teacher, he's constantly scratching his head. When the nurse assesses his hair and scalp, she finds evidence of lice. What did she see? 1. Flaking of the scalp with pink, irritated skin exposed 2. Small white spots that adhere to the hair shaft, close to the scalp 3. Scaly, circumscribed patches on the scalp, with mild alopecia in these areas 4. Multiple tiny pustules on the scalp with no abnormal findings on the hair shafts

2. Small white spots that adhere to the hair shaft, close to the scalp The small white spots that adhere to the hair shafts are the eggs, or nits, of lice. These are easy to see and can't be brushed off like dandruff. Flaking of the scalp may indicate dandruff or a dry scalp. Scaly pustules, due to the scratching, may accompany a lice infestation, but nits would also be found on the hair shafts.

The nurse is auscultating heart sounds in a 2-year-old child. She notes a grade 1 heart murmur. Which characteristic best describes a grade 1 heart murmur? 1. Equal in loudness to the heart sounds 2. Softer than the heart sounds 3. Can be heard without a stethoscope 4. Associated with a precordial thrill

2. Softer than the heart sounds A grade 1 heart murmur is commonly difficult to hear and softer than heart sounds. A grade 2 murmur is usually equal in sound to the heart sounds. A grade 4 murmur is associated with a precordial thrill (a palpable manifestation associated with a loud murmur). A grade 6 murmur can be heard without a stethoscope.

When attempting to reduce the risk for impaired skin integrity related to immobility in a toddler, which action should be avoided? 1. Gently massaging the skin with a lubricating substance 2. Spreading a thin layer of lotion over pressure points 3. Changing the toddler's position frequently 4. Cleaning the skin as often as necessary

2. Spreading a thin layer of lotion over pressure points Using a lotion on the pressure points will soften the skin and promote its breakdown. Gently massaging the skin with a lubricating substance will stimulate circulation and help prevent breakdown. Changing the toddler's position frequently will help minimize pressure, prevent edema, and stimulate circulation. Keeping the skin clean will lessen the chances of irritation and breakdown.

The nurse is caring for a young child with tetralogy of Fallot (TOF). The child is upset and crying. The nurse observes that he's dyspneic and cyanotic. Which position would help relieve the child's dyspnea and cyanosis? 1. Sitting in bed with the head of the bed at a 45-degree angle 2. Squatting 3. Lying flat in bed 4. Lying on his right side

2. Squatting Placing the child in a squatting position sequesters a large amount of blood to the legs, reducing venous return. The other positions don't reduce venous return; therefore, they won't relieve the child's dyspnea and cyanosis. A child with TOF may also assume a knee-chest position to reduce venous return to the heart.

After a tonsillectomy, a nurse reviews the physician's postoperative prescriptions. Which of the following physician's prescriptions does the nurse question? 1. Monitor for bleeding. 2. Suction every 2 hours. 3. Give no milk or milk products. 4. Give clear, cool liquids when awake and alert.

2. Suction every 2 hours. A tonsillectomy is the surgical removal of the tonsils. After tonsillectomy, suction equipment should be available, but suctioning is not performed unless there is an airway obstruction because of the risk of trauma to the surgical site. Monitoring for bleeding is an important nursing intervention after any type of surgery. Milk and milk products are avoided initially because they coat the throat, cause the child to clear the throat, and increase the risk of bleeding. Clear, cool liquids are encouraged.

Which of the following is an early sign of heart failure in an infant with a congenital heart defect? 1. Tachypnea 2. Tachycardia 3. Poor weight gain 4. Pulmonary edema

2. Tachycardia The earliest sign of heart failure in infants is tachycardia (sleeping heart rate greater than 160 beats/minute) as a direct result of sympathetic stimulation. Tachypnea (respiratory rate greater than 60 breaths/minute in infants) occurs in response to decreased lung compliance. Poor weight gain is a result of the increased energy demands to the heart and breathing efforts. Pulmonary edema occurs as the left ventricle fails and blood volume and pressure increase in the left atrium, pulmonary veins, and lungs.

A nurse working on the adolescent unit has a strained working relationship with a coworker and finds it difficult to work well with her. What is the best way for her to go about diffusing this situation? 1. Get advice from other nurses assigned to the unit to see what they think might help the situation. 2. Talk with the other nurse and try to work out their differences so they don't affect client care. 3. Complain to the nurse-manager about the coworker's attitude. 4. Avoid the other nurse by working different shifts.

2. Talk with the other nurse and try to work out their differences so they don't affect client care. When personal conflicts arise, it's always best to have the two individuals try to work them out. If the differences are irreconcilable, then other trained professionals may be needed to mediate the situation. Gossiping to other nurses, complaining to the nurse-manager, and avoiding the situation by working different shifts don't help resolve the problem.

The nurse is developing a care plan for a 10-year-old child who has recently been diagnosed with type 1 diabetes mellitus. Which of the following would be inappropriate to include in a teaching plan focusing on proper hygiene? 1. Encourage regular dental care. 2. Teach blood glucose monitoring. 3. Teach care of cuts and scratches. 4. Teach proper foot care.

2. Teach blood glucose monitoring. Teaching blood glucose monitoring and the use of equipment is necessary in diabetic teaching within the care plan that focuses on demonstrating testing blood glucose levels. Encouraging regular dental care is important for the child's general health. Teaching proper care of cuts and scratches will minimize the risk of infection. Teaching proper foot care is necessary and becomes a priority when the child becomes an adult.

A physician needs to obtain written informed consent for a surgical procedure on an adolescent. Which situation allows the physician to obtain written informed consent from the adolescent and not his parents? 1. The adolescent's 18th birthday is next week. 2. The adolescent is estranged from his parents and lives independently. 3. The adolescent gives his verbal consent to the procedure. 4. The physician doesn't need to obtain consent because the procedure is a minor one.

2. The adolescent is estranged from his parents and lives independently. An emancipated minor is a person younger than age 18 who is legally recognized as an adult under certain conditions. These conditions include becoming pregnant, getting married, graduating from high school, and living independently. Otherwise, an adolescent is considered a minor until his 18th birthday. Written consent must be obtained, even if verbal consent is given. Major surgery, minor surgery, diagnostic tests such as biopsies, and treatments such as blood transfusions, are all examples of procedures that require written informed consent.

During a health teaching session, a student, age 14, asks the school nurse the reason for using underarm deodorant. The nurse should base the response on which physiologic change occurring during adolescence? 1. An increase in adipose tissue distribution, which boosts sweat production 2. The apocrine sweat glands reaching secretory capacity 3. The eccrine sweat glands becoming fully functional 4. The sebaceous glands becoming highly active

2. The apocrine sweat glands reaching secretory capacity The apocrine sweat glands grow in conjunction with hair follicles in the underarm areas; during adolescence, they reach their secretory capacity. Although adipose tissue increases during adolescence, this isn't associated with sweat production. The eccrine sweat glands, distributed over the entire body, aren't associated with pubertal physiologic changes. The sebaceous glands contribute to acne, not sweat production.

A day care nurse is observing a 2-year-old child and suspects that the child may have strabismus. Which observation made by the nurse might indicate this condition? 1. The child has difficulty hearing. 2. The child consistently tilts the head to see. 3. The child consistently turns the head to see. 4. The child does not respond when spoken to.

2. The child consistently tilts the head to see. Strabismus is a condition in which the eyes are not aligned because of lack of coordination of the extraocular muscles. The nurse may suspect strabismus in a child when the child complains of frequent headaches, squints, or tilts the head to see. Other manifestations include crossed eyes, closing one eye to see, diplopia, photophobia, loss of binocular vision, or impairment of depth perception. Options 1, 3, and 4 are not indicative of this condition.

An emergency department nurse is caring for a child diagnosed with epiglottitis. Assessing the child, the nurse monitors for which indication that the child may be experiencing airway obstruction? 1. The child exhibits nasal flaring and bradycardia. 2. The child is leaning forward, with the chin thrust out. 3. The child has a low-grade fever and complains of a sore throat. 4. The child is leaning backward, supporting himself or herself with the hands and arms.

2. The child is leaning forward, with the chin thrust out. Epiglottitis is a bacterial form of croup. A primary concern is that it can progress to acute respiratory distress. Clinical manifestations suggestive of airway obstruction include tripod positioning (leaning forward while supported by arms, chin thrust out, mouth open ), nasal flaring, the use of accessory muscles for breathing, and the presence of stridor. Option 4 is an incorrect position. Options 1 and 3 are incorrect because epiglottitis causes a high fever and tachycardia.

One day after an appendectomy, a 9-year-old rates his pain at 4 out of 5 on the pain scale but is playing video games and laughing with his friend. Which of the following would the nurse document on the child's chart? 1. The child is in no apparent distress, and no pain medication is needed at this time. 2. The child rates pain at 4 out of 5. Pain medication administered as prescribed. 3. The child doesn't understand the pain scale. Performed teaching to help child match his pain rating to how he appears to be feeling. 4. The child rates his pain at 4 out of 5; however, he appears to be in no distress. Reassess when he's visibly showing signs of pain.

2. The child rates pain at 4 out of 5. Pain medication administered as prescribed. Pain is what the child says it is, and the nurse must document what the child reports. If a child's behavior appears to differ from the child's rating of pain, believe the pain rating. A child who uses passive coping behaviors (such as distraction and cooperative) may rate pain as more intense than children who use active coping behaviors (such as crying and kicking). Nurses frequently make judgments about pain based on behavior, which can result in children being inadequately medicated for pain.

Where should the nurse instill an ophthalmic ointment in a 6-year-old child? 1. The sclera 2. The lower conjunctival sac 3. The upper conjunctival sac 4. The outer canthus

2. The lower conjunctival sac Ophthalmic ointment is best instilled in the lower conjunctival sac. Eyedrops are instilled in the sclera, upper conjunctival sac, or outer canthus.

The nurse is concerned about another nurse's relationship with the members of a family and their ill preschooler. Which behavior would be most worrisome and should be brought to the attention of the nurse-manager? 1. The nurse keeps communication channels open among herself, the family, physicians, and other health care providers. 2. The nurse attempts to influence the family's decisions by presenting her own thoughts and opinions. 3. The nurse works with the family members to find ways to decrease their dependence on health care providers. 4. The nurse has developed teaching skills to instruct the family members so they can accomplish tasks independently.

2. The nurse attempts to influence the family's decisions by presenting her own thoughts and opinions. When a nurse attempts to influence a family's decision with her own opinions and values, the situation becomes one of overinvolvement on the nurse's part and a nontherapeutic relationship. When a nurse keeps communication channels open, works with family members to decrease their dependence on health care providers, and instructs family members so they can accomplish tasks independently, she has developed an appropriate therapeutic relationship.

The nurse is caring for a toddler who was diagnosed with an inoperable brain tumor. The parents are having difficulty deciding on a course of action for their child. Why is it important to have the nurse involved in an ethical discussion about a planned course of treatment? 1. The nurse is viewed as the authority on ethical issues at the hospital. 2. The nurse can act as a liaison between the client, the client's parents, and the health care team. 3. The nurse can easily make time to discuss issues with the parents. 4. It isn't important to involve the nurse in this type of discussion.

2. The nurse can act as a liaison between the client, the client's parents, and the health care team. Because the nurse has the most direct contact with the client and his parents, she can listen to and communicate their wishes for treatment. She can also aid in interpreting information about the client's condition and course of treatment, helping the parents to make an informed decision. Time shouldn't be a factor when it comes to helping parents make decisions about their child's care. Hospitals commonly employ ethicists who can help with ethical dilemmas.

An overweight girl, age 15, has lost 12 lb (5.4 kg) in 8 weeks by dieting. Now, after reaching a weight plateau, she has become discouraged. She and the nurse decide she should keep a food diary. What is the primary purpose of keeping such a diary? 1. To help the client stay busy and more focused on losing weight 2. To help the nurse and client analyze the amount of food the client is eating and determine when food intake occurs 3. To help the nurse and client determine whether the client has been cheating on her diet 4. To provide a written record for the client's next visit

2. To help the nurse and client analyze the amount of food the client is eating and determine when food intake occurs This client's cognitive level of formal operations will help her identify and evaluate eating behaviors of which she may not be aware. She needs to engage in other activities instead of focusing on her diet. Checking for cheating represents a punishment approach, which is relatively ineffective. The food diary is primarily for the client's benefit, although the nurse can use it, too.

A child's parents state that they childproofed their home for their 2-year-old. During a home visit, the nurse discovers some situations that show the parents don't fully understand the developmental abilities of their toddler. Which situation displays misunderstanding by the parents? 1. Safety latches on kitchen cabinets 2. Toy chest in front of a second-story, locked window 3. Pot handles turned toward the back of the stove 4. Hot water heater temperature set at 120° F (48.9° C) or below

2. Toy chest in front of a second-story, locked window Toddlers are able to climb on low furniture and open windows that may not always be locked, especially in the summer. In such situations, the child could fall out of the window. Keeping child safety latches on kitchen cabinets, turning pot handles toward the back of the stove, and setting the hot water heater at a nonscalding temperature are all safeguards against toddler injury.

The nurse is caring for a 9-year-old client who has a grave prognosis after receiving a closed injury from being struck by a car. Which health team member should approach the family about organ donation? 1. Nurse-manager 2. Transplant coordinator 3. Emergency department nurse 4. Pastoral care staff member

2. Transplant coordinator The transplant coordinator is the best health team member to approach the family about organ donation. The transplant coordinator is typically available to hospitals that routinely perform organ transplants. When the coordinator isn't available, the attending physician or another physician not directly involved in determining brain death should approach the family. Although the emergency department nurse may have admitted the patient, she and the nurse-manager aren't directly involved with the client's care or with the family. Pastoral care staff members provide emotional and religious support and aren't involved with approaching the family about organ donation; they may, however, be present in a supportive capacity if the family wishes.

The grandmother of a preschool-age child calls the clinic to report that the child has had a fever, has been fussy, and now has a rash that started on the neck and has spread to the rest of the child's body. The child was exposed to chickenpox about 3 weeks ago. Which statement would be the most important advice to give the grandmother? 1. Bring the child in immediately so the diagnosis can be confirmed. 2. Treat the child's symptoms and use diphenhydramine (Benadryl) for itching. 3. Be sure the child stays quiet, and limit the amount of television viewing. 4. After the fever is gone, the child can return to day care.

2. Treat the child's symptoms and use diphenhydramine (Benadryl) for itching. The most likely explanation for the child's illness is chickenpox. The nurse should review the treatment for chickenpox, which includes acetaminophen for fever and fussiness, and oatmeal baths and diphenhydramine for itching. Unless the child is severely ill or has complications, the child doesn't need to be seen in the clinic. Limiting a preschooler's television viewing is appropriate but isn't the most important advice. Typically, children will limit their own activities as needed.

A preschool-age child is admitted to the facility with nephrotic syndrome. Nursing assessment reveals a blood pressure of 100/60 mm Hg, lethargy, generalized edema, and dark, frothy urine. After prednisone (Deltasone) therapy is initiated, which nursing action takes highest priority? 1. Monitoring the child for hypertension 2. Turning and repositioning the child frequently 3. Providing a high-sodium diet 4. Discussing the adverse effects of steroids with the parents

2. Turning and repositioning the child frequently The child with nephrotic syndrome is at risk for skin breakdown from generalized edema. Because this syndrome typically impairs independent movement, the nurse must turn and reposition the child frequently to help prevent skin breakdown. Frequent turning also helps prevent respiratory infections, which may arise during the edematous phase of nephrotic syndrome. The syndrome typically causes hypotension, not hypertension, from significant loss of intravascular protein and a subsequent drop in oncotic pressure. Dietary sodium should be restricted because it worsens edema. Although the nurse should discuss the adverse effects of steroids with the parents, this isn't a priority at this time.

A nurse is evaluating the developmental level of a 2-year-old. Which of the following does the nurse expect to observe in this child? 1. Uses a fork to eat 2. Uses a cup to drink 3. Pours own milk into a cup 4. Uses a knife for cutting food

2. Uses a cup to drink By age 2 years, the child can use a cup and spoon correctly but with some spilling . By age 3 to 4, the child begins to use a fork. By the end of the preschool period, the child should be able to pour milk into a cup and begin to use a knife for cutting.

The parents of a 2-year-old child with chronic otitis media are concerned that the disorder has affected their child's hearing. Which behavior suggests that the child has a hearing impairment? 1. Stuttering 2. Using gestures to express desires 3. Babbling continuously 4. Playing alongside rather than interacting with peers

2. Using gestures to express desires Using gestures instead of verbal communication to express desires — especially in a child older than age 15 months — may indicate a hearing or communication impairment. Stuttering is normal in children ages 2 to 4, especially boys. Continuous babbling is a normal phase of speech development in young children; its absence, not presence, would be cause for concern. Parallel play — playing alongside peers without interacting — is typical of toddlers. However, in an older child, difficulty interacting with peers or avoiding social situations may indicate a hearing deficit.

The physician prescribes penicillin G, 300,000 U I.M., for an 18-month-old child. What is the best site for the nurse to administer this injection? 1. Deltoid muscle 2. Vastus lateralis muscle 3. Dorsogluteal muscle 4. Ventrogluteal muscle

2. Vastus lateralis muscle For a child younger than age 3, the thigh (vastus lateralis muscle) is the best site for I.M. injections because it has few major nerves and blood vessels. The deltoid, dorsogluteal, and ventrogluteal sites aren't recommended for a child younger than age 3 because of the lack of muscle development and the risk of nerve injury during injection. Before the dorsogluteal or ventrogluteal sites can be used safely, the child should have been walking for at least 1 year to ensure sufficient muscle development.

When assessing a child with muscular dystrophy, the nurse expects which finding? 1. Pain 2. Waddling gait 3. Joint swelling 4. Limited range of motion (ROM)

2. Waddling gait A waddling, wide-based gait is a sign of muscular dystrophy. Pain, joint swelling, and limited ROM are rare with this disease.

A 2-year-old boy is brought into the clinic with an upper respiratory tract infection. During the assessment, the nurse notes some bruising on the arms, legs, and trunk. Which findings would prompt the nurse to evaluate for suspected child abuse? 1. A few superficial scrapes on the lower legs 2. Welts or bruises in various stages of healing on the trunk 3. A deep blue-black patch on the buttocks 4. One large bruise on the child's thigh 5. Circular, symmetrical burns on the lower legs 6. A parent who is hypercritical of the child and pushes the frightened child away

2. Welts or bruises in various stages of healing on the trunk 5. Circular, symmetrical burns on the lower legs 6. A parent who is hypercritical of the child and pushes the frightened child away Injuries at various stages of healing in protected or padded areas can be signs of inflicted trauma, leading the nurse to suspect abuse. Burns that are bilateral as well as symmetrical and regular are typical of child abuse. The shape of the burn may resemble the item used to create it, such as a cigarette. When a child is burned accidentally, the burns form an erratic pattern and are usually irregular or asymmetrical. Pushing away the child and being hypercritical are typical behaviors of abusive parents. Superficial scrapes and bruises on the lower extremities are normal in a healthy, active child. A deep blue-black macular patch on the buttocks is more consistent with a Mongolian spot rather than a traumatic injury that would suggest abuse.

A child with leukemia has just completed a course of methotrexate therapy. How soon should the nurse expect to see signs of bone marrow depression in this client? 1. Within hours 2. Within 2 weeks 3. Within 1 month 4. After induction therapy is completed

2. Within 2 weeks Bone marrow depression is most likely to occur 10 days after methotrexate is administered.

A child is receiving peritoneal dialysis to treat renal failure. To detect early signs of peritonitis, the nurse should stay alert for: 1. redness at the catheter site. 2. abdominal tenderness. 3. abdominal fullness. 4. headache.

2. abdominal tenderness. Abdominal tenderness is an early sign of peritonitis. Redness at the catheter site indicates a skin infection. Abdominal fullness is expected during dialysate infusion. Headache isn't associated with peritonitis.

When performing a physical assessment on a postterm neonate, the nurse expects to find: 1. absent plantar creases. 2. abundant subcutaneous (S.C.) fat. 3. abundant lanugo. 4. patchy fine hair distribution.

2. abundant subcutaneous (S.C.) fat. Typical assessment findings for a postterm neonate include abundant S.C. fat; long, silky hair; absent vernix caseosa; dry, cracked skin; and long nails. The other options are typical assessment findings for a preterm neonate.

For an infant who's about to undergo a lumbar puncture, the nurse should place the infant in: 1. an arched, side-lying position, with the neck flexed onto the chest. 2. an arched, side-lying position, avoiding flexion of the neck onto the chest. 3. a mummy restraint. 4. a prone position, with the head over the edge of the bed.

2. an arched, side-lying position, avoiding flexion of the neck onto the chest. For a lumbar puncture, the nurse should place the infant in an arched, side-lying position to maximize the space between the third and fifth lumbar vertebrae. The nurse's hands should rest on the back of the infant's shoulders to prevent neck flexion, which could block the airway and cause respiratory arrest. The infant should be placed at the edge of the bed or table during the procedure, and the nurse should speak quietly to calm the infant. A mummy restraint would limit access to the lumbar area because it involves wrapping the child's trunk and extremities snugly in a blanket or towel. A prone position wouldn't cause separation of the vertebral spaces.

An 8-year-old child is receiving conscious sedation for a medical procedure. The nurse is assessing the client's level of sedation. His gag reflex is intact, he's breathing comfortably on his own, and he opens his eyes on verbal request. The nurse recognizes that the client is: 1. undersedated. 2. appropriately sedated. 3. deeply sedated. 4. oversedated.

2. appropriately sedated. Conscious sedation is an induced state of depressed consciousness. While under conscious sedation, the client should maintain protective reflexes (such as the gag reflex), maintain a patent airway independently, and respond to physical stimuli or verbal commands such as, "Open your eyes." In this scenario, the nurse assesses that the client is under conscious sedation. In deep sedation, the client isn't as easily aroused and doesn't have protective reflexes or the ability to maintain a patent airway; this type of sedation is closer to general anesthesia. With oversedation, the patient is difficult to arouse; however, he is able to maintain a patent airway independently.

A 6-year-old child is being discharged from the emergency department after being diagnosed with varicella (chickenpox). What over-the-counter medication should the nurse instruct the parents to avoid administering to the child? 1. ibuprofen (Motrin) 2. aspirin 3. acetaminophen (Tylenol) 4. naproxen (Aleve)

2. aspirin Using aspirin during a viral infection has been linked to Reye's syndrome, a serious illness that can lead to brain damage and death in children. If the child requires medication for fever or discomfort, the nurse should recommend acetaminophen (Tylenol) or ibuprofen (Motrin). Naproxen (Aleve) isn't indicated for the treatment of fever.

When caring for a 2-year-old child, the nurse should offer choices, when appropriate, about some aspects of care. According to Erikson, doing this helps the child achieve : 1. trust. 2. autonomy. 3. industry. 4. initiative.

2. autonomy. According to Erikson's theory of development, a 2-year-old child is at the stage of autonomy versus shame and doubt. An infant is at the stage of trust versus mistrust; a school-age child, industry versus inferiority; and a preschooler, initiative versus guilt.

A child, age 4, is admitted with a tentative diagnosis of congenital heart disease. When assessment reveals a bounding radial pulse coupled with a weak femoral pulse, the nurse suspects that the child has: 1. patent ductus arteriosus. 2. coarctation of the aorta. 3. a ventricular septal defect. 4. truncus arteriosus.

2. coarctation of the aorta. Coarctation of the aorta causes signs of peripheral hypoperfusion, such as a weak femoral pulse and a bounding radial pulse. These signs are rare in patent ductus arteriosus, ventricular septal defect (VSD), and truncus arteriosus.

The nurse is caring for an adolescent client who underwent surgery for a perforated appendix. When caring for this client, the nurse should keep in mind that the main life-stage task for an adolescent is to: 1. resolve conflict with parents. 2. develop an identity and independence. 3. develop trust. 4. plan for the future.

2. develop an identity and independence. The adolescent strives for a sense of independence and identity. During this time, conflicts are heightened, not resolved. Trust begins to develop during infancy and matures along with development. Adolescents rarely finalize plans for the future; this normally happens later in adulthood.

A child with asthma is receiving theophylline. The nurse knows that theophylline is administered primarily to: 1. decrease coughing induced by postnasal drip. 2. dilate the bronchioles. 3. reduce airway inflammation. 4. eradicate the infection.

2. dilate the bronchioles. Methylxanthines such as theophylline are highly potent bronchodilators used to relieve asthma symptoms. Antihistamines typically are used to relieve a cough induced by postnasal drip; corticosteroids, to reduce airway inflammation; and antibiotics, to treat infection.

When planning care for a child with epiglottitis, the nurse should assign highest priority to: 1. providing psychological support. 2. ensuring airway patency. 3. instituting infection control. 4. administering prescribed drug therapy.

2. ensuring airway patency. Because airway obstruction is a life-threatening complication of epiglottitis, ensuring airway patency takes highest priority. The other options are important but don't take precedence over ensuring airway patency.

A 4-year-old child arrives in the emergency department with a history of transient consciousness and unconsciousness. The nurse should suspect: 1. subdural hematoma. 2. epidural hematoma. 3. subarachnoid hemorrhage. 4. concussion.

2. epidural hematoma. An initial loss of consciousness followed by transient consciousness leading to unconsciousness is caused by epidural hematoma. Subdural hematoma results in rapid deterioration in level of consciousness. Subarachnoid hemorrhage causes irritability rather than loss of consciousness. Concussion may result in a brief loss of consciousness.

When developing a care plan for an adolescent, the nurse considers the child's psychosocial needs. During adolescence, psychosocial development focuses on: 1. becoming industrious. 2. establishing an identity. 3. achieving intimacy. 4. developing initiative.

2. establishing an identity. According to Erikson, the primary psychosocial task during adolescence is to establish a personal identity while overcoming role or identity confusion. The adolescent attempts to establish a group identity by seeking acceptance and approval from peers, and strives to attain a personal identity by becoming more independent from the family. Becoming industrious is the developmental task of the school-age child; achieving intimacy is the task of the young adult; and developing initiative is the task of the preschooler.

For a child who's admitted to the emergency department with an acute asthmatic attack, nursing assessment is most likely to reveal: 1. apneic periods. 2. expiratory wheezing. 3. inspiratory stridor. 4. absent breath sounds.

2. expiratory wheezing. Expiratory wheezing is common during an acute asthmatic attack and results from narrowing of the airway caused by edema. Acute asthma rarely causes apneic periods. Inspiratory stridor more commonly accompanies croup. Acute asthma is more likely to cause adventitious breath sounds than absent breath sounds.

The nurse is administering I.V. fluids to an infant. Infants receiving I.V. therapy are particularly vulnerable to: 1. hypotension. 2. fluid overload. 3. cardiac arrhythmias. 4. pulmonary emboli.

2. fluid overload. Infants, small children, and children with compromised cardiopulmonary status receiving I.V. therapy are particularly vulnerable to fluid overload. To prevent fluid overload, the nurse should use a volume-control set and an infusion pump or syringe and place no more than 2 hours' worth of I.V. fluid in the volume-control set at a time. Hypotension, cardiac arrhythmias, and pulmonary emboli aren't problems associated with I.V. therapy in infants.

The nurse is caring for a toddler who has just been immunized. When teaching the child's parents about potential adverse effects, the nurse should instruct the parents to immediately report: 1. pain at the injection site. 2. generalized urticaria. 3. mild temperature elevation. 4. local swelling at the injection site.

2. generalized urticaria. Generalized urticaria can herald the onset of a life-threatening episode, and medical assistance should be sought immediately. A child may experience some pain, redness at the sight, localized swelling, or mild temperature elevation. These reactions can be treated symptomatically and aren't life-threatening.

A 13-year-old boy visits the school nurse because he's experiencing back pain, fatigue, and dyspnea. The nurse suspects that the child may have scoliosis. The nurse should first: 1. send the child home to recover. 2. inspect the child for uneven shoulder height or uneven hip height. 3. arrange for the child to have spinal X-rays as soon as possible. 4. ask the child's mother to take him to a physician immediately.

2. inspect the child for uneven shoulder height or uneven hip height. Before deciding on any specific intervention, the school nurse should perform a basic assessment for scoliosis, including inspecting for uneven shoulder or hip height. The nurse will then have more specific information to give to the parent.

An infant, age 10 months, is brought to the well-baby clinic for a follow-up visit. The mother tells the nurse that she has been having trouble feeding her infant solid foods. To help correct this problem, the nurse should: 1. point out that tongue thrusting is the infant's way of rejecting food. 2. instruct the mother to place the food at the back and toward the side of the infant's mouth. 3. advise the mother to puree foods if the child resists them in solid form. 4. suggest that the mother force-feed the child if necessary.

2. instruct the mother to place the food at the back and toward the side of the infant's mouth. Placing the food at the back and toward the side of the infant's mouth encourages swallowing. Tongue thrusting is a physiologic response to food placed incorrectly in the mouth. Offering pureed foods wouldn't encourage swallowing, which is a learned behavior. Force-feeding may be frustrating for both the mother and child and may cause the child to gag and choke when attempting to reject the undesired food; also, it may lead to a higher-than-normal caloric intake, resulting in obesity.

A 10-month-old child with phenylketonuria (PKU) is being weaned from breast-feeding. When teaching the parents about the proper diet for their child, the nurse should stress the importance of restricting: 1. vegetables and meats. 2. meats and dairy products. 3. grains and meats. 4. sugar and vegetables.

2. meats and dairy products. PKU is an inherited disorder characterized by the inability to metabolize phenylalanine, an essential amino acid. Phenylalanine accumulation in the blood results in central nervous system damage and progressive mental retardation. However, early detection of PKU and dietary restriction of phenylalanine can prevent disease progression. Intake of high-protein foods, such as meats and dairy products, must be restricted because they contain large amounts of phenylalanine. The child may consume measured amounts of vegetables, grains, and sugar, which are low in phenylalanine.

A preschooler has vomiting, diarrhea, and a potassium level of 3 mEq/L. The physician prescribes an I.V. infusion of 500 ml of dextrose 5% in water and half-normal saline solution with 10 mEq of potassium chloride. The nurse knows that a child with vomiting and diarrhea needs fluids and potassium chloride to: 1. eliminate the cause of diarrhea. 2. meet physiologic needs. 3. avoid hyperglycemia. 4. promote normal stool elimination.

2. meet physiologic needs. A child with vomiting and diarrhea loses excessive fluids and electrolytes, which must be replaced. Fluid and electrolyte replacement can't eliminate the cause of diarrhea, which may result from various factors. Administration of I.V. fluids that contain glucose (such as dextrose 5% in water) may induce, not prevent, hyperglycemia. Fluid and electrolyte replacement has no effect on stool elimination.

A child is receiving total parenteral nutrition (TPN). During TPN therapy, the most important nursing action is: 1. assessing vital signs every 30 minutes. 2. monitoring the blood glucose level closely. 3. elevating the head of the bed 60 degrees. 4. providing a daily bath.

2. monitoring the blood glucose level closely. Most TPN solutions contain a high glucose content, placing the client at risk for hyperglycemia. Therefore, the most important nursing action is to monitor the child's blood glucose level closely. A child receiving TPN isn't likely to require vital sign assessment every 30 minutes or elevation of the head of the bed. A daily bath isn't a priority.

The best way for the nurse to assess pain in an 18-month-old child is to: 1. check the child's pupils. 2. observe for behavioral changes. 3. ask the child, "Are you feeling any pain?" 4. tell the parents to call if the child has pain.

2. observe for behavioral changes. Behavioral changes are common signs of pain and are especially valuable indicators in an 18-month-old child, who has limited verbal skills. Evaluating pupillary response isn't an appropriate technique for assessing pain. Requesting a parental report of a child's pain isn't a reliable assessment technique.

An emergency department nurse suspects neglect in a 3-year-old boy admitted for failure to thrive. Signs of neglect in the child would include: 1. slapping, kicking, and punching others. 2. poor hygiene and weight loss. 3. loud crying and screaming. 4. pulling hair and hitting.

2. poor hygiene and weight loss. Neglect can involve failure to provide food, bed, shelter, health care, or hygiene. Slapping, kicking, pulling hair, hitting, and punching are examples of forms of physical abuse. Loud crying and screaming aren't abnormal findings in a 3-year-old boy.

The school nurse is evaluating a 7-year-old client who is having an asthma attack. The client is cyanotic and unable to speak, with decreased breath sounds and shallow respirations. Based on these physical findings, the nurse should first: 1. monitor the client with a pulse oximeter in her office. 2. prepare to ventilate the client. 3. return the client to class. 4. contact the client's parent or guardian.

2. prepare to ventilate the client. The nurse should recognize these physical findings as signs and symptoms of impending respiratory collapse. The nurse's top priority is to assess airway, breathing, and circulation, and prepare to ventilate the client if necessary. The nurse should then notify the emergency medical systems (EMS) to transport the client to a local hospital. When the client's condition allows, the nurse can notify the parents or guardian. This client shouldn't be returned to class. Because the client's condition requires immediate intervention, simply monitoring pulse oximetry would delay treatment.

A child's physician prescribes a drug for home use. Before the child is discharged, the nurse should: 1. teach the family how to adjust the drug schedule according to the child's needs. 2. provide the family with the drug's name, dosage, route, and frequency of administration. 3. instruct the family to encourage the child to take responsibility for ensuring timely drug administration. 4. tell the family to avoid explaining the purpose of the medication to the child.

2. provide the family with the drug's name, dosage, route, and frequency of administration. The nurse should provide the family with essential facts: the drug's name, dosage, route, and frequency of administration. Generally the physician, not the family or nurse, adjusts dosages. It's unrealistic and unsafe to expect a child to take responsibility for ensuring timely administration of any drug. A child has a right to know the reasons for taking the drug.

A child is diagnosed with nephrotic syndrome. When planning the child's care, the nurse understands that the primary goal of treatment is to: 1. manage urinary changes by monitoring fluid intake and output and observing for hematuria. 2. reduce the excretion of urinary protein. 3. help prevent cardiac or renal failure by carefully monitoring fluid and electrolyte balance. 4. decrease edema and hypertension through bed rest and fluid restriction.

2. reduce the excretion of urinary protein. The primary goal of treatment for a child with nephrotic syndrome is to reduce excretion of urinary protein and maintain a protein-free urine. Nephrotic syndrome isn't associated with hematuria, cardiac failure, or hypertension. Fluid restriction isn't warranted.

A child is diagnosed with pituitary dwarfism. Which pituitary agent will the physician probably prescribe to treat this condition? 1. corticotropin zinc hydroxide (Cortrophin-Zinc) 2. somatrem (Protropin) 3. desmopressin acetate (DDAVP) 4. vasopressin (Pitressin)

2. somatrem (Protropin) Somatrem is used to treat linear growth failure stemming from hormonal deficiency. Corticotropin zinc hydroxide is used to treat adrenal insufficiency and a variety of other conditions; DDAVP and vasopressin are used to treat diabetes insipidus.

A mother, who is visibly upset, carries her 2-month-old infant into the crowded emergency department. The child appears limp and lifeless. The mother screams to the nurse for help. The nurse should: 1. take the infant from the mother and offer to help. 2. take the infant and mother back to a treatment room. 3. call the resuscitation team and the supervisor. 4. call security and the hospital administration.

2. take the infant and mother back to a treatment room. Taking the infant and mother into a treatment room for assessment provides privacy and a controlled environment. The mother should be allowed to remain with her child if she wishes. If she doesn't want to be present, the nurse should find a private area for her. The nurse must assess the child before calling the resuscitation team. Security isn't warranted in this situation.

The nurse is caring for a newborn boy who has hypospadias. His parents are planning to have the baby circumcised before discharge. When teaching the parents about their child's condition, the nurse should tell them: 1. the baby can still be circumcised as planned. 2. the foreskin will be needed at the time of surgical correction. 3. circumcision is necessary because the foreskin obstructs the urethral meatus. 4. circumcision will correct the hypospadias.

2. the foreskin will be needed at the time of surgical correction. Circumcision is the surgical removal of the foreskin of the penis. In hypospadias, the urethral meatus is on the underside of the penis. A newborn with hypospadias shouldn't be circumcised because the surgeon may use the foreskin for surgical repair. The foreskin doesn't block the urethral meatus, which may be located near the glans, along the underside of the penis, or at the base. Circumcision doesn't correct hypospadias because the location of the urethral meatus isn't changed during circumcision.

When feeding a neonate with a cleft lip, the nurse should expect to: 1. administer I.V. fluids. 2. use a bulb syringe with a rubber tip. 3. provide thickened formula. 4. perform gastric gavage.

2. use a bulb syringe with a rubber tip. A bulb syringe with a rubber tip is a safe, effective feeding device for a neonate with a cleft lip. I.V. fluids are required only during the immediate postoperative period, until the neonate can tolerate oral fluids. Thickened formula and gastric gavage rarely are necessary for a neonate with a cleft lip.

To calculate drug dosages for a 4-year-old child, the physician might use a formula that involves the child's: 1. weight in pounds and ounces. 2. weight in kilograms. 3. height in inches. 4. height in centimeters.

2. weight in kilograms. To calculate drug dosages for a child, the physician might use a formula that involves the child's weight in kilograms. A second recommended method involves the child's body surface area. Other methods of dosage calculation aren't recommended.

A child, age 5, with an intelligence quotient (IQ) of 65 is admitted to the facility for evaluation. When planning care, the nurse should keep in mind that this child: 1. is within the lower range of normal intelligence. 2. would have a diagnosis of mild mental retardation. 3. would have a diagnosis of moderate mental retardation. 4. would have a diagnosis of severe mental retardation.

2. would have a diagnosis of mild mental retardation. According to the American Association on Mental Deficiency, a person with an IQ between 50 and 70 is classified as mildly mentally retarded. One with an IQ between 36 and 50 is classified as moderately retarded. One with an IQ below 36 is severely impaired.

Erikson - Late childhood

3 - 6 years Initiative vs. guilt Task: Becoming purposeful and directive Successful: Ability to initiate ones own activities; sense of purpose Unsuccessful: Aggression-fear conflict; sense of inadequacy or guilt

A pediatric client with iron deficiency anemia is prescribed ferrous sulfate (Ferralyn), an oral iron supplement. When teaching the child and parent how to administer this preparation, the nurse should provide which instruction? 1. "Administer ferrous sulfate with meals to prevent stomach upset." 2. "Administer ferrous sulfate with milk to promote absorption." 3. "Administer ferrous sulfate with fruit juice to promote absorption." 4. "Administer ferrous sulfate with antacids to prevent stomach upset."

3. "Administer ferrous sulfate with fruit juice to promote absorption." Administering an oral iron supplement such as ferrous sulfate with fruit juice or another vitamin C source enhances its absorption. Preferably, doses should be administered between meals because gastric acidity and absence of food promote iron absorption. In contrast, food, milk, and antacids impair iron absorption.

The mother of a hospitalized 2-year-old child with viral laryngotracheobronchitis (croup ) asks a nurse why the physician did not prescribe antibiotics. The appropriate response is: 1. "The child may be allergic to antibiotics." 2. "The child is too young to receive antibiotics." 3. "Antibiotics are not indicated unless a bacterial infection is present." 4. "The child still has the maternal antibodies from birth and does not need antibiotics."

3. "Antibiotics are not indicated unless a bacterial infection is present." Laryngotracheobronchitis (croup) is the inflammation of the larynx, trachea, and bronchi and is the most common type of croup. It can be viral or bacterial. Antibiotics are not indicated in the treatment of croup unless a bacterial infection is present. Options 1, 2, and 4 are incorrect. In addition , no supporting data in the question indicate that the child may be allergic to antibiotics.

The mother of an 8-year-old child tells the clinic nurse that she is concerned about the child because the child seems to be more attentive to friends than anything else. Using Erikson's psychosocial development theory, the appropriate nursing response is which of the following? 1. "You need to be concerned." 2. "You need to monitor the child's behavior closely." 3. "At this age, the child is developing his own personality." 4. "You need to provide more praise to the child to stop this behavior."

3. "At this age, the child is developing his own personality." According to Erikson, during school-age years (6 to 12 years of age), the child begins to move toward peers and friends and away from the parents for support. The child also begins to develop special interests that reflect his or her own developing personality instead of the parents. Therefore options 1, 2, and 4 are incorrect responses.

Question 51 of 65 An adolescent, age 16, is brought to the clinic for evaluation for a suspected eating disorder. To best assess the effects of role and relationship patterns on the child's nutritional intake, the nurse should ask: 1. "What activities do you engage in during the day?" 2. "Do you have any allergies to foods?" 3. "Do you like yourself physically?" 4. "What kinds of foods do you like to eat?"

3. "Do you like yourself physically?" Role and relationship patterns focus on body image and the client's relationship with others, which commonly interrelate with food intake. Questions about activities and food preferences elicit information about health promotion and health protection behaviors. Questions about food allergies elicit information about health and illness patterns.

A child, age 2, with a history of recurrent ear infections is brought to the clinic with a fever and irritability. To elicit the most pertinent information about the child's ear problems, the nurse should ask the parent: 1. "Does your child's ear hurt?" 2. "Does your child have any hearing problems?" 3. "Does your child tug at either ear?" 4. "Does anyone in your family have hearing problems?"

3. "Does your child tug at either ear?" Although all of the options are appropriate questions to ask when assessing a young child's ear problems, questions about the child's behavior are most useful because a young child usually can't describe symptoms accurately.

A mother brings her child, age 3, to the clinic for an annual checkup. After plotting the child's height and weight on a pediatric growth chart, the nurse identifies which percentile range as normal? 1. 25th to 75th percentile 2. 50th to 100th percentile 3. 5th to 95th percentile 4. 10th to 100th percentile

3. 5th to 95th percentile Height and weight measurements that fall between the 5th and 95th percentiles represent normal growth for most children. Children whose measurements fall outside this range require further evaluation.

A 15-year-old client confides in the nurse that he has been contemplating suicide. He says he has developed a specific plan to carry it out and pleads with the nurse not to tell anyone. What is the nurse's best response? 1. "We can keep this between you and me, but promise me you won't try anything." 2. "I need to protect you. I will tell your physician, but we don't need to involve your parents. We want you to be safe." 3. "For your protection, I can't keep this secret. After I notify the physician, we will need to involve your family. We want you to be safe." 4. "I will need to notify the local authorities of your intentions."

3. "For your protection, I can't keep this secret. After I notify the physician, we will need to involve your family. We want you to be safe." In situations in which the client is a threat to himself, the nurse can't honor confidentiality. Because this client has expressed a specific plan to commit suicide, the nurse must take immediate action to ensure his safety. The physician and mental health professionals should be notified as well as the client's family. The nurse should inform the client that she must do this, while at the same time convey a sense of caring and understanding. The local authorities don't need to be notified in this situation.

The nurse must administer an oral medication to a 3-year-old child. The best way for the nurse to proceed is by saying: 1. "It's time for you to take your medicine right now." 2. "If you take your medicine now, you'll go home sooner." 3. "Here is your medicine. Would you like apple juice or grape drink after?" 4. "See how Jimmy took his medicine? He's a good boy. Now it's your turn."

3. "Here is your medicine. Would you like apple juice or grape drink after?" Involving the child promotes cooperation, and permitting the child to make choices provides a sense of control. Telling a child to take the medicine "right now" could provoke a negative response. Promising that the child will go home sooner could decrease the child's trust in nurses and physicians. Comparing one child with another wouldn't encourage cooperation.

To treat a child's atopic dermatitis, a physician prescribes a topical application of hydrocortisone cream twice daily. After medication instruction by the nurse, which statement by the parent indicates effective teaching? 1. "I will spread a thick coat of hydrocortisone cream on the affected area and will wash this area once a week." 2. "I will gently scrape the skin before applying the cream to promote absorption." 3. "I will avoid using soap and water on the affected area and will apply an emollient cream on this area frequently." 4. "I will apply a moisturizing cream sparingly and will wash the affected area frequently."

3. "I will avoid using soap and water on the affected area and will apply an emollient cream on this area frequently." The parent should avoid washing the affected area with soap and water because this removes moisture from the horny layer of the skin. Applied in a thin layer, emollient cream holds moisture in the skin, provides a barrier to environmental irritants, and helps prevent infection. Topical steroid creams such as hydrocortisone should be applied sparingly as a light film; the affected area should be cleaned gently with water before the cream is applied. Scraping or abrading the skin may increase the risk of infection and alter drug absorption. Excessive application of steroidal creams may result in systemic absorption and Cushing's syndrome. Frequent washing dries the skin, making it more susceptible to cracking and further breakdown.

A bottle-fed infant, age 3 months, is brought to the pediatrician's office for a well-child visit. During the previous visit, the nurse taught the mother about infant nutritional needs. Which statement by the mother during the current visit indicates effective teaching? 1. "I started the baby on cereals and fruits because he wasn't sleeping through the night." 2. "I started putting cereal in the bottle with formula because the baby kept spitting it out." 3. "I'm giving the baby iron-fortified formula and a fluoride supplement because our water isn't fluoridated." 4. "I'm giving the baby skim milk because he was getting so chubby."

3. "I'm giving the baby iron-fortified formula and a fluoride supplement because our water isn't fluoridated." Iron-fortified formula supplies all the nutrients an infant needs during the first 6 months; however, fluoride supplementation is necessary if the local water supply isn't fluoridated. Before age 6 months, solid foods such as cereals aren't recommended because the GI tract tolerates them poorly. Also, a strong extrusion reflex causes the infant to push food out of the mouth. Mixing solid foods in a bottle with liquids deprives the infant of experiencing new tastes and textures and may interfere with development of proper chewing. Skim milk doesn't provide sufficient fat for an infant's growth.

Which statement by the mother of a toddler with nephrotic syndrome indicates that the nurse's discharge teaching was effective? 1. "I know that I'll need to keep my child as quiet as possible." 2. "I just went out and bought all I'll need for the special diet." 3. "I've been checking the urine for protein so I'll be able to do it at home." 4. "I'm sure that my child will be back to normal soon."

3. "I've been checking the urine for protein so I'll be able to do it at home." Protein in urine is an indication of the progression of nephrotic syndrome, so parents are taught to test the child's urine. The child doesn't need to be kept quiet and usually isn't on a specific diet. How the child feels will dictate the child's activity level. Most children return to normal soon but may relapse.

For the last 6 days, a 7-month-old infant has been receiving amoxicillin trihydrate (Amoxil) to treat an ear infection. Now the parents report redness in the diaper area and small, red patches on the infant's inner thighs and buttocks. After diagnosing Candida albicans, the physician prescribes topical nystatin (Mycostatin) to be applied to the perineum four times daily. Medication teaching should include which instruction? 1. "Discontinue amoxicillin until the diaper area is no longer red." 2. "Avoid using superabsorbent disposable diapers." 3. "Inspect your infant's mouth for white patches." 4. "Switch your infant to a soy-based formula."

3. "Inspect your infant's mouth for white patches." The nurse should instruct the parents to inspect the infant's mouth for white patches. Signs of thrush, these patches are common in children with C. albicans infections and should be reported to the physician. The parents shouldn't discontinue amoxicillin; to treat an ear infection effectively, this drug must be administered for at least 10 days. The parents should use, not avoid, superabsorbent diapers because they help keep the diaper area dry — especially if this area is red and irritated. Changing the infant's formula isn't necessary because the diaper irritation probably results from amoxicillin therapy.

A nurse prepares a teaching plan for a mother of a child diagnosed with bacterial conjunctivitis. Which of the following, if stated by the mother, indicates a need for further teaching? 1. "I need to wash my hands frequently." 2. "I need to clean the eye as prescribed." 3. "It is okay to share towels and washcloths." 4. "I need to give the eye drops as prescribed."

3. "It is okay to share towels and washcloths." Conjunctivitis is an inflammation of the conjunctiva . Bacterial conjunctivitis is highly contagious, and the nurse should teach infection control measures. These include good handwashing and not sharing towels and washcloths. Options 1, 2, and 4 are correct treatment measures.

A child, age 15 months, is admitted to the health care facility. During the initial nursing assessment, which statement by the mother most strongly suggests that the child has a Wilms' tumor? 1. "My child has grown 3" in the past 6 months." 2. "My child seems to be napping for longer periods." 3. "My child's abdomen seems bigger, and his diapers are much tighter." 4. "My child's appetite has increased so much lately."

3. "My child's abdomen seems bigger, and his diapers are much tighter." The most common presenting sign of a Wilms' tumor is abdominal swelling or an abdominal mass. A rapid increase in length (height) isn't associated with this type of tumor. Although lethargy may accompany a Wilms' tumor, abdominal swelling is a more specific sign. Children with a Wilms' tumor usually have a decreased appetite.

After the nurse explains dietary restrictions to the parents of a child with celiac disease, which statement by the parents indicates effective teaching? 1. "We'll follow these instructions until our child's symptoms disappear." 2. "Our child must maintain these dietary restrictions until adulthood." 3. "Our child must maintain these dietary restrictions for life." 4. "We'll follow these instructions until our child has completely grown and developed."

3. "Our child must maintain these dietary restrictions for life." A client with celiac disease must maintain dietary restrictions for life to avoid recurrence of clinical manifestations of the disease. The other options are incorrect because signs and symptoms will reappear if the client eats prohibited foods.

A client who has tested positive for the human immunodeficiency virus (HIV) delivers a girl. When she asks whether her baby has acquired immunodeficiency syndrome (AIDS), how should the nurse respond? 1. "Don't worry. It's too soon to tell." 2. "Chances are she'll be okay because you don't have AIDS yet." 3. "She may have acquired HIV in utero, but we won't know for sure until she's older." 4. "All babies born to HIV-positive women are infected with HIV, but your baby won't have symptoms for years."

3. "She may have acquired HIV in utero, but we won't know for sure until she's older." Diagnosing AIDS in neonates is difficult because all neonates of women with HIV receive maternal antibodies and therefore initially test positive for HIV antibodies. However, not all such neonates actually are infected. The neonate of an HIV-positive mother has a 25% to 30% chance of developing HIV.

The mother of a 12-month-old child expresses concern about the effects of her child's frequent thumb-sucking. After the nurse provides instruction on this topic, which response by the mother indicates that teaching has been effective? 1. "Thumb-sucking should be discouraged at age 12 months." 2. "I'll give my baby a pacifier instead." 3. "Sucking is important to the baby." 4. "I'll wrap the baby's thumb in a bandage."

3. "Sucking is important to the baby." Sucking is the infant's chief pleasure. However, thumb-sucking may cause malocclusion if it persists after age 4. Many fetuses begin sucking on their fingers in utero and, as infants, refuse a pacifier as a substitute. A young child is likely to chew on a bandage, possibly leading to airway obstruction.

A 15-month-old child is being discharged after treatment for severe otitis media and bacterial meningitis. Which statement by the parents indicates effective discharge teaching? 1. "We should have gone to the physician sooner. Next time, we will." 2. "We'll take our child to the physician's office every week until everything is okay." 3. "We'll go to the physician if our child pulls on the ears or won't lie down." 4. "We're just so glad this is all behind us."

3. "We'll go to the physician if our child pulls on the ears or won't lie down." The parents indicate full understanding of discharge teaching by repeating the specific, common signs of otitis media in toddlers, such as pulling on the ears and refusing to lie down, and by verbalizing the need for immediate follow-up care if these signs arise. Option 1 implies a sense of guilt. Option 2 addresses only weekly follow-up care. Option 4 is unrealistic because the child's condition may recur.

A mother and grandmother bring a 3-month-old infant to the well-baby clinic for a routine checkup. As the nurse weighs the infant, the grandmother asks, "Shouldn't the baby start eating solid food? My kids started on cereal when they were 2 weeks old." Which response by the nurse would be appropriate? 1. "The baby is gaining weight and doing well. There is no need for solid food yet." 2. "Things have changed a lot since your children were born." 3. "We've found that babies can't digest solid food properly until they're 3 or 4 months old." 4. "We've learned that introducing solid food early leads to eating disorders later in life."

3. "We've found that babies can't digest solid food properly until they're 3 or 4 months old." Infants younger than 3 or 4 months lack the enzymes needed to digest complex carbohydrates. Option 1 doesn't address the grandmother's question directly. Option 2 is a cliché that may block further communication with the grandmother. Option 4 is incorrect because no evidence suggests that introducing solid food early causes eating disorders.

The nurse suspects that a child, age 4, is being neglected physically. To best assess the child's nutritional status, the nurse should ask the parents which question? 1. "Has your child always been so thin?" 2. "Is your child a picky eater?" 3. "What did your child eat for breakfast?" 4. "Do you think your child eats enough?"

3. "What did your child eat for breakfast?" The nurse should obtain objective information about the child's nutritional intake such as by asking about what the child ate for a specific meal. The other options ask for subjective replies that would be open to interpretation.

When assessing an 18-month-old child, the nurse determines that the child's height and weight fall below the 5th percentile on the growth chart. In all previous visits, the child's height and weight fell between the 30th and 40th percentiles. The child's mother expresses concern about the slowed growth rate. How should the nurse respond? 1. "What do you feed your child?" 2. "Don't worry. Your child is bound to have a growth spurt soon." 3. "Your child's height and weight must be checked again in 1 month." 4. "How much weight did you gain when you were pregnant with this child?"

3. "Your child's height and weight must be checked again in 1 month." Although the growth rate usually slows between ages 1 and 3, it normally doesn't drop as dramatically as this child's. Therefore, the nurse should advise the mother to have the child's growth rate monitored frequently, such as every month. Option 1 implies that the mother is at fault for the child's slow growth. Option 2 doesn't address the mother's concern about the child. Option 4 is inappropriate because maternal weight gain during pregnancy wouldn't affect a child's growth rate at 18 months.

The physician prescribes meperidine (Demerol), 30 mg I.M., as preoperative medication for a school-age child who weighs 66 lb (30 kg). The meperidine is supplied as 50 mg/ml. How much meperidine should the nurse administer? 1. 0.3 ml 2. 0.5 ml 3. 0.6 ml 4. 0.8 ml

3. 0.6 ml By using the fraction method and cross-multiplying to solve for X, the nurse can determine that 0.6 ml should be administered: X ml/30 mg = 1 ml/50 mg X ml × 50 mg = 30 mg × 1 ml X = 0.6 ml.

The nurse is preparing to administer short-acting insulin to a child with type 1 diabetes mellitus. When should the nurse measure the child's blood glucose level? 1. Immediately before administering insulin 2. 15 minutes after administering insulin 3. 1 hour after administering insulin 4. 4 hours after administering insulin

3. 1 hour after administering insulin Short-acting insulins peak in 30 minutes to 2 hours after administration. The nurse should check the child's blood glucose level during this period, such as 1 hour after administration.

An 8-month-old infant is admitted with a febrile seizure. The infant weighs 17 lb (7.7 kg). The physician orders ceftriaxone (Rocephin), 270 mg I.M. every 12 hours. (The safe dosage range is 50 to 75 mg/kg daily.) The pharmacy sends a vial containing 500 mg, to which the nurse adds 2 ml of preservative-free normal saline solution. The nurse should administer how many milliliters? 1. None because this isn't a safe dose 2. 0.08 ml 3. 1.08 ml 4. 1.8 ml

3. 1.08 ml Because the infant weighs 17 lb (7.7 kg), the safe dosage range is 385 to 578 mg daily. The ordered dose, 540 mg daily, is safe. To calculate the amount to administer, the nurse may use the following fraction method: 500 mg/2 ml = 270 mg/X ml 500X = 270 × 2 500X = 540 X = 540/500 X = 1.08 ml

The nurse should begin screening for lead poisoning when a child reaches which age? 1. 6 months 2. 12 months 3. 18 months 4. 24 months

3. 18 months The nurse should start screening a child for lead poisoning at age 18 months and perform repeat screenings at ages 24, 30, and 36 months. High-risk infants, such as premature infants and formula-fed infants not receiving iron supplementation, should be screened for iron deficiency anemia at age 6 months. Regular dental visits should begin at age 24 months.

Which of the following is the recommended immunization schedule for diphtheria, tetanus, and pertussis (DTP)? 1. Birth, 2 months, 6 months, 15 to 18 months, and 10 to 12 years 2. 1 month, 2 months, 6 months, 15 to 18 months, and 4 to 6 years 3. 2 months, 4 months, 6 months, 15 to 18 months, and 4 to 6 years 4. Birth, 3 months, 6 months, 12 months, and 4 to 6 years

3. 2 months, 4 months, 6 months, 15 to 18 months, and 4 to 6 years According to the American Academy of Pediatrics and the Committee on Infectious Diseases, the DTP vaccine should be administered at 2 months, 4 months, 6 months, 15 to 18 months, and 4 to 6 years (before the start of school). The other options are incorrect.

The physician prescribes meperidine (Demerol), 1.1 mg/kg I.M., for a 16-month-old child who has just had abdominal surgery. When administering this drug, the nurse should use a needle of which size? 1. 18G 2. 20G 3. 23G 4. 27G

3. 23G For an infant, the nurse should use a needle with the smallest appropriate gauge for the medication to be given. For an I.M. injection of meperidine, a 25G to 22G needle is appropriate.

The nurse is teaching the mother of a newborn. The nurse should instruct the mother to introduce her infant to solid foods at what age? 1. 2 months 2. 4 months 3. 6 months 4. 8 months

3. 6 months Solid foods are typically introduced around age 6 months. They aren't recommended at an earlier age because of the protrusion and sucking reflexes and the immaturity of the infant's GI tract and immune system. By age 8 months, the infant usually has been introduced to iron-fortified infant cereal and vegetables and will begin to try fruits.

The nurse expects an infant to sit up without support at which age? 1. 4 months 2. 6 months 3. 8 months 4. 10 months

3. 8 months Most infants can sit up without support by age 8 months. At age 4 months, the infant can lift the head off the mattress up to a 90-degree angle. Between ages 6 and 7 months, the infant can sit while leaning forward on the hands. At age 10 months, the infant typically can move from a prone to a sitting position and pull himself up to a standing position.

Before administering a tube feeding to a toddler, which of the following methods should the nurse use to check the placement of a nasogastric (NG) tube? 1. Abdominal X-rays 2. Injection of a small amount of air while listening with a stethoscope over the abdominal area 3. A check of the pH of fluid aspirated from the tube 4. Visualization of the measurement mark on the tube made at the time of insertion

3. A check of the pH of fluid aspirated from the tube Intestinal, gastric, and respiratory fluids have different pH values. Therefore, checking the pH of fluid aspirated from the tube is the most reliable technique for checking proper NG tube placement without taking X-rays before each feeding. X-rays can't be performed multiple times a day on a daily basis. Because auscultation of air can be heard when the tube is in the esophagus as well as in the stomach, this isn't the best test for checking placement. Observing the insertion measurement mark isn't a good check either because the mark may remain the same even though the tube has migrated up or down into the esophagus, lungs, or intestines.

Which of the following is not a good indicator of an impending I.V. infiltration in an infant? 1. Erythema and pain 2. Edema 3. A lack of blood return 4. Blanching or streaking along the vein

3. A lack of blood return A blood return or a lack thereof isn't always an indicator of infiltration, especially in infants and small children, whose veins are small and fragile. Erythema, pain, edema at the site or around it, blanching, and streaking are signs of infiltration. The infusion should be discontinued immediately if any of these signs are observed.

An 18-month-old boy is admitted to the pediatric unit with a diagnosis of celiac disease. What finding would the nurse expect in this child? 1. A concave abdomen 2. Bulges in the groin area 3. A protuberant abdomen 4. A palpable abdominal mass

3. A protuberant abdomen A child with celiac disease has a protuberant abdomen due to the presence of fat, bulky stools, undigested food, and flatus. A concave abdomen, bulges in the groin area, and a palpable abdominal mass aren't associated with celiac disease.

When discharging a 5-month-old infant from the hospital, the nurse checks to see whether the parent's car restraint system for the infant is appropriate. Which of the following restraint systems would be safest? 1. A front-facing convertible car seat in the middle of the back seat 2. A rear-facing infant safety seat in the front passenger seat 3. A rear-facing infant safety seat in the middle of the back seat 4. A front-facing convertible car seat in the back seat next to the window

3. A rear-facing infant safety seat in the middle of the back seat Infants from birth to 20 lb (9.1 kg) and under age 1 must be in a rear-facing infant or convertible seat in the back seat, preferably in the middle. Infants and small children should never be placed in the front seat because of the risk of injuries from a breaking front windshield and an expanding airbag. The position next to the window isn't preferred.

A child with suspected rheumatic fever is admitted to the pediatric unit. When obtaining the child's history, the nurse considers which information to be most important? 1. A fever that started 3 days ago 2. Lack of interest in food 3. A recent episode of pharyngitis 4. Vomiting for 2 days

3. A recent episode of pharyngitis A recent episode of pharyngitis is the most important factor in establishing the diagnosis of rheumatic fever. Although the child may have a history of fever or vomiting or lack interest in food, these findings aren't specific to rheumatic fever.

When administering gentamicin to a preschooler, which of the following monitoring schedules is best for determining the drug's effectiveness? 1. A serum trough level every morning 2. A serum peak level after the second dose 3. A serum trough and peak level around the third dose 4. Serial serum trough levels after three doses (24 hours)

3. A serum trough and peak level around the third dose Aminoglycosides such as gentamicin have a narrow range between therapeutic and toxic serum levels. A serum peak and trough level (taken half an hour before the dose and half an hour after the dose has been administered) around the third dose (the third dose provides enough medication buildup in the blood stream to be measured) is the most accurate way to determine the correct serum values. A trough level every morning, a serum peak level after the second dose, and serial serum trough levels won't provide sufficient data about the effectiveness of the antibiotic.

Which immunization would be inappropriate for an adolescent as a component of preventative care? 1. A tetanus-diphtheria (Td) vaccine, given 10 years after the most recent childhood diphtheria, tetanus toxoids, and acellular pertussis (DTaP) vaccine 2. A second measles-mumps-rubella (MMR) vaccine 3. A tuberculin skin test every other year 4. The hepatitis B vaccine, if not received earlier

3. A tuberculin skin test every other year A tuberculin skin test is necessary for adolescents who have been exposed to active tuberculosis, have lived in a homeless shelter, have been incarcerated, have lived in or come from an area with a high prevalence of tuberculosis, or are currently working in a health care setting. The frequency of tuberculin testing depends on risk factors of the individual adolescents. It isn't routinely administered every other year. A Td vaccine given 10 years after the most recent childhood DTaP vaccination, a second MMR vaccine, and a hepatitis B vaccine, if not given earlier, are all recommended immunizations for adolescents.

A child, age 5, is diagnosed with mycoplasmal pneumonia and has a persistent productive cough. When assessing the child's respirations, the nurse should keep in mind that children normally use which muscles to breathe? 1. Accessory 2. Thoracic 3. Abdominal 4. Intercostal

3. Abdominal Normally, children and men use the abdominal muscles to breathe, whereas women use the thoracic muscles. Use of the accessory or intercostal muscles would indicate a respiratory problem.

A nurse prepares to administer digoxin (Lanoxin) to a 3-year-old child with a diagnosis of congestive heart failure and notes that the apical heart rate is 110 beats/ min. Based on this finding which nursing action is appropriate? 1. Hold the medication. 2. Notify the physician. 3. Administer the digoxin. 4. Recheck the apical rate in 15 minutes.

3. Administer the digoxin. The normal apical heart rate for a 3-year-old is 80 to 120 beats/ min. Because the apical rate is within the normal range, options 1, 2, and 4 are inappropriate and unnecessary.

An adolescent girl who's receiving chemotherapy for leukemia is admitted for pneumonia. The adolescent's platelet count is 50,000 μl. Which of the following would be inappropriate to include in the care plan? 1. A sign over the bed that reads "NO NEEDLE STICKS AND NOTHING PER RECTUM" 2. Two peripheral I.V. intermittent infusion devices, one for blood draws and one for infusions 3. Administration of oxygen at a rate of 4 L/minute using a nonhumidified nasal cannula 4. Use of a tympanic membrane sensor to measure her temperature at the bedside

3. Administration of oxygen at a rate of 4 L/minute using a nonhumidified nasal cannula Oxygen should be humidified to assure that irritation of the mucosa doesn't occur. This teen's platelet level is decreased, so she's at risk for bleeding. The nose is a vascular region that can bleed easily if the mucosa is dried by the oxygen. A sign to remind others to avoid needle sticks and to not give anything via the rectum, the presence of two peripheral I.V.s, and the use of a tympanic temperature device are all aspects of care that would decrease the client's risk of bleeding.

A 16-year-old is admitted to the hospital for acute appendicitis and an appendectomy is performed. Which nursing intervention is appropriate to facilitate normal growth and development postoperatively? 1. Encourage the child to rest and read. 2. Encourage the parents to room in with the child. 3. Allow the child to interact with others in his or her same age group. 4. Allow the family to bring in the child's favorite computer games.

3. Allow the child to interact with others in his or her same age group. Adolescents often are not sure whether they want their parents with them when they are hospitalized. Because of the importance of their peer group, separation from friends is a source of anxiety. Ideally, the members of the peer group will support their ill friend. Options 1, 2, and 4 isolate the child from the peer group.

When the physician prescribes an antibiotic to treat a child's ear infection, the nurse checks to see whether the ordered antibiotic could cause ototoxicity. Which options belong in this category? 1. Cephalosporins 2. Penicillins 3. Aminoglycosides 4. Carbapenems

3. Aminoglycosides Aminoglycosides are ototoxic and may cause tinnitus, vertigo, and hearing loss. Cephalosporins, penicillins, and carbapenems aren't associated with ototoxicity.

A chronically ill school-age child is most vulnerable to which stressor? 1. Mutilation anxiety 2. Anticipatory grief 3. Anxiety over school absences 4. Fear of hospital procedures

3. Anxiety over school absences The school-age child is becoming industrious and attempts to master school-related activities. Therefore, school absences are likely to cause extreme anxiety for a school-age child who's chronically ill. Mutilation anxiety is more common in adolescents. Anticipatory grief is rare in a school-age child. Fear of hospital procedures is most pronounced in preschool-age children.

When developing a care plan for a toddler with a seizure disorder, which of the following would be inappropriate? 1. Padded side rails 2. Oxygen mask and bag system at bedside 3. Arm restraints while asleep 4. Cardiorespiratory monitoring

3. Arm restraints while asleep Restraints should never be used on a child with a seizure disorder because they could harm him if a seizure occurs. Padded side rails will prevent the child from injuring himself during a seizure. The bag and mask system should be present in case the child needs oxygen during a seizure. Cardiopulmonary monitoring should be readily available for checking vital signs during a seizure.

To establish a good interview relationship with an adolescent, which strategy is most appropriate? 1. Asking personal questions unrelated to the situation 2. Writing down everything the teen says 3. Asking open-ended questions 4. Discussing the nurse's own thoughts and feelings about the situation

3. Asking open-ended questions Open-ended questions allow the teen to share information and feelings. Asking personal questions not related to the situation jeopardizes the trust that must be established because the adolescent may feel as though he's being probed with unnecessary questions. Writing everything down during the interview can be a distraction and won't allow the nurse to observe how the adolescent behaves. Discussing the nurse's thoughts and feelings may bias the assessment and is inappropriate when interviewing any client.

Which is the priority intervention for a preschool child with epiglottitis and a deteriorating respiratory status? 1. Administering oxygen by face mask 2. Administering parenteral antibiotics 3. Assisting with intubation 4. Monitoring the electrocardiogram for arrhythmias

3. Assisting with intubation The most important intervention for a child with epiglottitis is airway management. Children are at high risk for developing abrupt airway obstruction. Intubation should be performed as soon as possible in a controlled environment. Children need supplemental oxygen, but most are so anxious that they will never allow a mask to stay in place. Provide humidified "blow-by" oxygen administered by the parent if possible. The child needs parenteral antibiotics; however, the priority

The nurse is giving instructions to parents of a school-age child diagnosed with sickle cell disease. The instructions should include which of the following? 1. Apply cold to affected areas to reduce the child's discomfort. 2. Restrict the child's fluids during crisis situations. 3. Avoid areas of low oxygen concentration such as high altitudes. 4. Encourage the child to exercise to reduce the likelihood of crisis.

3. Avoid areas of low oxygen concentration such as high altitudes. The child should avoid areas of low oxygen, such as high altitudes, because they may precipitate sickle cell crisis. Applying warm compresses will reduce discomfort to the affected area; cold compresses, however, may add to discomfort by increasing sickling and impairing circulation. The child should be encouraged to drink fluids to rehydrate cells. Strenuous exercise may induce sickle cell crisis.

For a child with a Wilms' tumor, which preoperative nursing intervention takes highest priority? 1. Restricting oral intake 2. Monitoring acid-base balance 3. Avoiding abdominal palpation 4. Maintaining strict isolation

3. Avoiding abdominal palpation Because manipulating the abdominal mass may disseminate cancer cells to adjacent and distant sites, the most important intervention for a child with a Wilms' tumor is to avoid palpating the abdomen. Restricting oral intake and monitoring acid-base balance are routine interventions for all preoperative clients; they have no higher priority in one with a Wilms' tumor. Isolation isn't required because a Wilms' tumor isn't infectious.

The nurse is caring for an 8-year-old child with acute asthma exacerbation. Which of the following would be of greatest concern to the nurse? 1. The child's respiratory rate is now 24 breaths/minute. 2. Recent blood gas analysis indicates an oxygen saturation of 95%. 3. Before a respiratory therapy treatment, wheezing isn't heard on auscultation. 4. The child's mother reports that the child sometimes forgets to take the inhalers.

3. Before a respiratory therapy treatment, wheezing isn't heard on auscultation. Typically, before a respiratory therapy treatment, wheezing has increased and the client has increased respiratory distress. No wheezing on auscultation is an indication that the child isn't moving air in and out and is in respiratory distress. A respiratory rate of 24 breaths/minute in an 8-year-old child is normal. An oxygen saturation of 95% is somewhat of a concern, possibly indicating that the client needs oxygen or needs to clear the airways. The fact that the mother makes the 8-year-old child responsible for taking medications is of concern and needs to be investigated, but this isn't as important at this time as the lack of wheezing.

When performing cardiopulmonary resuscitation on a 7-month-old, which location would the nurse use to evaluate the presence of a pulse? 1. Carotid artery 2. Femoral artery 3. Brachial artery 4. Radial artery

3. Brachial artery The brachial artery is the best location for evaluating the pulse of an infant younger than age 1. A child of this age has a very short and often fat neck, so the carotid artery is inaccessible. The femoral artery is usually inaccessible because of clothing and diapers. The radial artery may not be palpable if cardiac output is low, even if there is a heart beat.

For a child with a circumferential chest burn, what is the most important factor for the nurse to assess? 1. Wound characteristics 2. Body temperature 3. Breathing pattern 4. Heart rate

3. Breathing pattern All of the options are important. However, breathing pattern is the most important factor to assess because eschar impedes chest expansion in a child with a circumferential chest burn, causing breathing difficulty.

An infant undergoes surgery to remove a myelomeningocele. To detect increased intracranial pressure (ICP) as early as possible, the nurse should stay alert for which postoperative finding? 1. Decreased urine output 2. Increased heart rate 3. Bulging fontanels 4. Sunken eyeballs

3. Bulging fontanels Because an infant's fontanels remain open, the skull may expand in response to increased ICP. Therefore, bulging fontanels are a cardinal sign of increased ICP in an infant. Decreased urine output and sunken eyeballs indicate dehydration, not increased ICP. With increased ICP, the heart rate decreases.

How should the nurse prepare a suspension before administration? 1. By diluting it with normal saline solution 2. By diluting it with 5% dextrose solution 3. By shaking it so that all the drug particles are dispersed uniformly 4. By crushing remaining particles with a mortar and pestle

3. By shaking it so that all the drug particles are dispersed uniformly The nurse should shake a suspension before administration to disperse drug particles uniformly. Diluting the suspension and crushing particles aren't recommended for this drug form.

When assessing a preschooler who has sustained a head trauma, the nurse notes that the child appears to be obtunded. Which of the following denotes the child's level of consciousness? 1. No motor or verbal response to noxious (painful) stimuli 2. Remains in a deep sleep; responsive only to vigorous and repeated stimulation 3. Can be aroused with stimulation 4. Limited spontaneous movement; sluggish speech

3. Can be aroused with stimulation The child is obtunded if he can be aroused with stimulation. If the child shows no motor or verbal response to noxious stimuli, he's comatose. If the child remains in a deep sleep and is responsive only to vigorous and repeated stimulation, he's stuporous. If the child has limited spontaneous movement and sluggish speech, he's lethargic.

A toddler is receiving an infusion of total parenteral nutrition (TPN) via a Broviac catheter. As the child plays, the I.V. tubing becomes disconnected from the catheter. What should the nurse do first? 1. Turn off the infusion pump. 2. Position the child on the side. 3. Clamp the catheter. 4. Flush the catheter with heparin.

3. Clamp the catheter. First, the nurse must clamp the catheter to prevent air entry, which could lead to air embolism. If an air embolism occurs, the nurse should position the child on the side after clamping the catheter. The nurse may turn off the infusion pump after ensuring the child's safety. If blood has backed up in the catheter, the nurse may need to flush the catheter with heparin; however, this isn't the initial priority.

A parent brings a toddler, age 19 months, to the clinic for a regular checkup. When palpating the toddler's fontanels, what should the nurse expect to find? 1. Closed anterior fontanel and open posterior fontanel 2. Open anterior fontanel and closed posterior fontanel 3. Closed anterior and posterior fontanels 4. Open anterior and posterior fontanels

3. Closed anterior and posterior fontanels By age 18 months, the anterior and posterior fontanels should be closed. The diamond-shaped anterior fontanel normally closes between ages 9 and 18 months. The triangular posterior fontanel normally closes between ages 2 and 3 months.

A nurse is preparing for the admission of a child with a diagnosis of acute-stage Kawasaki disease. On assessment of the child, the nurse expects to note which clinical manifestation of the acute stage of the disease? 1. Cracked lips 2. Normal appearance 3. Conjunctival hyperemia 4. Desquamation of the skin

3. Conjunctival hyperemia Kawasaki disease is also known as mucocutaneous lymph node syndrome and is an acute systemic inflammatory illness. In the acute stage, the child has a fever, conjunctival hyperemia, red throat, swollen hands, rash, and enlargement of the cervical lymph nodes. In the subacute stage, cracking lips and fissures, desquamation of the skin on the tips of the fingers and toes, joint pain, cardiac manifestations, and thrombocytosis occur. In the convalescent stage, the child appears normal, but signs of inflammation may be present.

Which action illustrates the responsibilities of a pediatric case manager on the pediatric orthopedic unit? 1. Providing direct client care 2. Writing orders in the medical chart 3. Consulting with health care providers to make sure the client is following the critical pathway 4. Assisting the orthopedic surgeon in the operating room

3. Consulting with health care providers to make sure the client is following the critical pathway Case managers follow a group of clients, ensuring that their care follows the appropriate critical pathway. These pathways contain a timeline designed to coordinate the multidisciplinary team toward a common goal of providing a short, safe, and healthy length of stay in the hospital. Registered nurses handle most of the direct bedside client care, while physicians and nurse practitioners are responsible for writing medical orders. The circulating nurse and scrub nurse work in the operating room, assisting the orthopedic surgeon.

A mother calls the clinic to report that her 9-month-old infant has diarrhea. Upon further questioning, the nurse determines that the child has mild diarrhea and no signs of dehydration. Which advice is most appropriate to give this mother? 1. Call back if your baby has 10 stools in 1 day. 2. Feed your baby clear liquids only. 3. Continue your baby's normal feedings. 4. Notify your baby's day care of his illness.

3. Continue your baby's normal feedings. If an infant has mild diarrhea, his mother should be advised to continue his normal diet and to call back if the diarrhea doesn't stop or if he shows signs of dehydration. Notifying the day care about the infant's illness is important but doesn't take priority.

When meeting with a family who'll learn that their 3-year-old is seriously ill, which action demonstrates the nurse's role as collaborator of care? 1. Providing the parents with information about financial assistance programs. 2. Informing the family of the diagnosis and recently discovered findings. 3. Coordinate the multidisciplinary services and providing information about them. 4. Referring and consulting with other specialties to help in treating the diagnosis.

3. Coordinate the multidisciplinary services and providing information about them. The nurse can coordinate care when multiple services are involved, explaining the function of service (social service, case management, counseling services, and so forth). For instance, providing parents with information about financial assistance programs is the responsibility of social services. Informing the family of the diagnosis and recently discovered findings is a physician's responsibility as is referring and consulting with other specialties.

A child is suspected of having amblyopia ("lazy eye"). To help diagnose this disorder, the child will undergo which test? 1. Snellen's test 2. Near vision test 3. Cover-uncover test 4. Peripheral vision test

3. Cover-uncover test The cover-uncover test assesses the fusion reflex, which makes binocular vision possible. During this test, the "lazy eye" exhibits wandering. Snellen's test assesses visual acuity, the near vision test evaluates near vision, and the peripheral vision test evaluates peripheral vision.

A mother brings her 4-month-old to the clinic for a wellness checkup. Which immunizations should the infant receive? 1. Diphtheria, tetanus toxoids, and acellular pertussis (DTaP), inactivated polio virus (IPV), rotavirus, and measles-mumps-rubella (MMR) 2. Haemophilus influenzae type B (Hib), rotavirus, DTaP, and IPV 3. DTaP, IPV, Hib, hepatitis B, and pneumococcal conjugate vaccine (PCV) 4. DTaP, hepatitis B, Hib, and varicella

3. DTaP, IPV, Hib, hepatitis B, and pneumococcal conjugate vaccine (PCV) DTaP, IPV, Hib, hepatitis B, and PCV are administered at ages 2 and 4 months. Rotavirus vaccine is no longer recommended because of the associated risk of intussusception. The MMR vaccine is typically administered at age 15 months. The varicella vaccine is commonly administered between ages 12 and 18 months.

When the nurse assesses a 2-year-old child with suspected dehydration, which condition should be reported to the physician immediately? 1. Irritability for the past 12 hours 2. Capillary refill less than 2 seconds 3. Decreased blood pressure 4. Tachycardia, dry skin, and dry mucous membranes

3. Decreased blood pressure Decreased blood pressure is a late sign of dehydration that usually indicates that the dehydration is severe. This is because compensatory mechanisms in children are able to sustain blood pressure in the low-normal range for some time. Irritability, capillary refill less than 2 seconds, tachycardia, dry skin, and dry mucous membranes are all early signs of dehydration.

The nurse is obtaining the history of a pediatric client, age 4. Which area usually takes longer to evaluate in a child than in an adult? 1. Past health status 2. Family health status 3. Developmental status 4. Review of physiologic systems

3. Developmental status Because children undergo rapid physiologic and psychological changes that affect growth and development, evaluating development usually takes longer in a child and involves more detail. Obtaining information about the child's achievement of specific developmental milestones is essential. Typically, the past health status, family health status, and review of physiologic systems take no longer to assess in a child than in an adult.

Which sign is least likely to indicate abuse in a 4-year-old child? 1. Conflicting stories about the accident or injury from the parents 2. History inconsistent with the child's developmental level 3. Disheveled parental appearance and low socioeconomic status 4. Exaggerated or absent emotional response by the caregiver

3. Disheveled parental appearance and low socioeconomic status The physical appearance and income level of parents aren't indicators of expected or potential abuse. Nurses must be aware of their biases regarding child abuse. Conflicting stories about the accident or injury from the parents, history inconsistent with the child's developmental level, and an inappropriate response by the parents, such as an exaggerated or absent emotional response, are warning signs of abuse.

A nurse is monitoring a 3-month-old infant for signs of increased intracranial pressure. On palpation of the fontanels, the nurse notes that the anterior fontanel is soft and flat. Based on this finding, which nursing action is appropriate? 1. Increase oral fluids. 2. Notify the physician. 3. Document the finding. 4. Elevate the head of the bed to 90 degrees.

3. Document the finding. The anterior fontanel is diamond-shaped and located on the top of the head. The fontanel should be soft and flat in a normal infant, and it normally closes by 12 to 18 months of age. The nurse would document the finding because it is normal. There is no useful reason to increase oral fluids, notify the physician, or elevate the head of the bed to 90 degrees.

A nurse assesses the vital signs of a 12-month-old infant with a respiratory infection and notes that the respiratory rate is 35 breaths/ min. Based on this finding, which action is appropriate? 1. Administer oxygen. 2. Notify the physician. 3. Document the findings. 4. Reassess the respiratory rate in 15 minutes.

3. Document the findings. The normal respiratory rate in a 12-month -old infant is 20 to 40 breaths/ min. The normal apical heart rate is 90 to 130 beats/ min, and the average blood pressure is 90/ 56 mm Hg. The nurse would document the findings.

The nurse is finishing her shift on the pediatric unit. Because her shift is ending, which intervention takes top priority? 1. Changing the linens on the clients' beds 2. Restocking the bedside supplies needed for a dressing change on the upcoming shift 3. Documenting the care provided during her shift 4. Emptying the trash cans in the assigned client rooms

3. Documenting the care provided during her shift Documentation should take top priority. Documentation is the only way the nurse can legally claim that interventions were performed. The other three options would be appreciated by the nurses on the oncoming shift but aren't mandatory and don't take priority over documentation.

The nurse is about to give an I.M. injection to a 2-year-old child. Which site should be avoided? 1. Vastus lateralis muscle 2. Ventrogluteal muscle 3. Dorsogluteal muscle 4. Deltoid muscle

3. Dorsogluteal muscle The dorsogluteal muscle isn't sufficiently developed to be a safe injection site for infants and small children. The vastus lateralis, ventrogluteal, and deltoid muscles are considered safe sites for infants and small children.

A child with hemophilia is hospitalized after falling. Now the child complains of severe pain in the left wrist. What should the nurse do first? 1. Perform passive range-of-motion (ROM) exercises on the wrist. 2. Massage the wrist and apply a warm compress. 3. Elevate the affected arm and apply ice to the injury site. 4. Notify the physician.

3. Elevate the affected arm and apply ice to the injury site. Severe joint pain in a child with hemophilia indicates bleeding. During a bleeding episode, the nurse should elevate the affected extremity and apply ice to the injury site to promote vasoconstriction. ROM exercises may worsen discomfort and bleeding. Massage and warm compresses also may increase bleeding. The nurse should notify the physician after taking measures to stop the bleeding.

A child with a fractured left femur receives a cast. A short time later, the nurse notices that the toes on the child's left foot are edematous. Which nursing action would be most appropriate? 1. Applying ice to the foot 2. Massaging the toes 3. Elevating the foot of the bed 4. Placing the child on the right side

3. Elevating the foot of the bed To relieve edema of the toes, the nurse should raise the affected extremity above heart level such as by elevating the foot of the bed. The other options wouldn't reduce swelling.

When a toddler with croup is admitted to the facility, the physician orders treatment with a mist tent. As the parent attempts to put the toddler in the crib, the toddler cries and clings to the parent. What is the nurse's best approach for gaining the child's cooperation with the treatment? 1. Turn off the mist so the noise doesn't frighten the toddler. 2. Let the toddler sit on the parent's lap next to the mist tent. 3. Encourage the parent to stand next to the crib and stay with the child. 4. Put the side rail down so the toddler can get into and out of the crib unaided.

3. Encourage the parent to stand next to the crib and stay with the child. By encouraging the parent to stand next to the crib and stay with the child, the nurse promotes compliance with treatment while minimizing the toddler's separation anxiety. Because the mist helps thin secretions and make them easier to clear, turning off the mist or letting the toddler sit next to the mist tent defeats the treatment's purpose. To prevent falls, the nurse should keep the side rails up and shouldn't permit the toddler to climb into and out of the crib.

A clinic nurse reviews the record of a child just seen by a physician and diagnosed with suspected aortic stenosis. The nurse expects to note documentation of which clinical manifestation specifically found in this disorder? 1. Pallor 2. Hyperactivity 3. Exercise intolerance 4. Gastrointestinal disturbances

3. Exercise intolerance Aortic stenosis is a narrowing or stricture of the aortic valve, causing resistance to blood flow in the left ventricle, decreased cardiac output, left ventricular hypertrophy, and pulmonary vascular congestion. A child with aortic stenosis shows signs of exercise intolerance, chest pain, and dizziness when standing for long periods. Pallor may be noted, but is not specific to this type of disorder alone. Options 2 and 4 are not related to this disorder.

The parents of an adolescent girl have recently learned that their daughter has a terminal illness. At first, as they try to cope, they display avoidance behaviors. Then they demonstrate behaviors that indicate possible acceptance of the diagnosis. Which behavior would indicate acceptance? 1. Failure to recognize the seriousness of the child's condition despite physical evidence 2. Intellectualization about the illness in areas unrelated to the child's condition 3. Expression of feelings, such as sorrow and anger, about the child's condition 4. Avoidance of staff, family members, or the child

3. Expression of feelings, such as sorrow and anger, about the child's condition The ability to express feelings and relate them to the diagnosis is the first step in accepting the situation. Failing to recognize the seriousness of the child's condition despite physical evidence, intellectualizing about the illness in areas unrelated to the child's condition, and avoiding staff, family members, or the child are all avoidance behaviors that represent a parent's inability to cope with the situation.

The nurse is assigned to an adolescent. Which nursing diagnosis is most appropriate for a hospitalized adolescent? 1. Anxiety related to separation from parents 2. Fear related to the unknown 3. Fear related to altered body image 4. Ineffective coping related to activity restrictions

3. Fear related to altered body image Fear related to altered body image is the most appropriate nursing diagnosis for a hospitalized adolescent because of the adolescent's developmental level and concern for physical appearance. An adolescent may fear disfigurement resulting from procedures and treatments. Separation is rarely a major stressor for the adolescent, eliminating a diagnosis of Anxiety related to separation from parents. Adolescents may have Fear related to the unknown but typically ask questions if they want information. A diagnosis of Ineffective coping related to activity restrictions may be appropriate for a toddler who has difficulty tolerating activity restrictions but is an unlikely nursing diagnosis for an adolescent.

A 6-year-old client has tested positive for West Nile virus infection. The nurse suspects the client has the severe form of the disease when she recognizes which signs and symptoms? 1. Fever, rash, and malaise 2. Anorexia, nausea, and vomiting 3. Fever, muscle weakness, and change in mental status 4. Fever, lymphadenopathy, and rash

3. Fever, muscle weakness, and change in mental status Severe West Nile virus infection (also called West Nile encephalitis or West Nile meningitis) affects the central nervous system and may cause headache, neck stiffness, fever, muscle weakness or paralysis, changes in mental status, and seizures. Such signs and symptoms as fever, rash, malaise, anorexia, nausea and vomiting, and lymphadenopathy suggest the mild form of West Nile virus infection.

A toddler with hemophilia is hospitalized with multiple injuries after falling off a sliding board. X-rays reveal no bone fractures. When caring for the child, what is the nurse's highest priority? 1. Administering platelets as prescribed 2. Taking measures to prevent infection 3. Frequently assessing the child's level of consciousness (LOC) 4. Discussing a safe play environment with the parents

3. Frequently assessing the child's level of consciousness (LOC) In hemophilia, one of the factors required for blood clotting is absent, significantly increasing the risk of hemorrhage after injury. The nurse must assess the child frequently for signs and symptoms of intracranial bleeding, such as an altered LOC, slurred speech, vomiting, and headache. To manage hemophilia, the absent blood clotting factor is replaced via I.V. infusion of factor, cryoprecipitate, or fresh frozen plasma; this may be done prophylactically or after a traumatic injury. Platelet transfusions aren't necessary. Hemophiliacs aren't at increased risk for infection. Discussing a safe play environment with the parents is important but isn't the highest priority.

Craniocerebral injury in a child differs substantially from craniocerebral trauma in an adult. Which of the following identifies a negative difference between children and adults that could produce a life-threatening complication for a child? 1. Cerebral tissues in children are softer, thinner, and more flexible. 2. A child's skull can expand more than an adult's can. 3. Greater portions of a child's blood volume flows to the head. 4. Hematomas in children can include subdural, epidural, and intracerebral.

3. Greater portions of a child's blood volume flows to the head. If hemorrhage is associated with a head injury and it goes undetected, a child may experience hypovolemic shock because a large portion of a child's blood volume goes to the head. In children, cerebral tissues are softer, thinner, and more flexible — conditions that permit diffusion of the impact. Because a child's skull can expand more than an adult's can, a greater amount of posttraumatic edema can occur without evidence of neurologic deficits. Subdural, epidural, and intracerebral hematomas are the different types of head injury that can occur in children and adults.

An adolescent is diagnosed with iron deficiency anemia. After emphasizing the importance of consuming dietary iron, the nurse asks the child to select iron-rich breakfast items from a sample menu. Which selection demonstrates knowledge of dietary iron sources? 1. Grapefruit and toast 2. Pancakes and a banana 3. Ham and eggs 4. Bagel and cream cheese

3. Ham and eggs Good sources of dietary iron include red meat, egg yolks, whole wheat breads, seafood, nuts, legumes, iron-fortified cereals, and green, leafy vegetables. Fresh fruits and milk products contain only small amounts of iron. Breads (except for whole wheat and iron-fortified breads) aren't good iron sources.

Which technique is most effective in preventing nosocomial infection transmission when caring for a preschooler? 1. Client isolation 2. Standard precautions 3. Hand washing 4. Needleless syringe system

3. Hand washing Hand washing is the single most important measure for preventing infection transmission. Isolating the child and using infection control precautions are required for certain diseases, such as varicella, diphtheria, mumps, pertussis, measles, and meningitis. Standard precautions, which include hand washing, are guidelines for treating all clients as potentially infectious. A needleless syringe system will prevent transmission through needle sticks but not from body fluid contact.

A preschool-age child scheduled for surgery in the morning is admitted to the facility for the first time. Which nursing action would ease the child's anxiety? 1. Beginning preoperative teaching as soon as possible 2. Explaining that the child will be "put to sleep" during the operation and will feel nothing 3. Having the child act out the surgical experience using dolls and medical equipment 4. Explaining preoperative and postoperative procedures step by step

3. Having the child act out the surgical experience using dolls and medical equipment Having the child act out the surgical experience using dolls and medical equipment would ease anxiety and give the nurse an opportunity to clarify the child's misconceptions. Preschoolers have a limited concept of time, so the nurse should provide preoperative teaching just before surgery rather than starting it as soon as possible; also, a delay between teaching and surgery may heighten anxiety by giving the child a chance to worry or fantasize. The nurse should avoid using such phrases as "put to sleep" because these may have a dual or negative meaning to a young child. Long explanations are inappropriate for the preschooler's developmental level and may increase anxiety.

Which of the following would be an effective relaxation strategy for a school-age child to use during a painful procedure? 1. Having the child keep his eyes shut at all times 2. Having the child hold his breath and not yell 3. Having the child take a deep breath and blow it out until told to stop 4. Being honest with the child and telling him the procedure will hurt a lot

3. Having the child take a deep breath and blow it out until told to stop Having the child take a deep breath, then blow it out is a form of distraction and will help the child cope better with the procedure. A child may prefer to keep his eyes open during a procedure so he can see what is going on and can anticipate what is going to happen. Letting a child yell during a procedure is a form of distraction. Holding the breath isn't beneficial and could have adverse effects (such as feeling dizzy or faint). The nurse should prepare a child for a procedure by using nonpain descriptors and not suggesting pain. For example, the nurse might say, "Sometimes this feels like pushing or sticking, and sometimes it doesn't bother children at all."

The nurse is performing a Denver Developmental Screening Test II on a 4 1/2-year-old child. What behaviors should the nurse expect the child to demonstrate? 1. He balances on each foot for at least 6 seconds. 2. He copies a square using straight lines and square corners. 3. He prepares his own cereal without help. 4. He copies a circle that's closed or very nearly closed. 5. He speaks clearly. 6. He draws a person with at least three body parts.

3. He prepares his own cereal without help. 4. He copies a circle that's closed or very nearly closed. 5. He speaks clearly. 6. He draws a person with at least three body parts. By age 4 1/2, a child should be able to prepare a bowl of cereal without help, copy a circle, speak clearly, and draw a person using at least three body parts. The majority of children don't achieve balancing on each foot for 6 seconds until about age 5 1/2. Less than 25% of all children are able to correctly copy a square by age 4.

The physician prescribes terbutaline (Brethaire), 2.5 mg by mouth four times a day, for a child with bronchitis. If the child receives an I.V. infusion of terbutaline, which serious adverse reaction is possible? 1. Hypocalcemia 2. Hypercalcemia 3. Hypokalemia 4. Hyperkalemia

3. Hypokalemia The nurse should monitor the client receiving an I.V. infusion of terbutaline for hypokalemia, lactic acidosis, chest pain, arrhythmias, dyspnea, bloating, chills, or anaphylactic shock. Terbutaline doesn't cause calcium imbalances.

What is the best advice for a nurse to give to the parents of a 2-year-old child who frequently throws temper tantrums? 1. Move the toddler to a different setting. 2. Allow the toddler more choices. 3. Ignore the behavior when it happens. 4. Give into the toddler's demands.

3. Ignore the behavior when it happens. Ignore tantrum behavior because attention to the behavior can reinforce the undesirable behavior. Changing settings can increase the tantrum behavior. Allowing the toddler more choices may increase tantrum behavior if the toddler is unable to follow through with choices. The toddler should be offered only allowable and reasonable choices. It's ill-advised to give into the toddler's demands because doing so only promotes tantrum behavior.

A tuberculosis intradermal skin test to detect tuberculosis infection is given to a high-risk adolescent. How long after the test is administered should the results be evaluated? 1. Immediately after 2. Within 24 hours 3. In 48 to 72 hours 4. After 5 days

3. In 48 to 72 hours Tuberculin skin tests are tests of delayed hypersensitivity. If the test results are positive, a reaction should appear in 48 to 72 hours. Immediately after and within 24 hours are both too soon to observe a reaction. Waiting more than 5 days to evaluate the test is too long because any reaction may no longer be visible.

According to Erikson's theory of development, chronic illness can interfere with which stage of development in an 11-year-old child? 1. Intimacy versus isolation 2. Trust versus mistrust 3. Industry versus inferiority 4. Identity versus role confusion

3. Industry versus inferiority According to Erikson, an 11-year-old child is working through the stage of industry versus inferiority. Chronic illness may interfere with this stage of development in an 11-year-old child. Intimacy is the developmental task of a young adult. Trust is the developmental task to be achieved during infancy. Identity is the developmental task of adolescence.

According to Erikson's psychosocial theory of development, an 8-year-old child would be in which stage? 1. Trust versus mistrust 2. Initiative versus guilt 3. Industry versus inferiority 4. Identity versus role confusion

3. Industry versus inferiority In middle childhood, the 6- to 12-year-old child is mastering the task of industry versus inferiority. The trust versus mistrust task is in infancy (birth to 1 year). In early childhood, the 1- to 3-year-old child is in the stage of initiative versus guilt. Identity versus role confusion occurs during adolescence.

Which is the most appropriate nursing diagnosis for a preschool child with epiglottitis? 1. Anxiety related to separation from parent 2. Decreased cardiac output related to bradycardia 3. Ineffective airway clearance related to laryngospasm 4. Impaired gas exchange related to noncompliant lungs

3. Ineffective airway clearance related to laryngospasm Epiglottitis is an immediate threat to life because complete upper airway obstruction may occur suddenly and be precipitated by improper examination or intervention. The upper airway obstruction is the result of laryngospasm and edema. The client is anxious because of respiratory distress. The nurse should allow the parent to stay with the child and should encourage the parent to hold and reassure the child. The child has impaired gas exchange from impeded airflow, not from a noncompliant lung. The child will probably be tachycardic until respiratory failure ensues.

At the health clinic, a sexually active girl, age 15, tells the nurse she's worried that her parents may find out about her sexual activity. "They would never approve," she says. The nurse should formulate which nursing diagnosis? 1. Delayed growth and development related to sexual activity 2. Impaired social interaction related to boyfriend's expectations 3. Ineffective sexuality patterns related to parent's expectations 4. Fear related to boyfriend's expectations

3. Ineffective sexuality patterns related to parent's expectations This child is expressing concerns about the conflict between her parent's expectations and her own desires. Sexual activity is a normal experimental pattern for many adolescents, but this client verbalizes parental expectations against this behavior. No evidence suggests she's having a conflict with her boyfriend or problems with social interactions.

Which of the following is the least effective strategy for interviewing an adolescent? 1. Maintaining objectivity by avoiding assumptions, judgments, and lectures 2. Beginning with less sensitive issues and proceed to more sensitive ones 3. Interviewing adolescents with the parents present 4. Asking open-ended questions and moving to more directive questions when possible

3. Interviewing adolescents with the parents present When possible, adolescents should be interviewed without their parents present to ensure confidentiality and privacy. Avoid assumptions, judgments, and lectures to increase the adolescent's comfort in disclosing sensitive information. Begin with less sensitive questions so the adolescent won't feel threatened and uncomfortable and become uncooperative during the interview. Ask open-ended questions to give adolescents opportunities to share their psychosocial context.

A 4-year-old has just returned from surgery. He has a nasogastric (NG) tube in place and is attached to intermittent suction. The child says to the nurse, "I'm going to throw up." What should the nurse do first? 1. Notify the physician because the child has an NG tube. 2. Immediately give the child an antiemetic I.V. 3. Irrigate the NG tube to ensure patency. 4. Encourage the mother to calm the child down.

3. Irrigate the NG tube to ensure patency. If the NG tube isn't draining properly or is kinked, the child will experience nausea. Therefore, the nurse should assess the tube's patency and irrigate it as ordered. There is no reason to notify the physician immediately because a nurse should be able to handle the situation. Giving the child an antiemetic doesn't really address the problem. Encouraging the mother to calm the child is always a good intervention but isn't the first thing to do in this case.

When assessing a child with hemophilia, the nurse identifies which of the following as an early sign of hemarthrosis? 1. Decreased peripheral pulses 2. Active bleeding 3. Joint stiffness 4. Hematuria

3. Joint stiffness Joint stiffness is an early sign of hemarthrosis. Hemarthrosis doesn't affect pulses. Bleeding into the joints can't be observed directly. Hematuria indicates bleeding in the urinary tract.

A 10-year-old boy falls, injures his left shoulder, and is taken to the emergency department. While the client waits to be seen by the physician, what intervention should the nurse perform first? 1. Apply a warm compress to the injured shoulder. 2. Ask him to demonstrate full range of motion of his left arm. 3. Keep him in a comfortable position and apply ice to the injured shoulder. 4. Give him a nonnarcotic analgesic for pain.

3. Keep him in a comfortable position and apply ice to the injured shoulder. Ice would be applied first to reduce swelling and pain. The client should also be helped into a comfortable position. Warm compresses may increase swelling and cause bleeding into the injured tissue. Demonstrating full range of motion of the left arm may cause further damage to the injured area. In the emergency department, the nurse must have a physician's order to administer an analgesic.

An 8-year-old child is suspected of having meningitis. Signs of meningitis include which of the following? 1. Cullen's sign 2. Koplik's spots 3. Kernig's sign 4. Chvostek's sign

3. Kernig's sign In Kernig's sign, the client is in the supine position with knees flexed; a leg is then flexed at the hip so that the thigh is brought to a position perpendicular to the trunk. An attempt is then made to extend the knee. If meningeal irritation is present, the knee can't be extended and attempts to extend the knee result in pain. Other common symptoms include stiff neck, headache, and fever. Cullen's sign is the bluish discoloration of the periumbilical skin due to intraperitoneal hemorrhage. Koplik's spots are reddened areas with grayish blue centers that are found on the buccal mucosa of a client with measles. Chvostek's sign is elicited by tapping the client's face lightly over the facial nerve, just below the temple. A calcium deficit is suggested if the facial muscles twitch.

The nurse has just administered a drug to a child. Which organ is most responsible for drug excretion in children? 1. Heart 2. Lungs 3. Kidneys 4. Liver

3. Kidneys The kidneys are most responsible for drug excretion in children. Less commonly, some drugs may be excreted via the lungs or liver. Drugs are never excreted by the heart in children or adults.

After assessing an adolescent with sickle cell anemia, the nurse formulates a nursing diagnosis of Impaired skin integrity. Which assessment finding best supports this nursing diagnosis? 1. Swelling of the hands and feet 2. Petechiae 3. Leg ulcers 4. Hemangiomas

3. Leg ulcers In sickle cell anemia, sickling of red blood cells leads to increased blood viscosity and impaired circulation. Diminished peripheral circulation makes the adolescent or adult with sickle cell anemia susceptible to chronic leg ulcers. In children under age 2 who have sickle cell anemia, swelling of the hands and feet (hand-foot syndrome) commonly occurs during a vaso-occlusive crisis as a result of infarction of short tubular bones. Petechiae aren't associated specifically with sickle cell anemia. Hemangiomas, benign tumors of dilated blood vessels, aren't linked to sickle cell anemia.

An adolescent, age 17, with acute lymphoblastic leukemia is discharged with written information about chemotherapy administration and the outpatient appointment schedule. The child now is in the maintenance phase of chemotherapy but has missed clinic appointments for blood work and admits to omitting some chemotherapy doses. To improve the client's compliance, the nurse should include which intervention in the care plan? 1. Emphasizing the long-term consequences of noncompliance 2. Reprimanding the client for failing to comply 3. Letting the client participate in the planning and scheduling of treatments 4. Threatening to discontinue care if the client doesn't comply

3. Letting the client participate in the planning and scheduling of treatments Because the adolescent is striving for independence, health care providers should promote self-reliance whenever possible such as by letting the child participate in the planning and scheduling of treatments. The client can help establish realistic goals and evaluation outcomes as well as help schedule procedures and chemotherapy doses to minimize lifestyle disruptions. Adolescents are oriented in the present and have relatively little concern for the long-term consequences of their behavior. Reprimanding the client or threatening to discontinue care isn't likely to improve compliance and isn't in the client's best interest.

The mother of a preschooler recently diagnosed with type 1 diabetes mellitus makes an urgent call to the pediatrician's office. She says her child had an uncontrollable temper tantrum while playing and now is lethargic and hard to arouse. The nurse should instruct the mother to take which action first? 1. Obtain a urine sample and measure the glucose level. 2. Force the child to drink orange juice. 3. Measure the child's blood glucose level. 4. Call 911 because this is an emergency.

3. Measure the child's blood glucose level. In a child type 1 diabetes mellitus, behavioral changes may signal either hypoglycemia or hyperglycemia; measuring the blood glucose level is the only way to determine which condition is present. Urine glucose measurement doesn't accurately reflect the current blood glucose level. Forcing a lethargic child to drink fluids could cause aspiration. After measuring the child's blood glucose level, the mother may need to take additional emergency measures such as administering insulin or a simple glucose source. If the child doesn't respond to these measures, she may need to call for emergency help.

The nurse is performing a psychosocial assessment on an adolescent, age 14. Which emotional response is typical during early adolescence? 1. Frequent anger 2. Cooperativeness 3. Moodiness 4. Combativeness

3. Moodiness During early adolescence, a child may become moody. Frequent anger and combativeness are more typical of middle adolescence. Cooperativeness typically occurs during late adolescence.

The nurse is caring for a school-age child with cerebral palsy. The child has difficulty eating using regular utensils and requires a lot of assistance. Which of the following referrals is most appropriate? 1. Registered dietitian 2. Physical therapist 3. Occupational therapist 4. Nursing assistant

3. Occupational therapist An occupational therapist helps physically disabled clients adapt to physical limitations and is most qualified to help a child with cerebral palsy eat and perform other activities of daily living. A registered dietitian manages and plans for the nutritional needs of children with cerebral palsy but isn't trained in modifying or fitting utensils with assistive devices. A physical therapist is trained to help a child with cerebral palsy gain function and prevent further disability but not to assist the child in performing activities of daily living. A nursing assistant can help a child eat; however, the nursing assistant isn't trained in modifying utensils.

After assessing a newly admitted 5-year-old child, the nurse makes the nursing diagnosis of Parental role conflict related to child's hospitalization. Which defining characteristic would most suggest this diagnosis? 1. Supportive child-parent interaction (speaking, listening, touching, and eye-to-eye contact) 2. Parents' active participation in child's physical or emotional care 3. Parents' expression of feelings of inadequacy in providing for their child's needs 4. Evidence of adaptation to parental role changes

3. Parents' expression of feelings of inadequacy in providing for their child's needs Expression of feelings of inadequacy in providing for their child's needs is a defining characteristic for this nursing diagnosis. Supportive child-parent interaction, parents' active participation in the child's care, and evidence of adaptation to parental role changes don't suggest this diagnosis.

Assessment findings of an infant admitted to the hospital reveal a machinery-like murmur on auscultation of the heart and signs of congestive heart failure. The nurse reviews congenital cardiac anomalies and identifies the infant's condition as which of the following? Refer to the circled area in the figure to determine the condition. 1. Aortic stenosis 2. Atrial septal defect 3. Patent ductus arteriosus 4. Ventricular septal defect

3. Patent ductus arteriosus A patent ductus arteriosus is failure of the fetal ductus arteriosus (artery connecting the aorta and the pulmonary artery) to close. A characteristic machinery-like murmur is present, and the infant may show signs of congestive heart failure. Aortic stenosis is a narrowing or stricture of the aortic valve. Atrial septal defect is an abnormal opening between the atria. Ventricular septal defect is an abnormal opening between the right and left ventricles.

A child, age 4, is brought to the clinic for a routine examination. When observing the tympanic membrane, the nurse identifies which color as normal? 1. Light pink 2. Deep red 3. Pearly gray 4. Yellowish white

3. Pearly gray The tympanic membrane normally appears gray, shiny, and translucent. A light pink, deep red, or yellowish white tympanic membrane is abnormal.

An adolescent client is admitted to the adolescent unit with pain caused by sickle cell crisis. Who should be consulted first for this client's care? 1. Nutritionist 2. Physical therapist 3. Pediatric pain specialist 4. Case manager

3. Pediatric pain specialist Children hospitalized with sickle cell crisis are often in excruciating pain. Therefore, the pediatric pain specialist should be consulted first to help relieve the client's pain. The client also requires hydration with I.V. fluids, but consulting a nutritionist isn't important at this time. Bed rest is commonly ordered to minimize the client's energy expenditure and oxygen demand; therefore, consulting a physical therapist isn't necessary at this time. It isn't necessary to consult the case manager first; pain relief is most important at this time.

A 3-month-old infant just had a cleft lip and palette repair. To prevent trauma to the operative site, the nurse should do which of the following? 1. Give the baby a pacifier to help soothe him. 2. Lie the baby in the prone position. 3. Place the infant's arms in soft elbow restraints. 4. Avoid touching the suture line, even to clean.

3. Place the infant's arms in soft elbow restraints. Soft restraints from the upper arm to the wrist prevent the infant from touching his lip but allow him to hold a favorite item such as a blanket. Because they could damage the operative site, such objects as pacifiers, suction catheters, and small spoons shouldn't be placed in a baby's mouth after cleft palette repair. A baby in a prone position may rub his face on the sheets and traumatize the operative site. The suture line should be cleaned gently to prevent infection, which could interfere with healing and damage the cosmetic appearance of the repair. Dried blood collecting on the suture line can widen the scar.

When assessing a toddler, age 18 months, the nurse should interpret which reflex as a sign of a neurologic dysfunction? 1. Positive gag reflex 2. Positive tonic neck reflex 3. Positive Babinski's reflex 4. Positive corneal reflex

3. Positive Babinski's reflex Babinski's reflex should disappear by age 12 months; its presence after this age indicates neurologic dysfunction. The gag reflex, tonic neck reflex, and corneal reflex are normal findings for a toddler.

It's important to assess the maturity of enzyme systems (kidney and liver) in which pediatric population before administering medications? 1. Adolescents 2. Newborns 3. Premature infants 4. Toddlers

3. Premature infants Factors related to growth and maturation significantly alter an individual's capacity to metabolize and excrete drugs. Thus, the premature infant is at risk for problems because of immaturity. Deficiencies associated with immaturity become more important with decreasing age. Enzyme systems develop quickly, with most increasing to adult levels within 1 to 8 weeks after birth. Within the 1st year of life, all are probably as active as they will ever be.

A child, age 4, is hospitalized because of alleged sexual abuse. What is the best nursing intervention for this child? 1. Avoiding touching the child 2. Preventing the suspected abuser from visiting the child 3. Providing play situations that allow disclosure 4. Discouraging the child from talking about what happened

3. Providing play situations that allow disclosure Through certain play situations, a sexually abused child can disclose information without actually talking about himself or herself. An abused child needs to be touched and cared for like any other hospitalized child. The nurse can't restrict visitation unless the threat of repeated abuse exists while the child is hospitalized. The nurse shouldn't discourage discussion of the abuse if the child feels able to talk about it.

A toddler is having a tonic-clonic seizure. What should the nurse do first? 1. Restrain the child. 2. Place a tongue blade in the child's mouth. 3. Remove objects from the child's surroundings. 4. Check the child's breathing.

3. Remove objects from the child's surroundings. During a seizure, the nurse's first priority is to protect the child from injury. To prevent injury caused by uncontrolled movements, the nurse must remove objects from the child's surroundings and pad objects that can't be removed. Restraining the child or placing an object in the child's mouth during a seizure may cause injury. Once the seizure stops, the nurse should check for breathing and, if indicated, initiate rescue breathing.

A 4-year-old child is being treated for status asthmaticus. His arterial blood gas analysis reveals a pH of 7.28, PaCO2 of 55 mm Hg, and HCO3− of 26 mEq/L. What condition do these findings indicate? 1. Respiratory alkalosis 2. Metabolic acidosis 3. Respiratory acidosis 4. Metabolic alkalosis

3. Respiratory acidosis A pH less than 7.35 and a PaCO2 greater than 45 mm Hg indicate respiratory acidosis. Status asthmaticus is a medical emergency that's characterized by respiratory distress. Persistent hypoventilation leads to the accumulation of carbon dioxide, resulting in respiratory acidosis.

The parent of a preschooler with chickenpox asks the nurse about measures to make the child comfortable. The nurse instructs the parent to avoid administering aspirin or any other product that contains salicylates. When given to children with chickenpox, aspirin has been linked to which disorder? 1. Guillain-Barré syndrome 2. Rheumatic fever 3. Reye's syndrome 4. Scarlet fever

3. Reye's syndrome Research shows a correlation between the use of aspirin during chickenpox and the development of Reye's syndrome (a disorder characterized by brain and liver toxicity). Therefore, the nurse should instruct the parents to avoid administering aspirin or other products that contain salicylates and to consult the physician or pharmacist before administering any medication to a child with chickenpox. No research has found a link between aspirin use, chickenpox, and the development of Guillain-Barré syndrome, rheumatic fever, or scarlet fever.

During a well-baby visit, a mother asks the nurse when she should start giving her infant solid foods. The nurse should instruct her to introduce which solid food first? 1. Applesauce 2. Egg whites 3. Rice cereal 4. Yogurt

3. Rice cereal Rice cereal is the first solid food an infant should receive because it's easy to digest and is associated with few allergies. Next, the infant can receive pureed fruits, such as bananas, applesauce, and pears, followed by pureed vegetables, egg yolks, cheese, yogurt and, finally, meat. Egg whites shouldn't be given until age 9 months because they may trigger a food allergy.

While preparing to discharge a 9-month-old infant who's recovering from gastroenteritis and dehydration, the nurse teaches the parents about their infant's dietary and fluid requirements. The nurse should include which other topic in the teaching session? 1. Nursery schools 2. Toilet training 3. Safety guidelines 4. Preparation for surgery

3. Safety guidelines The nurse always should reinforce safety guidelines when teaching parents how to care for their child. By giving anticipatory guidance, the nurse can help prevent many accidental injuries. For parents of a 9-month-old infant, it's too early to discuss nursery schools or toilet training. Because surgery isn't used to treat gastroenteritis, this topic is inappropriate.

A child, age 3, with lead poisoning is admitted to the facility for chelation therapy. The nurse must stay alert for which adverse effect? 1. Anaphylaxis 2. Fever and chills 3. Seizures 4. Heart failure

3. Seizures Chelation therapy removes lead by combining it with another substance to form a soluble compound that the kidneys can excrete. As lead is mobilized from bone and other tissues, the serum lead level rises rapidly, increasing the client's risk of seizures. Chelation therapy doesn't cause anaphylaxis, fever, chills, or heart failure.

A dehydrated infant is receiving I.V. therapy. The mother tells the nurse she wants to hold her infant but is afraid this might cause the I.V. line to become dislodged. What should the nurse do? 1. Tell the mother it's best not to move the infant now. 2. Inform the mother that only a nurse should hold the infant during I.V. therapy. 3. Show the mother how to hold the infant properly. 4. Advise the mother to let the infant lie quietly in bed.

3. Show the mother how to hold the infant properly. Infants with I.V. lines should be held with care. The nurse should encourage and show the mother how to hold the infant properly and teach her about I.V. care measures to enhance her confidence and skill. The nurse should encourage the mother to participate in the child's care whenever possible.

An infant, age 8 months, has a tentative diagnosis of congenital heart disease. During physical assessment, the nurse measures a heart rate of 170 beats/minute and a respiratory rate of 70 breaths/minute. How should the nurse position the infant? 1. Lying on the back 2. Lying on the abdomen 3. Sitting in an infant seat 4. Sitting in high Fowler's position

3. Sitting in an infant seat Because the infant's assessment findings suggest that respiratory distress is developing, the nurse should position the infant with the head elevated at a 45-degree angle to promote maximum chest expansion; an infant seat maintains this position. Placing an infant flat on the back or abdomen or in high Fowler's position could increase respiratory distress by preventing maximal chest expansion.

A 6-year-old client was admitted to the pediatric unit after sustaining a broken leg in a motor vehicle accident. Which specialist would be most important to involve in this client's care during hospitalization? 1. Home care nurse 2. Nutritionist 3. Social worker 4. Infectious disease nurse

3. Social worker The nurse should collaborate with the social worker to provide care for the client involved in a motor vehicle accident. After such a traumatic life event, this client's care will involve dealing with his emotional health as well as his physical recovery. Home health care isn't usually needed for this type of injury, and nutrition isn't a top priority problem for this client. There's nothing to suggest that the infectious disease nurse is required to care for this client.

A child, age 4, fell and broke his arm. After assisting the physician in applying a cast, the nurse should include which intervention in the immediate cast care? 1. Rest the cast on the nearest table 2. Dispose of the water containing plaster in the sink 3. Support the cast with the palms of her hands 4. Allow the cast to dry before cleaning surrounding skin

3. Support the cast with the palms of her hands After a cast is applied, the nurse should support it with the palms of her hands. Later, the nurse should dispose of the water in a garbage bag, clean the surrounding skin before the cast dries, and make sure that the cast isn't resting on a hard or sharp surface.

A nurse caring for an infant with congenital heart disease is monitoring the infant closely for signs of congestive heart failure (CHF). The nurse assesses the infant for which early sign of CHF? 1. Pallor 2. Cough 3. Tachycardia 4. Slow and shallow breathing

3. Tachycardia Congestive heart failure (CHF) is the inability of the heart to pump a sufficient amount of oxygen to meet the metabolic needs of the body. The early signs of CHF include tachycardia, tachypnea, profuse scalp sweating, fatigue and irritability, sudden weight gain, and respiratory distress. A cough may occur in CHF as a result of mucosal swelling and irritation, but is not an early sign. Pallor may be noted in an infant with CHF, but is not an early sign.

Which of the following would be the best approach when trying to take a crying toddler's temperature? 1. Ignore the crying and screaming. 2. Encourage the mother to hold the child. 3. Talk to the mother first and then to the toddler. 4. Bring extra help so it can be done quickly.

3. Talk to the mother first and then to the toddler. When dealing with a crying toddler, the best approach is to talk to the mother and ignore the toddler at first. This approach helps the toddler get used to the nurse before she attempts any procedures. It also gives the toddler an opportunity to see that the mother trusts the nurse. Ignoring the crying and screaming may be the second step. Having the mother hold the toddler will help if she can do this. The last resort is to bring in assistance so the procedure can be completed quickly.

A school-age child reveals to the nurse that his father has been abusing him. What constitutes a breech of the child's right to confidentiality? 1. Telling the child you're required by law to report the abuse 2. Informing the child's attending physician about the conversation 3. Telling the client in the next room, who also suffered abuse, so the two children can talk to each other 4. Informing local authorities and reporting the case

3. Telling the client in the next room, who also suffered abuse, so the two children can talk to each other Clients have a right to privacy and confidentiality when it comes to their medical condition, treatment plans, and even the fact that they are hospitalized. Therefore, telling another client about the abuse (even if they have that in common) is a breech of confidentiality. The attending physician is part of the health care team and needs to be informed about the suspected abuse. A nurse is required by law to report suspected child abuse to the proper local authorities.

For a child with tracheobronchitis, the nurse formulates a nursing diagnosis of Ineffective airway clearance related to thick secretions. After implementing interventions, the nurse expects which client outcome? 1. The child exhibits a respiratory rate of 44 breaths/minute. 2. The child exhibits an arterial oxygen saturation of 85%. 3. The child exhibits clear breath sounds. 4. The child exhibits increased anxiety.

3. The child exhibits clear breath sounds. Clear breath sounds indicate an improved respiratory status and airway clearance. A respiratory rate of 44 breaths/minute is high and indicates a respiratory problem. An arterial oxygen saturation of 85% is abnormally low. Decreased, not increased, anxiety would indicate effective airway clearance.

A 10-year-old client must undergo a surgical procedure. Does the nurse need to obtain consent from the client? 1. The child doesn't need to know about the procedure because he is a minor. 2. The child must sign the form giving written informed consent. 3. The child must be informed of the procedure and concur with his mother, who is giving written consent. 4. The child only needs to know if the procedure is part of a research protocol.

3. The child must be informed of the procedure and concur with his mother, who is giving written consent. Assent, not consent, must be obtained from any child who is in the concrete operations thought stage of development (usually a child older than age 7.) Assent involves knowledge of the procedure and agreement with the person authorized to give written informed consent. A child should always be notified of the treatment plan but he is too young to authorize consent. Careful ethical consideration should be given when using any person younger than age 18 in a research protocol.

The nurse realizes she is 1 hour late in administering a dose of medication for her 4-year-old client. She gives the medication immediately, and assesses the client. The client is not harmed by the delay. Which action should the nurse take? 1. No further action is necessary. 2. The nurse should notify the physician of the error. 3. The nurse should follow facility procedures for reporting an error. 4. The nurse should document a medication error in the client's chart.

3. The nurse should follow facility procedures for reporting an error. Although no harm came to the client, this scenario is an example of a medication error. The nurse should follow the facility's procedure for reporting the error. This allows the facility to adequately assess the causes of medication errors, and isn't meant to place blame on any one person. The nurse in this instance doesn't need to notify the physician because there was no harm to the patient. The nurse shouldn't document that an error took place in the client's chart; doing so may place her at risk in the event of a lawsuit.

The parents of a preschool-age child ask the nurse about nutrition. Which statement about a preschooler's nutritional requirements is accurate? 1. Caloric requirements per kilogram of body weight increase slightly during the preschool-age period. 2. The preschooler's nutritional requirements differ greatly from those of a toddler. 3. The quality of food that a preschooler consumes is more important than the quantity. 4. Protein should account for 25% of the preschooler's total caloric intake.

3. The quality of food that a preschooler consumes is more important than the quantity. Food quality is more important than quantity; a high caloric intake may include many empty calories. The preschooler's caloric requirement is slightly lower than the toddler's. Overall, however, the preschooler's nutritional requirements are similar to a toddler's. The preschooler requires 1.5 g/kg of protein daily, satisfied by two meat servings, three milk servings, four bread servings, and four fruit and vegetable servings.

A 7-year-old child is admitted with epiglottitis. Which is the most likely finding on a lateral neck X-ray in a child with epiglottitis? 1. Supraglottic narrowing 2. Steeple sign 3. Thickened mass 4. Subglottic narrowing

3. Thickened mass X-ray assessment of the lateral neck helps diagnose common respiratory emergencies in children. The lateral neck X-ray of a child with epiglottitis shows a thickened mass. The steeple sign is found in the client with viral croup syndrome. Subglottic narrowing with membranous tracheal exudate is found in bacterial tracheitis. Supraglottic narrowing isn't a diagnostic indicator.

A mother tells the nurse that her 22-month-old child says no to everything. When scolded, the toddler gets angry and starts crying loudly but then immediately wants to be held. What is the best interpretation of this behavior? 1. The toddler isn't coping with stress effectively. 2. The toddler's need for affection isn't being met. 3. This behavior is normal in a 2-year-old child. 4. This behavior suggests the need for counseling.

3. This behavior is normal in a 2-year-old child. Toddlers are confronted with the conflict of achieving autonomy yet relinquishing their much-enjoyed dependence on — and affection of — others. Therefore, their negativism is a necessary assertion of self-control. Nothing about this behavior indicates that the child is under stress, isn't receiving sufficient affection, or requires counseling.

The physician prescribes corticosteroids for a child with nephrotic syndrome. What is the primary purpose of administering corticosteroids to this child? 1. To increase blood pressure 2. To reduce inflammation 3. To decrease proteinuria 4. To prevent infection

3. To decrease proteinuria The primary purpose of administering corticosteroids to a child with nephrotic syndrome is to decrease proteinuria. Corticosteroids have no effect on blood pressure. Although they help reduce inflammation, this isn't the reason for their use in clients with nephrotic syndrome. Corticosteroids may predispose a client to infection.

A child with a poor nutritional status and weight loss is at risk for a negative nitrogen balance. To help diagnose this problem, the nurse anticipates that the physician will order which laboratory test? 1. Total iron-binding capacity 2. Hemoglobin (Hb) 3. Total protein 4. Sweat test

3. Total protein A negative nitrogen balance may result from inadequate protein intake and is best detected by measuring the total protein level. Measuring total iron-binding capacity and Hb levels would help detect iron deficiency anemia, not a negative nitrogen balance.The sweat test helps diagnose cystic fibrosis. It doesn't detect a negative nitrogen balance.

When developing a care plan for a child, the nurse identifies which Eriksonian stage as corresponding to Freud's oral stage of psychosexual development? 1. Initiative versus guilt 2. Autonomy versus shame and doubt 3. Trust versus mistrust 4. Industry versus inferiority

3. Trust versus mistrust Freud defined the first 2 years of life as the oral stage and suggested that the mouth is the primary source of satisfaction for the developing child. Erikson posited that infancy (from birth to age 12 months) is the stage of trust versus mistrust, during which the infant learns to deal with the environment through the emergence of trustfulness or mistrust. Initiative versus guilt corresponds to Freud's phallic stage. Autonomy versus shame and doubt corresponds to Freud's anal/sensory stage. Industry versus inferiority corresponds to Freud's latency period.

After a tonsillectomy, a child begins to vomit bright red blood. The initial nursing action is to: 1. Notify the physician. 2. Maintain NPO status. 3. Turn the child to the side. 4. Administer the prescribed antiemetic.

3. Turn the child to the side. After tonsillectomy, if bleeding occurs, the nurse immediately turns the child to the side to prevent aspiration and then notifies the physician. NPO status would be maintained, and an antiemetic may be prescribed; however, the initial nursing action would be to turn the child to the side.

Parents of a 6-year-old tell a physician that the child has been having periods of unawareness with short periods of staring. Based on his history, the child is probably having which type of seizure? 1. Complex partial 2. Myoclonic 3. Typical absence 4. Tonic

3. Typical absence A typical absence seizure has an onset between ages 3 and 12. It's exhibited by an abrupt loss of consciousness, amnesia, or unawareness characterized by staring and a 3-cycle/second spike and waveform on an EEG. The attack lasts from 10 to 30 seconds and may occur as frequently as 50 to 100 times a day. No postictal or confused state follows the attack. A complex partial seizure causes a brief impairment of consciousness. A myoclonic seizure occurs in older children and is exhibited by lightning jerks without loss of consciousness. An abrupt increase in muscle tone, loss of consciousness, and marked autonomic signs and symptoms characterize the tonic seizure.

A child, age 5, is brought to the pediatrician's office for a routine visit. When inspecting the child's mouth, the nurse expects to find how many teeth? 1. Up to 10 2. Up to 15 3. Up to 20 4. Up to 32

3. Up to 20 A child may have up to 20 deciduous teeth by age 5. The first tooth usually erupts by age 6 months; the last, by age 30 months. Deciduous teeth usually are shed between ages 6 and 13.

The nurse should determine a child's body surface area by using: 1. weight. 2. height. 3. a nomogram. 4. the difference between weight and height.

3. a nomogram. The method for determining body surface area is a three-column chart called a nomogram. The nurse marks the child's height in the first column and weight in the third column, then draws a line between the two marks. The point at which the line intersects the vertical scale in the second column indicates the estimated body surface area of the child in square meters. The nurse uses this information in a formula to determine the child's dose of a drug.

While assessing a 2-month-old infant's airway, the nurse finds that he isn't breathing. After two unsuccessful attempts to establish an airway, the nurse should: 1. attempt rescue breaths. 2. attempt to establish an airway a third time. 3. administer five back blows. 4. attempt to ventilate with a handheld resuscitation bag.

3. administer five back blows. The infant's airway is blocked despite attempts to establish it. The next step is to clear the airway with back blows and chest thrusts. Breaths can't be administered until the airway is patent. After two attempts to establish an airway, the nurse can assume the airway is blocked. The nurse can't ventilate the infant with a handheld resuscitation bag until the airway is patent.

A mother brings her 2-month-old infant to the clinic for a well-baby checkup. To best assess the interaction between the mother and infant, the nurse should observe them: 1. as the infant plays. 2. as the infant sleeps. 3. as the mother feeds the infant. 4. as the mother rocks the infant.

3. as the mother feeds the infant. The nurse can best assess mother-infant interaction during feeding, such as by observing how closely the mother holds the infant and how she looks at the infant's face. These behaviors help reveal the mother's anxiety level and overall feelings for the infant. The infant's posture and response during feeding provide clues to the infant's comfort level and feelings. Sleeping doesn't provide an opportunity for mother-infant interaction. Although playing and rocking may provide clues about mother-infant interaction, they aren't the best activities to assess. During playing, for instance, the mother may interact with the infant at a distance. Rocking promotes closeness but not interaction; the mother can rock the infant while talking to someone else or staring off into the distance.

When planning care for a 7-year-old boy with Down syndrome, the nurse should: 1. plan interventions at the developmental level of a 7-year-old because that is the child's age. 2. plan interventions at the developmental level of a 5-year-old because the child will have developmental delays. 3. assess the child's current developmental level and plan care accordingly. 4. direct all teaching to the parents because the child can't understand.

3. assess the child's current developmental level and plan care accordingly. Nursing care should be planned at the developmental age of a child with Down syndrome, not the chronological age. Because children with Down syndrome can vary from mildly to severely mentally challenged, each child should be individually assessed. A child with Down syndrome is capable of learning, especially one with mild limitations. Gear teaching toward the appropriate developmental age.

To decrease the likelihood of bradyarrhythmias in children during endotracheal intubation, succinylcholine (Anectine) is used with which of the following agents? 1. epinephrine (Adrenalin) 2. isoproterenol (Isuprel) 3. atropine 4. lidocaine (Xylocaine)

3. atropine Succinylcholine is an ultra-short-acting depolarizing agent used for rapid-sequence intubation. Bradycardia can occur, especially in children. Atropine is the drug of choice in treating succinylcholine-induced bradycardia. Lidocaine is used in adults only. Epinephrine bolus and isoproterenol aren't used in rapid-sequence intubation because of their profound cardiac effects.

The nurse is teaching the parents of a young child how to handle suspected poisoning. If the child ingests poison, the parents should first: 1. administer ipecac syrup. 2. call an ambulance. 3. call the poison control center. 4. punish the child for being bad.

3. call the poison control center. Before intervening in any way, the parents should call the poison control center for specific instructions. Ipecac syrup is no longer recommended for the ingestion of poisons. The parents may have to call an ambulance after calling the poison control center. Punishment for being bad isn't appropriate because the parents are responsible for making the environment safe.

An 8-month-old is admitted to the pediatric unit following a fall from his high chair. The child is awake, alert, and crying. The nurse should know that a brain injury is more severe in children because of: 1. increased myelination. 2. intracranial hypotension. 3. cerebral hyperemia. 4. a slightly thicker cranium.

3. cerebral hyperemia. Cerebral hyperemia (excess blood in the brain) causes an initial increase in intracranial pressure in the head of an injured child. The brain is less myelinated in a child and more easily injured than an adult brain. Intracranial hypertension — not hypotension — places the child at greater risk for secondary brain injury. A child's cranium is thinner and more pliable, causing the child to receive a more severe injury.

A child, age 5, has acute lymphoblastic leukemia (ALL) and is to receive induction chemotherapy consisting of vincristine (Oncovin), asparaginase (L-asparaginase [Elspar]), and prednisone (Deltasone). When teaching the parents about the adverse effects of this regimen, the nurse should stress the importance of promptly reporting: 1. hair loss. 2. moon face. 3. constipation. 4. bone pain.

3. constipation. Neurotoxicity, the primary adverse effect of vincristine, may manifest as constipation — a sign of paralytic ileus that the parents must report promptly. (Neurotoxicity also may cause peripheral neuropathy.) Hair loss and moon face are expected adverse effects of this chemotherapy regimen and will resolve once therapy ends. Bone pain is common in clients with ALL and results from invasion of the periosteum by leukemic cells.

The nurse is caring for a teenage client involved in a motor vehicle accident. The client has a chest tube in place. If the chest tube is accidentally removed, the nurse should immediately: 1. reintroduce the tube and attach it to water seal drainage. 2. call the physician and obtain a chest tray. 3. cover the opening with petroleum gauze. 4. clean the wound with povidone-iodine and apply a gauze dressing.

3. cover the opening with petroleum gauze. If a chest tube is accidentally removed, the nurse should cover the insertion site with sterile petroleum gauze. The nurse should then observe the client for respiratory distress, as tension pneumothorax may develop. If so, the nurse should remove the gauze to allow air to escape. The nurse shouldn't reintroduce the tube. Rather, the nurse should have another staff member call a physician so another tube can be introduced by the physician under sterile conditions.

A nurse is administering a medication by intraosseous infusion to a child. Intraosseous drug administration is typically used when a child is: 1. younger than age 3. 2. older than age 3. 3. critically ill and younger than age 3. 4. critically ill and older than age 3.

3. critically ill and younger than age 3. In an emergency, intraosseous drug administration is typically used when a child is critically ill and younger than age 3.

A 14-month-old child with acquired immunodeficiency syndrome (AIDS) is admitted to the facility with an infection. When developing a care plan, the nurse must keep in mind that AIDS in children commonly is associated with: 1. Kaposi's sarcoma. 2. congenital heart anomalies. 3. developmental delays. 4. Wiskott-Aldrich syndrome.

3. developmental delays. Children with AIDS commonly exhibit developmental delays or regression. To plan developmentally appropriate care and establish realistic goals, the nurse must obtain information about the child's developmental status. Unlike adults with AIDS, children with this disease rarely develop Kaposi's sarcoma. AIDS isn't associated with congenital heart anomalies. Clinical manifestations of Wiskott-Aldrich syndrome, an X-linked recessive disorder characterized by immunodeficiency, resemble those of AIDS; however, the two syndromes aren't related.

The development of disaster plans should take into consideration that children are more susceptible to the effects of a chemical attack than adults because children: 1. have smaller body surface areas than adults. 2. breathe at a slower rate than adults. 3. have thinner skin than adults. 4. have a low risk of developing rapid dehydration.

3. have thinner skin than adults. Because of anatomical and physiological differences, children are more susceptible to the effects of chemical and biological attacks. Children have thinner skin than adults, increasing their risk of absorbing a chemical. They also have a larger, not smaller, body surface area in relation to their weight than do adults, which increases the chance of chemical absorption. Children breathe at a faster, not slower, rate than adults, allowing them to inhale greater amounts of a toxic agent. Additionally, some chemical agents are heavier than air and accumulate close to the ground, which is closer to a child's breathing zone than an adult's. Because they have less fluid reserve than adults, children are at greater risk of developing rapid dehydration from agents that cause vomiting or diarrhea.

The nurse is caring for a toddler with Down syndrome. To help the toddler cope with painful procedures, the nurse can: 1. prepare the child by positive self-talk. 2. establish a time limit to get ready for the procedure. 3. hold and rock him and give him a security object. 4. count and sing with the child.

3. hold and rock him and give him a security object. The child with Down syndrome may have difficulty coping with painful procedures and may regress during his illness. Holding, rocking, and giving the child a security object may be comforting to the child. An older child or a child without Down syndrome may benefit from positive self-talk, time limits, and diversionary tactics, such as counting and singing; however, the success of these tactics depends on the child.

A 3-year-old child with Down syndrome is admitted to the pediatric unit with asthma. The child doesn't enunciate words well and holds onto furniture when he walks. The nurse should ask the mother: 1. how long the child has been like this. 2. if the child is able to walk without holding onto furniture. 3. how the child's condition today differs from his normal condition. 4. if the child always drools.

3. how the child's condition today differs from his normal condition. Identify the chief complaint by asking how the child was previously behaving at home. Asking how long the child has been like this may be interpreted poorly by the caregiver. Focus on what the child can do — not on what he can't do — to preserve the family's self-esteem. Focusing on negative aspects of the child's behavior is inappropriate.

A child with a full-thickness burn is scheduled for debridement using hydrotherapy. Before hydrotherapy begins, the nurse should: 1. administer fluids as prescribed. 2. administer antibiotics as prescribed. 3. implement pain control measures. 4. provide nutritional supplements.

3. implement pain control measures. Because hydrotherapy is painful, the nurse should implement pain control measures before this treatment begins. Fluids and nutritional supplements can be given at any time and aren't required specifically before hydrotherapy. Antibiotics should be administered according to a specified schedule without regard to any treatment.

The parents of a child, age 6, who will begin school in the fall ask the nurse for anticipatory guidance. The nurse should explain that a child of this age: 1. still depends on the parents. 2. rebels against scheduled activities. 3. is highly sensitive to criticism. 4. loves to tattle.

3. is highly sensitive to criticism. In a 6-year-old child, a precarious sense of self causes overreaction to criticism and a sense of inferiority. By age 6, most children no longer depend on the parents for daily tasks and love the routine of a schedule. Tattling is more common at age 4 or 5; by age 6, the child wants to make friends and be a friend.

A child, age 3, is admitted to the hospital for surgery. The physician orders meperidine (Demerol), 25 mg I.M., to be administered at 7 a.m. on the morning of surgery. To carry out this order, the nurse should: 1. administer the medication before the child awakens. 2. ask the child whether she wants the injection now. 3. let the child choose which leg will receive the injection. 4. state that the nurse will return in a few minutes to give the injection.

3. let the child choose which leg will receive the injection. Preschoolers are developing initiative and benefit from participating in their care, such as by choosing which leg will receive the injection. The nurse always should awaken a child before giving an injection; otherwise, the child may become afraid to fall asleep for fear that something traumatic may occur. The nurse shouldn't give the child a choice when one doesn't exist; the injection will be given at the proper time regardless of when the child wants it. Young children have a limited concept of time; they cope best with injections and other threatening events if they receive a brief explanation and then undergo the event without delay. A delay between the explanation and event gives the child time to fantasize about what will happen, which may increase anxiety.

A child, age 5, is to receive potassium added to the I.V. fluid. Before initiating this therapy, the nurse first should: 1. assess the child's apical pulse rate. 2. measure the blood pressure. 3. monitor fluid intake and output. 4. assess respiratory rate and depth.

3. monitor fluid intake and output. Potassium shouldn't be added to the I.V. fluid until the child regains adequate kidney function, as indicated by balanced fluid intake and output and certain diagnostic test results. The other options aren't related to potassium administration.

Most oral pediatric medications are administered: 1. with the nighttime formula. 2. ½ hour after meals. 3. on an empty stomach. 4. with meals.

3. on an empty stomach. Most oral pediatric medications are administered on an empty stomach. They aren't usually administered with milk or formula because these can affect gastric pH and alter drug absorption. Because a child's meals usually contain milk or a milk product, the nurse wouldn't administer the drugs with meals or even ½ hour after meals.

The nurse is providing dietary teaching for the parents of a child with celiac disease. This child should avoid: 1. vegetables. 2. fruits. 3. prepared puddings. 4. rice.

3. prepared puddings. A child with celiac disease mustn't consume foods containing gluten and therefore should avoid prepared puddings, commercially prepared ice cream, malted milk, and all food and beverages containing wheat, rye, oats, or barley. The other options don't contain gluten and are permitted on a gluten-free diet.

A 4-year-old child is having a sickle cell crisis. The initial nursing intervention should be to: 1. place ice packs on the client's painful joints. 2. administer antibiotics. 3. provide oral and I.V. fluids. 4. administer folic acid supplements.

3. provide oral and I.V. fluids. Priority care for the child in a sickle cell crisis includes providing hydration and oxygenation to prevent more sickling. Pain relief is also a concern. However, painful joints are treated with analgesics and warm packs because cold packs may increase sickling. Antibiotics will be given to treat a sickle cell crisis if it's thought to be bacterial. Daily supplements of folic acid will help counteract anemia.

The nurse prepares to administer the Denver Developmental Screening Test to a 6-month-old infant during a well-baby checkup. This test assesses the child's: 1. intelligence quotient (IQ). 2. psychological development. 3. social, motor, and language development. 4. vision and eye muscle coordination.

3. social, motor, and language development. The Denver Developmental Screening Test evaluates the developmental level of social, motor, and language skills in children ages 1 month to 6 years. It doesn't assess the child's IQ, psychological development, vision, or eye muscle coordination.

The nurse is planning a health teaching session for a group of parents with toddlers. When describing a toddler's typical eating pattern, the nurse should mention that many children of this age exhibit: 1. consistent table manners. 2. an increased appetite. 3. strong food preferences. 4. a preference for eating alone.

3. strong food preferences. A toddler typically exhibits strong food preferences, eating one type of food for several days and excluding others. A toddler can't be expected to use consistent table manners. Generally, the appetite decreases during the toddler stage because of a slowed growth rate. A toddler typically enjoys socializing during meals and often imitates others.

The nurse is teaching parents how to reduce the spread of impetigo. The nurse should encourage parents to: 1. teach children to cover mouths and noses when they sneeze. 2. have their children immunized against impetigo. 3. teach children the importance of proper hand washing. 4. isolate the child with impetigo from other members of the family.

3. teach children the importance of proper hand washing. The spread of childhood infections, including impetigo, can be reduced when children are taught proper hand-washing technique. Because impetigo is spread through direct contact, covering the mouth and nose when sneezing won't prevent its spread. Currently, there is no vaccine to prevent a child from contracting impetigo. Isolating the child with impetigo is unnecessary.

The physician orders an antibiotic for a child, age 6, who has an upper respiratory tract infection. To avoid tooth discoloration, the nurse expects the physician to avoid prescribing which drug? 1. penicillin 2. erythromycin 3. tetracycline 4. amoxicillin

3. tetracycline Tetracycline should be avoided in children younger than age 8 because it may cause enamel hypoplasia and permanent yellowish gray to brownish tooth discoloration. Penicillin, erythromycin, and amoxicillin don't discolor the teeth.

The nurse observes a play group of 2-year-old children. The nurse would expect to see: 1. four children playing dodgeball. 2. three children playing tag. 3. two children side by side in the sandbox building sand castles. 4. one child digging a hole and another child blowing bubbles.

3. two children side by side in the sandbox building sand castles. Two-year-olds exhibit parallel play; that is, they engage in similar activity, side by side. Playing dodgeball and tag are examples of interactive play, common to school-age children. A 2-year-old wouldn't blow bubbles.

A child, age 14, is diagnosed with scoliosis and scheduled for brace application. The mother asks the nurse how long her child will have to wear the brace. What is the nurse's best response? 1. "About 6 to 8 weeks." 2. "About 6 months." 3. "About 2 to 4 years." 4. "About 4 to 7 years."

4. "About 4 to 7 years." Most children with scoliosis must wear a brace until the spine matures — typically between ages 18 and 21. Therefore, this 14-year-old child will need to wear the brace for 4 to 7 years.

A mother complains to the nurse that her 4-year-old son often "lies." What is the nurse's best response? 1. "Let the child know that he'll be punished for lying." 2. "Ask him why he isn't telling the truth." 3. "It's probably due to his vivid imagination and creativity." 4. "Acknowledge him by saying, 'That's a pretend story.'"

4. "Acknowledge him by saying, 'That's a pretend story.'" It's important to acknowledge the child's imagination, while also letting him know in a nice way that what he has said isn't real. Punishment isn't appropriate for a 4-year-old child using his imagination, and accusing him of lying is a negative reinforcement. The child isn't truly lying in the adult sense. Imagination and creativity need to be acknowledged.

The parents of a 9-year-old child in the terminal phase of a fatal illness ask the nurse for guidance in discussing death with their child. Which response is appropriate? 1. "Children of that age view death as temporary and reversible, which makes it hard to explain." 2. "Children of that age typically fantasize about what dying will be like, which is much better than knowing the truth." 3. "At this developmental stage, children are afraid of death, so it's best not to discuss it with them." 4. "At this developmental stage, most children have an adult concept of death and should be encouraged to discuss it."

4. "At this developmental stage, most children have an adult concept of death and should be encouraged to discuss it." By age 9 or 10, most children have an adult concept of death. Caregivers should discuss death with them in terms consistent with their developmental stage. School-age children respond well to concrete explanations about death and dying. Preschoolers, not school-age children, typically view death as temporary and reversible. School-age children may fantasize about the unknown aspects of death; these fantasies may increase their anxiety. Although a child may fear death, accurate information about death can ease anxiety.

A 15-year-old girl visits the neighborhood clinic seeking information on "how to keep from getting pregnant." What should the nurse say in response to her request? 1. "What would you like to know?" 2. "Let's discuss what your friends are doing to keep from getting pregnant." 3. "Can you tell me if you've told your parents you're having sex?" 4. "Can you tell me about the precautions you're taking now?"

4. "Can you tell me about the precautions you're taking now?" An attitude that requests only the information the girl is willing to give is nonthreatening and nonjudgmental. This may enhance the girl's willingness to talk about her experiences, thus enabling the nurse to better assess her needs. The first response assumes the girl knows what she needs to know. The precautions her friends are taking are irrelevant at this time. Reference to the girl's parents may make her defensive and fearful of help.

A nurse receives a telephone call from the admitting office and is told that a child with rheumatic fever will be arriving in the nursing unit for admission. On admission, the nurse prepares to ask the mother which question to elicit assessment information specific to the development of rheumatic fever? 1. "Has the child complained of back pain?" 2. "Has the child complained of headaches?" 3. "Has the child had any nausea or vomiting?" 4. "Did the child have a sore throat or fever within the last 2 months?"

4. "Did the child have a sore throat or fever within the last 2 months?" Rheumatic fever is an inflammatory autoimmune disease that affects the connective tissues of the heart, joints, subcutaneous tissues, and blood vessels of the central nervous system. Rheumatic fever characteristically manifests 2 to 6 weeks after an untreated or partially treated group A beta-hemolytic streptococcal infection of the upper respiratory tract. Initially, the nurse determines whether the child had a sore throat or an unexplained fever within the past 2 months. Options 1, 2, and 3 are unrelated to rheumatic fever.

A 10-year-old girl visits the clinic for a checkup before entering school. The child's mother questions the nurse about what to expect of her daughter's growth and development at this stage. Which response is most appropriate? 1. "Her physical development will be rapid at this stage, and rapid development will continue from now on." 2. "She'll become more independent and won't require parental supervision." 3. "Don't anticipate any changes at this stage in her growth and development." 4. "Friends will be very important to her, and she'll develop an interest in the opposite sex."

4. "Friends will be very important to her, and she'll develop an interest in the opposite sex." Friends become very important at this age. Children usually begin having an interest in the opposite sex around this age, although they aren't always willing to admit it. Her physical development towards maturity continues, but it isn't as rapid at this stage as in previous years. Although independence increases at this stage, children continue to need parental supervision. Growth and development slow down but gradual changes continue to occur.

A child, age 6, is brought to the health clinic for a routine checkup. To assess the child's vision, the nurse should ask: 1. "Do you have any problems seeing different colors?" 2. "Do you have trouble seeing at night?" 3. "Do you have problems with glare?" 4. "How are you doing in school?"

4. "How are you doing in school?" A child's poor progress in school may indicate a visual disturbance. The other options are more appropriate questions to ask when assessing vision in a geriatric client.

A boy, age 3, develops a fever and rash and is diagnosed with rubella. His mother has just given birth to a baby girl. Which statement by the mother best indicates that she understands the implications of rubella? 1. "I told my husband to give my son aspirin for his fever." 2. "I'll ask the physician about giving the baby an immunization shot." 3. "I don't have to worry because I've had the measles." 4. "I'll call my neighbor who's 2 months pregnant and tell her not to have contact with my son."

4. "I'll call my neighbor who's 2 months pregnant and tell her not to have contact with my son." Fetal defects can occur during the first trimester of pregnancy if the pregnant woman contracts rubella. Aspirin shouldn't be given to young children because aspirin has been implicated in the development of Reye's syndrome. Tylenol should be used instead of aspirin. Rubella immunization isn't recommended for children until ages 12 to 15 months. Measles is rubeola and won't provide immunity for rubella.

A nurse provides home care instructions to the parents of a child with congestive heart failure regarding the procedure for administration of digoxin (Lanoxin). Which statement made by the parent indicates the need for further instructions? 1. "I will not mix the medication with food." 2. "If more than one dose is missed, I will call the physician." 3. "I will take the child's pulse before administering the medication." 4. "If the child vomits after medication administration, I will repeat the dose."

4. "If the child vomits after medication administration, I will repeat the dose." Digoxin is a cardiac glycoside. The parents need to be instructed that if the child vomits after digoxin is administered, they are not to repeat the dose. Options 1, 2, and 3 are accurate instructions regarding the administration of this medication. In addition, the parents should be instructed that if a dose is missed and is not identified until 4 hours later, the dose should not be administered.

A nurse is teaching the parents of an infant with clubfeet about cast care. Which statement by the father indicates the need for further teaching? 1. "I hope this cast will cure his feet in the next several weeks." 2. "I know I will have to be careful when changing his diapers." 3. "We will have to be careful how we hold our baby." 4. "Immunizations will have to be delayed until the casts come off."

4. "Immunizations will have to be delayed until the casts come off." The father's statement about delaying immunizations indicates the need for further teaching. Immunizations can be administered in the thighs because the casts cover only the lower legs and feet. The other responses are correct statements.

A toddler is in the hospital. Which of the following would be the most appropriate response to the parents, who are concerned about the seriousness of the child's illness? 1. "Please try not to worry. Your child will be fine." 2. "If you look around, you'll see other children who are much sicker." 3. "What seems to concern you about your child being hospitalized?" 4. "It must be difficult for you when your child is ill and hospitalized."

4. "It must be difficult for you when your child is ill and hospitalized." Expressing concern about the parents' feelings is the most appropriate response. False reassurance, such as telling parents not to worry, isn't helpful. Encouraging parents to look at how ill other children are also isn't helpful because the focus of the parents is on their own child. Asking what the concern is reinforces the parents' concern without addressing it.

During chemotherapy, a boy, age 10, loses his appetite. When teaching the parents about his food intake, the nurse should include which instruction? 1. "Offer dry toast and crackers." 2. "Withhold all food and fluids." 3. "Ignore your child's lack of food intake." 4. "Let your child eat any food he wants."

4. "Let your child eat any food he wants." The nurse should instruct the parents to let the child eat any food he wants because any form of intake is better than none. Dry crackers would be appropriate for a child experiencing nausea. Withholding all foods and fluids or ignoring lack of food intake would be inappropriate.

The nurse in a clinic walks into a client's room and finds the mother of a 15-month-old in tears. The mother states that her child doesn't love her because the child says "no" to everything. Which of the following would be an appropriate response? 1. "Have you punished your child for saying 'no' to you?" 2. "This is normal at this age; it's best to ignore the behavior." 3. "Explain to your child that saying 'no' all of the time is inappropriate behavior." 4. "Saying 'no' is part of toddler development and is normal at this age."

4. "Saying 'no' is part of toddler development and is normal at this age." Saying "no" is normal at this age. It doesn't mean that the child doesn't love the mother. The child is attempting to exert independence. Punishing the child isn't appropriate because this is a normal stage of development. Ignoring the behavior is also inappropriate because the child needs to learn about limits. Children at this age may not understand all that they say because they repeat what they hear. Explaining to the child that his behavior is inappropriate isn't an age-appropriate response for this child.

During a visit to the well-baby clinic, a mother who's breast-feeding her 2-month-old infant expresses concern over the infant's bowel movements. Which statement by the mother would lead the nurse to believe that the infant's bowel movements are normal? 1. "The baby's stools are yellow and semiformed." 2. "The baby's stools are dark green and sticky." 3. "The baby's stools are green and watery." 4. "The baby's stools are bright yellow and soft."

4. "The baby's stools are bright yellow and soft." Breast-fed infants typically have soft, bright yellow or light green stools with no offensive odor. Formula-fed infants typically have pale yellow, semiformed stools with a strong odor. A neonate's first stools typically are dark green to black, sticky, and odorless (representing meconium, usually present for the first 3 days). By the 4th day, yellowish green transitional stools appear. Green, watery stools indicate diarrhea.

A clinic nurse is providing instructions to a mother of a child with cystic fibrosis regarding the immunization schedule for the child. Which statement would the nurse make to the mother? 1. "The immunization schedule will need to be altered." 2. "The child should not receive any hepatitis vaccines." 3. "The child will receive all the immunizations except for the polio series." 4. "The child will receive the recommended basic series of immunizations along with a yearly influenza vaccination."

4. "The child will receive the recommended basic series of immunizations along with a yearly influenza vaccination." Cystic fibrosis is a chronic multisystem disorder (autosomal recessive trait disorder) characterized by exocrine gland dysfunction. The mucus produced by the exocrine glands is abnormally thick, tenacious, and copious, causing obstruction of the small passageways of the affected organs, particularly in the respiratory, gastrointestinal, and reproductive systems. Adequately protecting children with cystic fibrosis from communicable diseases by immunization is essential. In addition to the basic series of immunizations, a yearly influenza immunization is recommended for children with cystic fibrosis. Options 1, 2, and 3 are incorrect.

The nurse is preparing a child, age 4, for cardiac catheterization. Which explanation of the procedure is most appropriate? 1. "Don't worry. It won't hurt." 2. "The test usually takes an hour." 3. "You must sleep the whole time that the test is being done." 4. "The special medicine will feel warm when it's put in the tubing."

4. "The special medicine will feel warm when it's put in the tubing." To prepare a 4-year-old child without increasing anxiety, the nurse should provide concrete information in small amounts about nonthreatening aspects of the procedure. Saying that it won't hurt may prevent the child from trusting the nurse in the future. Explaining the time needed for the procedure wouldn't provide sufficient information. Stating that the child will need to sleep could provoke anxiety; also, it's untrue.

When teaching the parents of a toddler with congenital heart disease, the nurse should explain all medical treatments and emphasize which instruction? 1. "Reduce your child's caloric intake to decrease cardiac demand." 2. "Relax discipline and limit-setting to prevent crying." 3. "Make sure your child avoids contact with small children to reduce overstimulation." 4. "Try to maintain your child's usual lifestyle to promote normal development."

4. "Try to maintain your child's usual lifestyle to promote normal development." Parents of a child with a congenital heart defect should treat the child normally and allow self-limited activity. Reducing the child's caloric intake wouldn't necessarily reduce cardiac demand. Altering disciplinary patterns and deliberately preventing crying or interactions with other children could foster maladaptive behaviors. Contact with peers promotes normal growth and development.

The nurse is teaching parents how to select appropriate toys for their 10-month-old infant. Which statement by the parents indicates effective teaching? 1. "We'll get a mobile to place over the baby's crib." 2. "We'll get a rattle for the baby to play with." 3. "We'll get the baby some brightly colored blocks." 4. "We'll get the baby a push toy."

4. "We'll get the baby a push toy." At age 10 months, a push toy promotes development of an infant's gross and fine motor skills and aids cognitive development. A mobile provides appropriate visual stimulation for an infant up to age 4 months; after this age, a mobile may pose a danger to an infant. Rattles and brightly colored blocks promote gross and fine motor abilities in infants ages 4 to 8 months.

Parents report that their daughter, age 4, resists going to bed at night. After instruction by the nurse, which statement by the parents indicates effective teaching? 1. "We'll let her fall asleep in our room, then move her to her own room." 2. "We'll lock her in her room if she gets up more than once." 3. "We'll play running games with her before bedtime to tire her out, and then she'll fall asleep easily." 4. "We'll read her a story and let her play quietly in her bed until she falls asleep."

4. "We'll read her a story and let her play quietly in her bed until she falls asleep." Spending time with the parents and playing quietly are positive bedtime routines that provide security and prepare a child for sleep. The child should sleep in her own bed. Locking the door is frightening and may cause insecurity. Active play before bedtime stimulates the child and increases the time needed to settle down for sleep.

A child, age 3, who tests positive for the human immunodeficiency virus (HIV) is placed in foster care. The foster parents ask the nurse how to prevent HIV transmission to other family members. How should the nurse respond? 1. "Make sure the child uses disposable plates and utensils." 2. "Use isopropyl alcohol to clean surfaces contaminated with the child's blood or body fluids." 3. "Don't let the child share toys with other children." 4. "Wear gloves when you're likely to come into contact with the child's blood or body fluids."

4. "Wear gloves when you're likely to come into contact with the child's blood or body fluids." HIV is transmitted by blood and body fluids. Therefore, family members should wear gloves when anticipating contact with the child's blood or body fluids. To disinfect HIV-contaminated surfaces, the nurse should instruct the foster parents to use a solution of 1 part bleach to 10 parts water. Standard household methods for cleaning dishes and utensils are adequate, so the child needn't use disposable plates and utensils. The child may share toys; any toys that become soiled with the child's blood or body fluids should be disinfected with the bleach solution.

The mother of a hospitalized infant appears anxious and displays anger with the staff. Which response is most appropriate? 1. "Some of the staff members don't want to talk to you because you might yell at them." 2. "Why do you seem so angry today? It makes it hard for us to help you." 3. "Is this your normal behavior or are you acting out because your child is hospitalized?" 4. "You seem upset. Having your child hospitalized must be difficult."

4. "You seem upset. Having your child hospitalized must be difficult." Acknowledging the mother's feelings and recognizing that it's difficult to cope with a hospitalized child allows the mother to express her feelings. Telling the mother that other staff members don't want to talk to her isn't therapeutic. Asking her to explain her behavior places the mother on the defensive and isn't therapeutic.

The nurse is approached by the mother of a child with hypospadias. She says to the nurse, "Why did this have to happen to my baby? Why couldn't he be perfect? How could this have happened?" What should the nurse say in response? 1. "This is only a minor problem. Many other babies are born with worse defects." 2. "Don't worry. After surgical repair you'll hardly remember there was anything wrong with your baby." 3. "I'll ask the physician to explain to you how this defect occurs." 4. "You seem upset. Tell me about it."

4. "You seem upset. Tell me about it." By verbalizing observations of the client's behavior, the nurse acknowledges the client's feelings. By listening, the nurse can help the client understand her feelings and begin to deal with them. Telling the client that there are babies with worse defects doesn't acknowledge — and may even belittle — her feelings. Providing a stock answer, such as "Don't worry," shows a lack of interest in the client's feelings. Offering to ask the physician also doesn't address the client's feelings.

A mother is playing with her infant, who's sitting securely alone on the floor of the clinic. The mother hides a toy behind her back and the infant looks for it. What age would the nurse estimate the infant to be? 1. 6 months 2. 4 months 3. 8 months 4. 10 months

4. 10 months A 10-month-old child can sit alone and understands object permanence, so he would look for the hidden toy. Between ages 4 and 6 months, children can't sit securely alone. At age 8 months, children can sit securely alone but don't understand the permanence of objects.

The nurse in a well-child clinic is assessing children for scoliosis. Which of the following children is most at risk for scoliosis? 1. 8-year-old boy 2. Teenage boy 3. 6-year-old girl 4. 10-year-old girl

4. 10-year-old girl The 10-year-old girl is most likely to have scoliosis. Scoliosis is five times more common in girls than boys, and its peak age of incidence is between ages 8 and 15. The 8-year-old boy or a teenage boy may develop scoliosis but it's more common in females. A 6-year-old girl is typically too young to be diagnosed with scoliosis.

Which assessment would alert the nurse that a hospitalized 7-year-old child is at high risk for a severe asthma exacerbation? 1. Oxygen saturation of 95% 2. Mild work of breath 3. Intercostal or substernal retractions 4. A history of steroid-dependent asthma

4. A history of steroid-dependent asthma A history of steroid-dependent asthma, a contributing factor to making this a high-risk client, requires the nurse to treat the situation as a severe exacerbation regardless of the severity of the current episode. Decreased oxygen saturation, cyanosis, and retractions are all assessments of an asthma exacerbation and should be treated with oxygen, nebulized respiratory treatments, and steroids. However, if a significant history of high-risk factors is absent, the episode can be moderately treated and followed up with the pediatrician.

When planning to administer medication to a 3-month-old infant, the nurse should keep which consideration in mind? 1. An infant's metabolic rate is slower than an adult's. 2. An infant's liver detoxifies drugs faster than an adult's. 3. An infant's systemic drug circulation is slower than an adult's. 4. An infant's kidneys excrete drugs more slowly than an adult's.

4. An infant's kidneys excrete drugs more slowly than an adult's. Most drugs are excreted by the kidneys. Because an infant has immature kidney function, drugs are excreted more slowly, significantly altering drug effects. An infant has a faster metabolic rate, slower drug detoxification, and faster systemic drug circulation than an adult.

When administering morphine, the drug of choice for pediatric pain relief, to a school-age child, which symptom should cause the nurse to be concerned? 1. Constipation 2. Nausea and vomiting 3. Pruritus 4. Anemia

4. Anemia Anemia isn't a typical adverse effect of morphine and should be investigated if it's discovered during treatment. Constipation, nausea and vomiting, and pruritus are all treatable adverse effects of morphine and don't necessitate cessation of the medication.

A nurse is caring for a child with a suspected diagnosis of rheumatic fever. The nurse reviews the laboratory results, knowing that which laboratory study would assist in confirming the diagnosis? 1. Immunoglobulin 2. Red blood cell count 3. White blood cell count 4. Antistreptolysin O titer

4. Antistreptolysin O titer Rheumatic fever is an inflammatory autoimmune disease that affects the connective tissues of the heart, joints, subcutaneous tissues, and blood vessels of the central nervous system. A diagnosis of rheumatic fever is confirmed by the presence of two major manifestations or one major and two minor manifestations from the Jones criteria. In addition, evidence of a recent streptococcal infection is confirmed by a positive antistreptolysin O titer, Streptozyme assay, or anti-DNase B assay. Options 1, 2, and 3 would not help to confirm the diagnosis of rheumatic fever.

A new mother expresses concern to a nurse regarding sudden infant death syndrome (SIDS). She asks the nurse how to position her new infant for sleep. The nurse appropriately tells the mother that the infant should be placed on the: 1. Side or prone 2. Back or prone 3. Stomach with the face turned 4. Back rather than on the stomach

4. Back rather than on the stomach Sudden infant death syndrome (SIDS) is the unexpected death of an apparently healthy infant younger than 1 year for whom an investigation of the death and a thorough autopsy fails to show an adequate cause of death. Several theories are proposed regarding the cause, but the exact cause is unknown. Nurses should encourage parents to place the infant on the back (supine) for sleep. Infants in the prone position (on the stomach) may be unable to move their heads to the side, increasing the risk of suffocation. The infant may have the ability to turn to a prone position from the side-lying position.

When teaching the mother of a 17-month-old about toilet training, which instruction would initially be most appropriate? 1. Place the toddler on the potty chair every 2 hours for 10 minutes. 2. Offer a reward every time the child has a bowel movement in the potty chair. 3. Remove the diaper and use training pants to begin the process. 4. Be sure the child is ready before starting to toilet train.

4. Be sure the child is ready before starting to toilet train. All of the instructions are appropriate, but knowing whether the child is ready to toilet train is initially most appropriate. Many 17-month-olds don't have the neuromuscular control to be able to be trained. Waiting a few more months until the child is closer to age 2 years allows the child to develop more control. The mother should be taught the signs of readiness for toilet training.

The nurse is developing a teaching plan for a child with acute poststreptococcal glomerulonephritis. What is the most important point to address in this plan? 1. Infection control 2. Nutritional planning 3. Prevention of streptococcal pharyngitis 4. Blood pressure monitoring

4. Blood pressure monitoring Because poststreptococcal glomerulonephritis may cause severe, life-threatening hypertension, the nurse must teach the parents how to monitor the child's blood pressure. Infection control, nutritional planning, and prevention of streptococcal pharyngitis are important but are secondary to blood pressure monitoring.

When assessing a neonate, the nurse observes a vaguely outlined area of scalp edema. Which term should the nurse use when documenting this observation? 1. Cephalhematoma 2. Petechiae 3. Subdural hematoma 4. Caput succedaneum

4. Caput succedaneum Caput succedaneum refers to a vaguely outlined area of scalp edema that crosses the suture line and typically clears within a few days after birth. Cephalhematoma is a swelling of the head that results from subcutaneous bleeding caused by pressure exerted on the soft tissues during delivery; it's characterized by sharply demarcated boundaries that don't cross the suture lines. Petechiae are minute, circumscribed, hemorrhagic areas of the skin. A subdural hematoma is a collection of blood between the dura and the brain.

Laboratory results for a child with a congenital heart defect with decreased pulmonary blood flow reveal an elevated hemoglobin (Hb) level, hematocrit (HCT), and red blood cell (RBC) count. These data suggest which condition? 1. Anemia 2. Dehydration 3. Jaundice 4. Compensation for hypoxia

4. Compensation for hypoxia A congenital heart defect with decreased pulmonary blood flow alters blood flow through the heart and lungs, resulting in hypoxia. To compensate, the body increases the oxygen-carrying capacity of RBCs by increasing RBC production, which causes the Hb level and HCT to rise. In anemia, the Hb level and HCT typically decrease. Altered electrolyte levels and other laboratory values are better indicators of dehydration. An elevated Hb level and HCT aren't associated with jaundice.

A nurse is preparing to care for a 5-year-old who has been placed in traction following a fracture of the femur. The nurse plans care, knowing that which of the following is the most appropriate activity for this child? 1. A radio 2. A sports video 3. Large picture books 4. Crayons and a coloring book

4. Crayons and a coloring book In the preschooler, play is simple and imaginative, and includes activities such as crayons and coloring books, puppets, felt and magnetic boards, and Play-Doh. A radio or sports video are most appropriate for the adolescent . Large picture books are most appropriate for the infant.

A 3-year-old Vietnamese child with a fever, decreased urine output, wheezing, and coughing is brought to the emergency department. On examination, the nurse discovers red, round, weltlike lesions on the child's upper back and chest. The nurse would interpret these lesions to be caused by which of the following? 1. Shingles 2. Child abuse 3. Allergic reaction 4. Cultural practice

4. Cultural practice Many Vietnamese perform coining, a cultural practice in which a coin is repeatedly rubbed lengthwise on the oiled skin to rid the body of a disease. Coining can produce weltlike lesions on the child's back or chest, and children subjected to the practice are often thought to have been abused. Interviewing the family and assessing its cultural background help distinguish between abuse and culture practice. Shingles, a form of herpes zoster, is a communicable disease usually affecting immunocompromised individuals and older adults. The disease produces small crusty pustules on the lower back and trunk. The description of the lesions doesn't fit those produced by an allergic reaction.

A child, age 5, is diagnosed with hyperphosphatemia secondary to chronic renal failure. When teaching the parents about diet therapy, the nurse should instruct them to eliminate which foods from the child's diet? 1. Meats 2. Carbohydrates 3. Fats 4. Dairy products

4. Dairy products Dairy products contain a significant amount of phosphorus and should be eliminated from the diet of a child with hyperphosphatemia to prevent this condition from worsening. Meats, carbohydrates, and fats are appropriate food choices for this child.

A 10-month-old infant is admitted to the facility with dehydration and metabolic acidosis. What is the most common cause of dehydration and acidosis in infants? 1. Early introduction of solid foods 2. Inadequate perianal hygiene 3. Tachypnea 4. Diarrhea

4. Diarrhea Diarrhea is the most common cause of dehydration and acidosis in infants. Early introduction of solid foods may cause loose stools but not dehydration or acidosis. Poor perianal hygiene may cause diaper dermatitis. Tachypnea is a sign — not a cause — of acidosis.

The nurse is assessing whether the client has received all recommended immunizations for his age. Which immunizations should he have received between ages 4 and 6? 1. Hepatitis A 2. Measles, mumps, and rubella (MMR) 3. Haemophilus influenzae, type B 4. Diphtheria, tetanus, and acellular pertussis (DTaP), MMR, inactivated polio virus (IPV), and pneumococcal conjugate vaccine (PCV).

4. Diphtheria, tetanus, and acellular pertussis (DTaP), MMR, inactivated polio virus (IPV), and pneumococcal conjugate vaccine (PCV). Between ages 4 and 6 the child should receive DTaP, MMR, PCV, and IPV. Hepatitis A isn't a required immunization. MMR alone is incomplete. H. influenzae, type B immunization is completed by age 15 months.

A school-age child with fever and joint pain has just received a diagnosis of rheumatic fever. The child's parents ask the nurse whether anything could have prevented this disorder. Which intervention is most effective in preventing rheumatic fever? 1. Immunization with the hepatitis B vaccine 2. Isolation of individuals with rheumatic fever 3. Use of prophylactic antibiotics for invasive procedures 4. Early detection and treatment of streptococcal infections

4. Early detection and treatment of streptococcal infections Rheumatic fever is a systemic inflammatory disease that follows a group A streptococcal infection. Therefore, early detection and treatment of streptococcal infections help prevent the development of rheumatic fever. Hepatitis B vaccine provides immunity against the hepatitis B virus — not streptococci. Because rheumatic fever isn't contagious, isolation measures aren't necessary. Prophylactic antibiotics are used for invasive procedures only in clients with a history of carditis to prevent bacterial endocarditis.

The mother of an 8-year-old child being treated for right lower lobe pneumonia at home calls the clinic nurse. The mother tells the nurse that the child complains of discomfort on the right side and that the ibuprofen (Motrin) is not effective. The nurse should tell the mother to: 1. Increase the dose of the ibuprofen. 2. Increase the frequency of the ibuprofen. 3. Encourage the child to lie on the left side. 4. Encourage the child to lie on the right side.

4. Encourage the child to lie on the right side. Pneumonia is an inflammation of the pulmonary parenchyma or alveoli or both caused by a virus, mycoplasmal agents, bacteria, or aspiration of foreign substances. Splinting of the affected side by lying on that side may decrease discomfort. It would be inappropriate to advise the mother to increase the dose or frequency of the ibuprofen. Lying on the left side would not be helpful in alleviating discomfort.

The school nurse is planning a program for a group of teenagers on skin cancer prevention. Which instruction should the nurse emphasize in her talk? 1. Stay out of the sun between 1 p.m. and 3 p.m. 2. Tanning booths are a safe alternative for those who wish to tan. 3. Sun exposure is safe, provided the client wears protective clothing. 4. Examine skin once per month, looking for suspicious lesions or changes in moles.

4. Examine skin once per month, looking for suspicious lesions or changes in moles. To detect skin cancer in its early stages, the nurse should emphasize the importance of monthly skin self-examinations and yearly examinations by a physician. To reduce the risk of skin cancer, the nurse should teach clients to avoid the sun's ultraviolet rays between 10 a.m. and 3 p.m. Repeated exposure to artificial sources of ultraviolet radiation, such as tanning booths, increases the risk of skin cancer. While protective clothing offers some protection, some of the sun's harmful rays can penetrate clothing.

A preschool-age child underwent a tonsillectomy 4 hours ago. Which assessment finding would make the nurse suspect postoperative hemorrhage? 1. Vomiting of dark brown emesis 2. Refusal to drink clear fluids 3. Decreased heart rate 4. Frequent swallowing

4. Frequent swallowing Frequent swallowing — an attempt to clear the throat of trickling blood — suggests postoperative hemorrhage. Emesis may be brown or blood-tinged after a tonsillectomy; only bright red emesis signals hemorrhage. The child may refuse fluids because of painful swallowing, not bleeding. Hemorrhage is associated with an increased, not decreased, heart rate.

A mother brings her preschool child to the emergency department after ingesting an unknown quantity of acetaminophen. Which treatment will the physician probably prescribe? 1. Administration of a dose of ipecac syrup 2. Insertion of a nasogastric tube and administration of an antacid 3. I.V. infusion of normal saline solution 4. Gastric lavage and administration of activated charcoal

4. Gastric lavage and administration of activated charcoal The nurse should prepare for gastric lavage or activated charcoal administration. Ipecac syrup is no longer recommended. An antacid isn't an effective treatment for poisoning. Infusing normal saline solution I.V. may be helpful in treating dehydration caused by vomiting but in itself isn't effective in eliminating the poisonous substance.

The parents of a 4-year-old with sickle cell anemia tell the nurse that they would like to have other children, but they're concerned about passing sickle cell anemia on to them. Which health care team member would be the most appropriate person for the nurse to refer them to? 1. Clergy 2. Social worker 3. Certified nurse-midwife 4. Genetic counselor

4. Genetic counselor A genetic counselor can educate the couple about an inherited disorder, screening tests that can be done, and treatments and can provide emotional support. Clergy are available to provide spiritual support. A social worker can provide emotional support and help with referrals for financial problems. A nurse-midwife cares for women during pregnancy and birth.

An 11-year-old client contracted severe acute respiratory syndrome (SARS) when traveling abroad with her parents. The nurse knows she must put on personal protective equipment to protect herself while providing care. Based on the mode of SARS transmission, which personal protective equipment should the nurse wear? 1. Gloves 2. Gown and gloves 3. Gown, gloves, and mask 4. Gown, gloves, mask, and eye goggles or eye shield

4. Gown, gloves, mask, and eye goggles or eye shield The transmission of SARS isn't fully understood. Therefore, all modes of transmission must be considered possible, including airborne, droplet, and direct contact with the virus. For protection from contracting SARS, any health care worker providing care for a patient with SARS should wear a gown, gloves, mask, and eye goggles or an eye shield.

A 2-month-old baby hasn't received any immunizations. Which immunizations should the nurse prepare to administer? 1. Measles, mumps, rubella (MMR); diphtheria, tetanus, pertussis (DTP); and hepatitis B (HepB) 2. Polio (IPV), DTP, MMR 3. Varicella, Haemophilus influenzae type b (HIB), IPV, and DTP 4. HIB, DTP, HepB; and IPV

4. HIB, DTP, HepB; and IPV The current immunizations recommended for a 2-month-old who hasn't received any immunizations are HIB, DTP, HepB, and IPV. The first immunizations for MMR and varicella are recommended when a child is 12 months old.

An 8-year-old child enters a health care facility. During assessment, the nurse discovers that the child is experiencing the anxiety of separation from his parents. The nurse makes the nursing diagnosis of Fear related to separation from familiar environment and family. Which nursing intervention is most likely to help the child cope with fear and separation? 1. Ask the parents not to visit the child until he is adjusted to the new environment. 2. Ask the physician to explain to the child why he needs to stay in the health care facility. 3. Explain to the child that he must act like an "adult" while he's in the facility. 4. Have the parents stay with the child and participate in his care.

4. Have the parents stay with the child and participate in his care. Allowing the parents to stay and participate in the child's care can provide support to both the parents and the child. The other interventions won't address the client's diagnosis and may exacerbate the problem.

How should the nurse position an infant when administering an oral medication? 1. Seated in a high chair 2. Restrained flat in the crib 3. Held on the nurse's lap 4. Held in the bottle-feeding position

4. Held in the bottle-feeding position The nurse should hold an infant in the bottle-feeding position when administering an oral medication: place the child's inner arm behind the back, support the head in the crook of the elbow, and hold the child's free hand with the hand of the supporting arm. An infant can't sit unsupported in a high chair. Administering medication to an infant lying flat could cause choking and aspiration. Holding the infant in the lap doesn't prevent spilling the medication with either hand.

An infant is having his 2-month checkup at the pediatrician's office. The physician tells the parents that she's assessing for Ortolani's sign. The nurse explains that the presence of Ortolani's sign indicates dislocation of what joint? 1. Shoulder 2. Elbow 3. Knee 4. Hip

4. Hip To assess for Ortolani's sign, the nurse abducts the infant's hips while flexing the legs at the knees. This is performed on all infants to assess for congenital hip dislocation. The examiner listens and feels for a "click" as the femoral head enters the acetabulum during the examination. This finding indicates a congenitally dislocated hip.

A 14-year-old girl in skeletal traction for treatment of a fractured femur is expected to be hospitalized for several weeks. When planning care, the nurse should take into account the client's need to achieve what developmental milestone? 1. Autonomy 2. Initiative 3. Industry 4. Identity

4. Identity According to Erikson's theory of personal development, the adolescent is in the stage of identity versus role confusion. During this stage, the body is changing as secondary sex characteristics emerge. The adolescent is trying to develop a sense of identity, and peer groups take on more importance. When an adolescent is hospitalized, she is separated from her peer group and body image may be altered. Toddlers are in the developmental stage of autonomy versus shame and doubt. Preschool children are in the stage of initiative versus guilt. School-age children are in the stage of industry versus inferiority.

The mother of a 16-year-old girl calls the emergency department, suspecting her daughter's abdominal pain may be appendicitis. In addition to pain, her daughter has a temperature of 100° F (37.7° C) and has vomited twice. What should the nurse tell the mother? 1. Give the daughter a laxative to rule out the possibility that constipation is causing the pain. 2. Gently press on the lower left quadrant of her daughter's abdomen to test for rebound tenderness. 3. It's most likely the flu because her daughter is too young to have appendicitis. 4. Immediately bring her daughter into the emergency department before the appendix has a chance to rupture.

4. Immediately bring her daughter into the emergency department before the appendix has a chance to rupture. Abdominal pain, low-grade fever, and vomiting are cardinal signs of appendicitis. Rebound tenderness is also a symptom but would be found in the right lower quadrant. Administration of a laxative during appendicitis is very dangerous because it may cause the appendix to rupture.

A 2-year-old child in the cardiac step-down unit is experiencing supraventricular tachycardia. Which intervention should be attempted first? 1. Administering digoxin (Lanoxin) I.V. 2. Administering verapamil (Calan) I.V. 3. Administering synchronized cardioversion 4. Immersing the child's hands in cold water

4. Immersing the child's hands in cold water Vagal maneuvers such as immersing the child's hands in cold water are often tried first as a mechanism to decrease heart rate. Other vagal maneuvers include breath-holding, gagging, and placing the child's head lower than the rest of the body. Synchronized cardioversion may be necessary if vagal maneuvers fail and drugs are ineffective. Verapamil isn't recommended. Digoxin may be given after vagal maneuvers to help decrease heart rate.

The nurse is deciding whether to report a suspected case of child abuse. Which criterion is the most important for the nurse to consider? 1. Inappropriate parental concern for the degree of injury 2. Absence of parents to question about the injury 3. Inappropriate response of the child to the injury 4. Incompatibility between the history and the injury

4. Incompatibility between the history and the injury Incompatibility between the history and the injury is the most important criterion on which to base the decision to report suspected child abuse. The other criteria also may suggest child abuse but are less reliable indicators.

The nurse is providing care to a 5-year-old child with a fractured femur whose nursing diagnosis is Imbalanced nutrition: Less than body requirements. Which of the following is most likely to occur with this condition? 1. Decreased protein catabolism 2. Increased calorie intake 3. Increased digestive enzymes 4. Increased carbohydrate need

4. Increased carbohydrate need Carbohydrate need increases because healing and repair of tissue requires more carbohydrates. Increased — not decreased — protein catabolism is present. Decreased appetite — not increased — is a problem. Digestive enzymes are decreased — not increased.

The nurse is assessing an infant for signs of increased intracranial pressure (ICP). What is the earliest sign of increased ICP in an infant? 1. Vomiting 2. Papilledema 3. Headache 4. Increased head circumference

4. Increased head circumference Increased head circumference is the first sign of increased ICP in an infant. Vomiting occurs later. Papilledema is a late sign of increased ICP and may not be evident. Because the infant can't speak, the nurse would have trouble determining whether the infant has a headache.

A child, age 15 months, is recovering from surgery to remove a Wilms' tumor. Which finding best indicates that the child is free from pain? 1. Decreased appetite 2. Increased heart rate 3. Decreased urine output 4. Increased interest in play

4. Increased interest in play A behavioral change is one of the most valuable clues to pain. A child who's pain-free likes to play; in contrast, a child in pain is less likely to play or to consume food or fluids. An increased heart rate indicates increased pain. Decreased urine output may signify dehydration.

Which factor will most likely decrease drug metabolism during infancy? 1. Decreased glomerular filtration 2. Reduced protein-binding ability 3. Increased tubular secretion 4. Inefficient liver function

4. Inefficient liver function Inefficient liver function will most likely decrease drug metabolism during infancy. As the liver matures during the 1st year of life, drug metabolism improves. Decreased glomerular filtration and increased tubular secretion may affect drug excretion rather than metabolism; reduced protein-binding ability may affect drug distribution but not metabolism.

A mother arrives at a clinic with her toddler and tells a nurse that she has a difficult time getting the child to go to bed at night. Which of the following is appropriate for the nurse to suggest to the mother? 1. Avoid a nap during the day. 2. Allow the child to set bedtime limits. 3. Allow the child to have temper tantrums. 4. Inform the child of bedtime a few minutes before it is time for bed.

4. Inform the child of bedtime a few minutes before it is time for bed. Toddlers often resist going to bed. Bedtime protests may be reduced by establishing a consistent before-bedtime routine and enforcing consistent limits regarding the child's bedtime behavior. Informing the child of bedtime a few minutes before it is time for bed is the most appropriate option. Most toddlers take an afternoon nap and, until their second birthday, also may require a morning nap. Firm, consistent limits are needed for temper tantrums or when toddlers try stalling tactics.

A mother is discontinuing breast-feeding after 5 months. The nurse should advise her to include which of the following in her infant's diet? 1. Iron-rich formula and baby food 2. Whole milk and baby food 3. Skim milk and baby food 4. Iron-rich formula alone

4. Iron-rich formula alone The American Academy of Pediatrics recommends iron-rich formula for 5-month-old infants and cautions against giving infants solid food — even baby food — until age 6 months. The Academy doesn't recommend whole milk before age 12 months or skim milk before age 2 years.

An infant arrives at the emergency department in full cardiopulmonary arrest. Efforts at resuscitation fail, and he's pronounced dead. The cause of death is sudden infant death syndrome (SIDS). Which statement is true regarding the etiology of SIDS? 1. It occurs in suspected child abuse cases. 2. It occurs primarily in infants with congenital lung problems. 3. It occurs primarily in black infants. 4. It occurs more commonly in infants who sleep in the prone position.

4. It occurs more commonly in infants who sleep in the prone position. SIDS occurs in seemingly healthy infants. However, more infants who sleep in the prone position are affected. Because of the pooling of blood that occurs in the child with SIDS, child abuse is sometimes suspected. No correlation to race or lung disease exists.

A child with laryngotracheobronchitis (croup) is placed in a cool mist tent. The mother becomes concerned because the child is frightened, consistently crying and trying to climb out of the tent. The appropriate nursing action is to: 1. Tell the mother that the child must stay in the tent. 2. Call the physician and obtain a prescription for a mild sedative. 3. Place a toy in the tent to make the child feel more comfortable. 4. Let the mother hold the child and direct the cool mist over the child's face.

4. Let the mother hold the child and direct the cool mist over the child's face. Laryngotracheobronchitis (croup) is the inflammation of the larynx, trachea, and bronchi and is the most common type of croup. Cool mist therapy may be prescribed to liquefy secretions and to assist in breathing. If the use of a tent or hood is causing distress, treatment may be more effective if the child is held by the parent and a cool mist is directed toward the child's face. A mild sedative would not be administered to the child. Crying would increase hypoxia and aggravate laryngospasm, which may cause airway obstruction. Options 1 and 3 would not alleviate the child's fear.

After a series of tests, a 6-year-old client weighing 50 lb (22.7 kg) is diagnosed with complex partial seizures. The physician prescribes phenytoin (Dilantin), 125 mg by mouth twice per day. After the nurse administers phenytoin, where is the drug metabolized? 1. Pancreas 2. Kidneys 3. Stomach 4. Liver

4. Liver Phenytoin is metabolized in the liver. The pancreas isn't involved in the pharmacokinetic activity of phenytoin. The stomach absorbs orally administered phenytoin, which is excreted by the kidneys in the urine.

The nurse manager of the pediatric unit is responsible for making sure that each staff member reviews the unit policies annually. What policy should the nurse manager emphasize with the clerical support staff? 1. Proper documentation of a verbal order from a physician 2. Policy changes in the administration of opioids 3. New education materials for the management of diabetes 4. Logging off a computer containing client information

4. Logging off a computer containing client information All members of the health care team are required to maintain strict patient confidentiality, including securing electronic patient information. Therefore, the clerical support staff should be instructed about the importance of logging off a computer containing client information immediately after use. Taking a verbal order, administering medications, and patient education aren't within the scope of practice of the clerical support staff.

An infant is hospitalized for treatment of inorganic failure to thrive. Which nursing action is most appropriate for this child? 1. Encouraging the infant to hold a bottle 2. Keeping the infant on bed rest to conserve energy 3. Rotating caregivers to provide more stimulation 4. Maintaining a consistent, structured environment

4. Maintaining a consistent, structured environment The nurse caring for an infant with inorganic failure to thrive should strive to maintain a consistent, structured environment. Encouraging the infant to hold a bottle would reinforce an uncaring feeding environment. The infant should receive social stimulation rather than be confined to bed rest. The number of caregivers should be minimized to promote consistency of care.

A 4-year-old girl has a urinary tract infection (UTI). When teaching the parents how to help her avoid recurrent UTIs, the nurse should emphasize which preventive measure? 1. Wiping her perineum from back to front after she uses the toilet 2. Administering prophylactic antibiotics 3. Giving her a warm bath for 15 minutes daily 4. Making sure she avoids bubble baths

4. Making sure she avoids bubble baths The child should avoid bubble baths because oils in the bubble bath preparation may irritate the urethra, contributing to UTIs. Girls and women should wipe the perineum from front to back, not back to front, to avoid contaminating the urinary tract with fecal bacteria. Although antibiotics are used to treat UTIs, they aren't given prophylactically. No evidence suggests that warm baths help prevent UTIs.

What should the nurse do to ensure a safe hospital environment for a toddler? 1. Place the child in a youth bed. 2. Move stacking toys out of reach. 3. Pad the crib rails. 4. Move the equipment out of reach.

4. Move the equipment out of reach. Toddlers are curious and may try to play with items such as equipment that is within their reach. Doing so is dangerous. Toddlers in a strange hospital environment still need the security of a crib. Padded crib rails are necessary if seizure activity is present. Stacking toys are appropriate for this age-group and don't present a safety hazard.

A toddler is diagnosed with a dislocated right shoulder and a simple fracture of the right humerus. Which behavior most strongly suggests that the child's injuries stem from abuse? 1. Trying to sit up on the stretcher 2. Trying to move away from the nurse 3. Not answering the nurse's questions 4. Not crying when moved

4. Not crying when moved Not crying when moved most strongly suggests child abuse. A victim of child abuse typically doesn't complain of pain, even with obvious injuries, for fear of further displeasing the abuser. Trying to sit up on the stretcher is a typical client response. Trying to move away from the nurse indicates fear of strangers, which is normal in a toddler. Difficulty answering the nurse's questions is expected in a toddler because of poorly developed cognitive skills.

The mother of a 5-year-old child who was admitted to the hospital has a Protection from Abuse order for the child against his father. A copy of the order is kept on the pediatric medical surgical unit where the child is being treated. The order prohibits the father from having any contact with the child. One night, the father approaches the nurse at the nurses' station, politely but insistently demanding to see his child, and refusing to leave until he does so. What should the nurse do first? 1. Firmly tell the father he must leave. 2. Notify the nursing coordinator on duty. 3. Notify the nurse-manager. 4. Notify hospital security or the local authorities.

4. Notify hospital security or the local authorities. The Protection from Abuse order legally prohibits the father from seeing the child. In this situation, the nurse should notify hospital security or the local authorities of this attempt to breach the order, and allow them to escort the father out of the building. The father could be jailed or fined if he violates the order. The nurse shouldn't argue or continue explaining to the father that he must leave, which could place her and the child at risk if the father becomes angry or agitated. The nursing coordinator and nurse-manager should be notified of the incident; the nurse's first priority, however, should be contacting security or the authorities.

Which intervention takes priority when admitting an infant with acute gastroenteritis? 1. Obtaining a stool specimen 2. Weighing the infant 3. Offering the infant clear liquids 4. Obtaining a history of the illness

4. Obtaining a history of the illness Obtaining a history of the infant's illness is a priority for developing a treatment plan. Getting a stool specimen and weighing the infant can follow taking the history. The nurse shouldn't offer clear liquids because they increase the risk of vomiting, which may worsen the infant's dehydration.

In the parents of a chronically ill child, which behavior may indicate feelings of guilt about the child's illness? 1. Anger 2. Sadness 3. Shock 4. Overindulgence

4. Overindulgence Parents who feel guilty about a child's illness may overindulge the child. Anger, sadness, and shock are common in parents of chronically ill children but don't necessarily indicate feelings of guilt.

In planning the care of an infant undergoing phototherapy for hyperbilirubinemia, which of the following would be least appropriate? 1. Repositioning the infant frequently to expose all body surfaces 2. Obtaining frequent serum bilirubin levels 3. Shielding the infant's eyes with an opaque mask to prevent exposure to the light 4. Performing frequent visual assessments of jaundice

4. Performing frequent visual assessments of jaundice Visual assessment of jaundice isn't a valid method for assessing jaundice. Serum bilirubin levels must be checked every 4 to 12 hours. Repositioning the infant and shielding the infant's eyes are appropriate interventions for an infant undergoing phototherapy.

A toddler is admitted to the facility for treatment of a severe respiratory infection. The child's recent history includes fatty stools and failure to gain weight steadily. The physician diagnoses cystic fibrosis. By the time of the child's discharge, the child's parents must be able to perform which task independently? 1. Allergy-proofing the home 2. Maintaining the child in an oxygen tent 3. Maintaining the child on a fat-free diet 4. Performing postural drainage

4. Performing postural drainage The client with cystic fibrosis is at risk for frequent respiratory infections secondary to increased viscosity of mucus gland secretions. To help prevent respiratory infections, caregivers must perform postural drainage several times daily to loosen and drain secretions. Because exocrine gland dysfunction, not an allergic response, causes bronchial obstruction in cystic fibrosis, allergy-proofing the home isn't necessary. Oxygen therapy may be indicated, but only during acute disease episodes. Also, such therapy must be supervised closely; home oxygen therapy is inappropriate because chronic hypoxemia poses the risk of oxygen toxicity. If steatorrhea can't be controlled, the child should reduce, but not eliminate, dietary fat intake.

A toddler requires emergency intervention for an obstructed airway. Which nursing intervention is appropriate? 1. Hyperextending the child's neck to open the airway and delivering four rescue breaths 2. Placing the child on the side and using a blind finger sweep to remove the foreign object 3. Delivering four back blows followed by four chest thrusts 4. Performing the tongue-jaw lift and removing the foreign object only if it's visible

4. Performing the tongue-jaw lift and removing the foreign object only if it's visible When checking for a foreign object in the airway of a child younger than age 8, the rescuer should perform the tongue-jaw lift and remove the object only if it's visible. Neck hyperextension may occlude the airway; the head tilt/chin lift method is the correct way to open the airway. A blind finger sweep is contraindicated in a child under age 8 because it may push the object into the airway. After checking for a foreign object, the rescuer should open the airway and attempt to deliver two rescue breaths. Abdominal thrusts (the Heimlich maneuver) are indicated only for children older than age 1. In a child younger than age 1, such thrusts may injure the abdominal organs; back blows and chest thrusts should be used instead.

A school-age child is admitted to the facility with a diagnosis of acute lymphoblastic leukemia (ALL). The nurse formulates a nursing diagnosis of Risk for infection. What is the most effective way for the nurse to reduce the child's risk of infection? 1. Implementing reverse isolation 2. Maintaining standard precautions 3. Requiring staff and visitors to wear masks 4. Practicing thorough hand washing

4. Practicing thorough hand washing Both ALL and its treatment cause immunosuppression. Thorough hand washing is the single most effective way to prevent infection in an immunosuppressed client. Reverse isolation doesn't significantly reduce the incidence of infection in immunosuppressed clients; furthermore, isolation may cause psychological stress. Standard precautions are intended mainly to protect caregivers from contact with infectious matter, not to reduce the client's risk of infection. Staff and others needn't wear masks when visiting because most infections are transmitted by direct contact. Instead of relying on masks and other barrier methods, the nurse should keep persons with known infections out of the client's room.

When assessing a child with bronchiolitis, which finding does the nurse expect? 1. Clubbed fingers 2. Barrel chest 3. Barking cough and stridor 4. Productive cough

4. Productive cough Bronchiolitis causes a productive cough. Clubbed fingers and a barrel chest are more likely in a client with chronic respiratory problems. A barking cough is associated with acute laryngotracheobronchitis.

When attempting to facilitate spiritual support for a school-age child with a life-threatening disease and her family, which of the following would be least effective? 1. Becoming familiar with the family's spiritual beliefs and practices 2. Seeking assistance or referrals to the facility chaplain or other resources 3. Being open to the family's and the child's expressions of spiritual concerns 4. Promoting the nurse's personal values and beliefs if she considers the family's to be inappropriate

4. Promoting the nurse's personal values and beliefs if she considers the family's to be inappropriate The nurse needs to recognize that her personal values and beliefs may not be effective for others and that she shouldn't judge others' beliefs. Becoming familiar with the family's spiritual beliefs and practices, seeking assistance or referrals to the facility chaplain or other resources, and being open to the family's and the child's expressions of spiritual concerns are all ways to help children and their families use their spiritual beliefs to cope with a life-threatening illness.

A girl, age 15, is 7 months pregnant. When teaching parenting skills to an adolescent, the nurse knows that which teaching strategy is least effective? 1. Providing a one-on-one demonstration and requesting a return demonstration, using a live infant model 2. Initiating a teenage-parent support group with first- and second-time mothers 3. Using audiovisual aids that show discussions of feelings and skills 4. Providing age-appropriate reading materials

4. Providing age-appropriate reading materials Because adolescents absorb less information through reading, providing age-appropriate reading materials is the least effective way to teach parenting skills to an adolescent. The other options engage more than one of the senses and therefore serve as effective teaching strategies.

When administering an I.M. injection to an infant, the nurse should use which site? 1. Deltoid 2. Dorsogluteal 3. Ventrogluteal 4. Vastus lateralis

4. Vastus lateralis The recommended injection site for an infant is the vastus lateralis or rectus femoris muscle. The deltoid is inappropriate. The dorsogluteal and ventrogluteal sites can be used only in toddlers who have been walking for about 1 year.

A 5-year-old child is brought to the emergency department after being stung multiple times on the face by yellow jackets. Which symptom of anaphylaxis requires priority medical intervention? 1. Blood pressure of 95/50 mm Hg 2. Diffuse facial urticaria 3. Respiratory rate of 20 breaths/minute 4. Pulse rate of 60 beats/minute

4. Pulse rate of 60 beats/minute Bradycardia is an ominous sign in children. Older children initially demonstrate tachycardia in response to hypoxemia. When tachycardia can no longer maintain tissue oxygenation, bradycardia follows. The development of bradycardia usually precedes cardiopulmonary arrest. The average systolic blood pressure of children ages 1 to 7 can be determined by this formula: age in years plus 90. Thus, an average blood pressure for a 5-year-old child is 95 mm Hg. Urticaria should be treated after airway control has been established. The normal respiratory rate for a 5-year-old is 20 to 25 breaths/minute.

Practice Questions 325. A clinic nurse is preparing to discuss the concepts of moral development with a mother. The nurse understands that according to Kohlberg's theory of moral development, in the preconventional level, moral development is thought to be motivated by which of the following? 1. Peer pressure 2. Social pressures 3. Parents' behavior 4. Punishment and reward

4. Punishment and reward In the preconventional stage, morals are thought to be motivated by punishment and reward. If the child is obedient and is not punished, then the child is being moral. The child sees actions as good or bad. If the child's actions are good, the child is praised. If the child's actions are bad, the child is punished. Options 1, 2, and 3 are incorrect for this stage of moral development.

A 2-month-old is brought to the clinic by his mother. His abdomen is distended, and he has been vomiting forcefully and with increasing frequency over the past 2 weeks. On examination, the nurse notes signs of dehydration and a palpable mass to the right of the umbilicus. Peristaltic waves are visible, moving from left to right. The nurse should suspect which of the following conditions? 1. Colic 2. Failure to thrive 3. Intussusception 4. Pyloric stenosis

4. Pyloric stenosis These are classic symptoms of pyloric stenosis caused by hypertrophy of the circular pylorus muscle. Surgery is the standard treatment for this disorder. Abdominal masses and abnormal peristalsis aren't necessarily related to colic or failure to thrive. Intussusception is usually characterized by acute onset and severe abdominal pain.

Which of the following activities should a 2-year-old child to be able to do? 1. Build a tower of eight cubes. 2. Point out a picture. 3. Wash and dry his hands. 4. Remove a garment.

4. Remove a garment. According to the Denver Developmental Screening Test, most 2-year-olds are able to remove one garment. A 2½-year-old can build a tower of eight cubes and point out a picture. A 3-year-old can wash and dry his hands.

The nurse suspects that a toddler, who is admitted to the pediatric unit, has been physically abused by his mother. What is the nurse required to do? 1. Talk with the child about she suspects. 2. Confront the mother with her suspicions. 3. Discuss the case with another nurse during lunch break. 4. Report the case to local authorities.

4. Report the case to local authorities. Every state in the United States has laws for mandatory reporting of suspected child abuse and neglect. These cases are then referred to local agencies, such as Child Protective Services, for investigation. Social workers should be consulted before approaching a child and discussing child abuse. Confronting the mother could increase the risk of harm to the child and to the nurse. Discussing the case with another nurse breaches the client's confidentiality.

A toddler with bacterial meningitis is admitted to the inpatient unit. Which infection control measure should the nurse be prepared to use? 1. Reverse isolation 2. Strict hand washing 3. Standard precautions 4. Respiratory isolation

4. Respiratory isolation Because bacterial meningitis is transmitted by droplets from the nasopharynx, the nurse should prepare to use respiratory isolation. This includes wearing a gown and gloves during direct client care and ensuring that everyone who enters the child's room wears a mask. Reverse isolation is unnecessary; it's used for immunosuppressed clients who are at high risk for acquiring infection. Strict hand washing and standard precautions are insufficient for this client because they don't require the use of a mask.

A 2 1/2-year-old child is being treated for left lower lobe pneumonia. In what position should the nurse position the toddler to maximize oxygenation? 1. Prone 2. Left lateral 3. Supine 4. Right lateral

4. Right lateral The toddler should be positioned on his right side. Gravity contributes to increased blood flow to the right lung, thereby allowing for better gas exchange. Positioning the child prone, supine, or in the left lateral position doesn't allow for better gas exchange in this child.

A female adolescent client refuses to allow male nurses to care for her while she's hospitalized. Which of these health care rights is this client exerting? 1. Right to competent care 2. Right to have an advance directive on file 3. Right to confidentiality of her medical record 4. Right to privacy

4. Right to privacy The client is exhibiting her right to privacy when she requests that she doesn't want a male nurse to care for her. The client also has a right to competent care, the right to have an advance directive on file, and a right to confidentiality. However, the client isn't exercising these rights in this scenario.

An infant, age 3 months, undergoes surgical repair of a cleft lip. After surgery, the nurse should use which equipment to feed the infant? 1. Single-hole nipple 2. Plastic spoon 3. Paper straw 4. Rubber dropper

4. Rubber dropper An infant with a surgically repaired cleft lip must be fed with a rubber dropper or Breck feeder to prevent sucking or suture line trauma. The other options wouldn't prevent these actions.

An infant, 6 weeks old, is brought to the clinic for a well-baby visit. To assess the fontanels, how should the nurse position the infant? 1. Supine 2. Prone 3. In the left lateral position 4. Seated upright

4. Seated upright For the most accurate results, the nurse should seat the infant upright to assess the fontanels and should perform this assessment when the infant is quiet. Pressure from postural changes or intense crying may cause the fontanels to bulge or seem abnormally tense. When the infant is in a recumbent position, the fontanel is less flat than it is normally, creating the false impression that intracranial pressure is increased.

When a preschooler's family displays high levels of mistrust, monitors everyone's performance, wants high levels of information, and asks for rule changes, which strategy would be inappropriate? 1. Ask their opinion and use their suggestions. 2. Be positive about building a trusting relationship. 3. Be flexible regarding rules. 4. Show support while controlling the care of the child.

4. Show support while controlling the care of the child. When a family shows high levels of mistrust, monitors everyone's performance, requires high levels of information, and requests rule changes, the nurse should attempt to build a partnership with the family to reach the goal of mutually caring for the child. Attempting to control care with this family will alienate the family, stress the relationship, and be counterproductive for the child. Good strategies for working with this family would be to ask their opinion, use their suggestions, be positive about building a trusting relationship, and be flexible regarding rules.

A preschool-age child refuses to take prescribed medication. Which nursing strategy would be most appropriate? 1. Mixing the medication in milk so the child isn't aware that it's there 2. Explaining the medication's effects in detail to ensure cooperation 3. Making the child feel ashamed for not cooperating 4. Showing trust in the child's ability to cooperate even with an unpleasant procedure

4. Showing trust in the child's ability to cooperate even with an unpleasant procedure To gain a preschooler's cooperation, the nurse should show trust and express faith in the child's ability to cooperate even with an unpleasant procedure. Hiding the medication in milk may foster mistrust. The nurse should provide simple, not detailed, explanations and should use terms the child can understand. Shaming the child is inappropriate and may lead to feelings of guilt.

Which of the following objects poses the most serious safety threat to a 2-year-old child in the hospital? 1. Crayons and paper 2. Stuffed teddy bear in the crib 3. Mobile hanging over the crib 4. Side rails in the halfway position

4. Side rails in the halfway position To prevent falls — one of the most common accidents in hospitals — the crib rails always should be raised and fastened securely unless an adult is at the bedside. Crayons and paper and a stuffed teddy bear are safe toys for a 2-year-old child. Although a mobile could pose a safety threat to this child, the threat is less serious than that posed by an incorrectly positioned side rail.

A child, age 13, is being evaluated for possible Crohn's disease. The nurse expects to prepare the child for which diagnostic study? 1. Genetic testing 2. Cystoscopy 3. Myelography 4. Sigmoidoscopy

4. Sigmoidoscopy Crohn's disease is an inflammatory bowel disorder characterized by inflammation, ulceration, and edema of the bowel wall (typically involving the terminal ileum). Sigmoidoscopy and biopsy are the primary procedures used to establish the diagnosis; a barium enema also may be indicated. Although genetics may play a role in Crohn's disease, genetic testing isn't part of the diagnostic workup. Cystoscopy visualizes the bladder and urinary tract and isn't indicated for this client. Myelography is a radiographic procedure used to evaluate the spinal cord.

To examine an infant's thyroid gland, the nurse should place the infant in which position? 1. Prone 2. Sitting 3. Standing 4. Supine

4. Supine To examine an infant's thyroid gland, the nurse should hyperextend the infant's neck slightly while the child is held supine on the caregiver's lap. Neck hyperextension promotes thyroid palpation by elongating the surface area of the infant's characteristically short neck. A prone position wouldn't allow an adequate area for palpation. A sitting position is appropriate when assessing the thyroid gland of an older child or an adult. An infant can't stand, so option 3 is inappropriate.

When caring for a 12-month-old infant with dehydration and metabolic acidosis, the nurse expects to see which of the following? 1. A reduced white blood cell (WBC) count 2. A decreased platelet count 3. Shallow respirations 4. Tachypnea

4. Tachypnea The body compensates for metabolic acidosis via the respiratory system, which tries to eliminate the buffered acids by increasing alveolar ventilation through deep, rapid respirations. Altered WBC and platelet counts aren't specific signs of metabolic imbalance.

The nurse on the pediatric unit is caring for a group of preschool clients. Which situation takes lowest priority? 1. A client who develops a fever during a blood transfusion 2. A client admitted from the postanesthesia care unit who has a blood-saturated surgical dressing 3. A physician waiting on the telephone to give the nurse a verbal order 4. Taking a lunch break

4. Taking a lunch break Taking a lunch break takes lowest priority over client care. If the nurse is unable to delegate client care responsibilities to another nurse or nursing assistant, the nurse's lunch break needs to be rescheduled. A fever indicates an adverse reaction to the blood transfusion, and requires immediate intervention. The post-surgical client is losing blood through the surgical incision, which also requires attention. The telephone call is important for medication changes and to prevent a delay in treatment.

The parents of a school-age child with asthma express concern about letting the child participate in sports. What should the nurse tell the parents about the relationship between exercise and asthma? 1. Asthma attacks are triggered by allergens, not exercise. 2. The child should avoid exercise because it may trigger asthma attacks. 3. Continuous activities such as jogging are less likely to trigger asthma than intermittent activities such as baseball. 4. Taking prophylactic drugs before the activity can prevent asthma attacks and enable the child to engage in most sports.

4. Taking prophylactic drugs before the activity can prevent asthma attacks and enable the child to engage in most sports. Although exercise may trigger asthma attacks, taking prophylactic asthma drugs before beginning the activity can prevent attacks, enabling the child to engage in most sports. Asthma attacks may be triggered by various factors, including allergens, exercise, medications, upper respiratory tract infections, and psychological stress. Provided the asthma is under control, most children can participate in sports and other physical activities; in fact, they benefit from exercise. Activity restrictions hamper peer interaction, which is essential to the development of the school-age child. A child with asthma may tolerate intermittent activities better than continuous ones.

The nurse is helping a pregnant client devise a plan to help her 2-year-old adjust to the birth of her second child. Which suggestion is least appropriate? 1. Provide the toddler with a doll so he can imitate the parents when they care for the newborn. 2. Enroll the toddler in a sibling class offered at the local hospital. 3. Discuss with the toddler what routines will be the same and what will be different after the baby arrives. 4. Tell the toddler that the new baby can be a playmate when he arrives.

4. Tell the toddler that the new baby can be a playmate when he arrives. Telling a toddler that he will have a new playmate when the baby arrives sets up unrealistic expectations. Instead, the parents should stress activities that will take place, such as feeding, changing, and crying. Giving the toddler a doll is a good strategy because having the doll allows the toddler to take part in the new routines. For example, the toddler can pretend to meet the needs of the doll just like the mother tends to the baby. Participation in a sibling preparation class may also decrease sibling rivalry behaviors. Discussing changes in family routines will help the toddler know what to expect.

The nurse teaches a mother how to provide adequate nutrition for her toddler, who has cerebral palsy. Which observation indicates that teaching has been effective? 1. The toddler stays neat while eating. 2. The toddler finishes the meal within a specified period of time. 3. The child lies down to rest after eating. 4. The child eats finger foods by himself.

4. The child eats finger foods by himself. The child with cerebral palsy should be encouraged to be as independent as possible. Finger foods allow the toddler to feed himself. Because spasticity affects coordinated chewing and swallowing as well as the ability to bring food to the mouth, it's difficult for the child with cerebral palsy to eat neatly. Independence in eating should take precedence over neatness. The child with cerebral palsy may require more time to bring food to the mouth; thus, chewing and swallowing shouldn't be rushed to finish a meal by a specified time. The child with cerebral palsy may vomit after eating owing to a hyperactive gag reflex. Therefore, the child should remain in an upright position after eating to prevent aspiration and choking.

The nurse is caring for a child with tetralogy of Fallot. The child's mother becomes concerned when she visits her son and notices him sucking his thumb, a behavior that he had previously given up. What does this behavior indicate? 1. The child is depressed. 2. The child is in pain. 3. The child wants attention. 4. The child is responding to stress.

4. The child is responding to stress. Regression (reverting back to previously outgrown behaviors) is a common response to stressful situations. The nurse should reassure the parents that thumb sucking and other regressive behaviors should disappear after the stressful situation is resolved. Thumb sucking isn't a sign of depression or pain or an attention-seeking behavior.

A mother asks the nurse how to handle her 4-year-old child, who recently started wetting the pants after being completely toilet-trained. The child just started attending nursery school 2 days a week. Which principle should guide the nurse's response? 1. The child forgets previously learned skills. 2. The child experiences growth while regressing, regrouping, and then progressing. 3. The parents may prefer less mature behaviors. 4. The child returns to a level of behavior that increases the sense of security.

4. The child returns to a level of behavior that increases the sense of security. The stress of starting nursery school may trigger a return to a level of successful behavior from earlier stages of development. A child's skills remain intact, although increased stress may prevent the child from using these skills. Growth occurs when the child doesn't regress. Parents rarely desire less mature behaviors.

A 10-year-old child is in the hospital for the first time. The nurse has provided support and teaching to help the family and child adjust and to reduce their anxiety related to the child's hospitalization. Which of the following would the nurse view as unexpected? 1. The parents relate readily with the staff and calmly with the child. 2. The child accepts and responds positively to comforting measures. 3. The child discusses procedures and activities without evidence of anxiety. 4. The parents choose to leave to let the child build a relationship with the staff.

4. The parents choose to leave to let the child build a relationship with the staff. The parents of an adolescent might leave to help the teen maintain a fragile identity, but a 10-year-old child would prefer to have his parents with him. Expected outcomes for a child and parents new to the hospital would include the parents' relating readily to the staff and calmly with the child, the child's accepting and responding positively to comforting measures, and the child's discussing procedures and activities without evidence of anxiety.

A toddler is hospitalized for treatment of injuries that the staff believes were caused by child abuse. A staff member states that the parents "shouldn't be allowed to visit because they caused the child's injuries." When responding to this staff member, the nurse should base the comments on which understanding? 1. The parents shouldn't be allowed to visit the child. 2. The parents shouldn't visit until the child is ready for discharge. 3. The parents should visit on a schedule established by the health care team and should be supervised during visits. 4. The parents should be encouraged to visit frequently and should be welcomed by the staff.

4. The parents should be encouraged to visit frequently and should be welcomed by the staff. Abusive parents should be encouraged to visit their child and should be welcomed by the staff. Many abusive parents love their children but lack effective parenting skills. The child's hospitalization offers an opportunity for the staff to demonstrate appropriate parenting behaviors to the parents.

Which desired outcome demonstrates effective parent teaching about disciplining a toddler? 1. The parents will set flexible rules. 2. The parents will verbalize requests for behavior in negative terms. 3. The parents will raise their voices when reprimanding the child. 4. The parents will call immediate attention to undesirable behavior.

4. The parents will call immediate attention to undesirable behavior. The parents of a toddler should praise desirable behavior and call immediate attention to undesirable behavior. This helps the child learn socially acceptable behavior and maintain self-esteem and a positive self-concept while learning to adapt to the rules of the larger group and society. Rules should be established clearly and enforced consistently. To reinforce desirable behavior, parents should voice requests for behavior in positive terms and use a normal speaking voice and tone when talking to or reprimanding the child. Screaming and shouting should be minimized.

Which assessment finding would the nurse identify as abnormal for a 4-month-old? 1. The abdominal wall is rising with inspiration. 2. The respiratory rate is between 30 and 35 breaths/minute. 3. The infant's skin is mottled during examination. 4. The spaces between the ribs (intercostal) are delineated during inspiration.

4. The spaces between the ribs (intercostal) are delineated during inspiration. The presence of intercostal retractions is a sign of respiratory distress from an obstruction or a disease such as pneumonia, which causes the infant to have to work to breathe. Infants and children up to age 7 are abdominal breathers; after that age, they change to an adult pattern of breathing, which uses the diaphragmatic and thoracic muscles. A normal respiratory rate for an infant up to age 1 is 20 to 40 breaths/minute. An infant's skin can become mottled if the infant is left uncovered during the examination.

A child, age 9, is admitted to the emergency department with pain in the right lower abdominal quadrant, suggesting appendicitis. To further assess for pain associated with appendicitis, the nurse should examine which region? 1. Left lower abdominal quadrant 2. Right upper abdominal quadrant 3. Left upper abdominal quadrant 4. Umbilical area

4. Umbilical area In a child, pain that is diffuse or centered on the umbilicus may be associated with appendicitis. (However, the pain may localize later.) Pain in the left lower abdominal quadrant is associated with descending and sigmoid colon problems; in the right upper quadrant, with gallbladder disease; and in the left upper quadrant, with pancreatitis.

Which assessment finding would lead the nurse to suspect dehydration in a preterm neonate? 1. Bulging fontanels 2. Excessive weight gain 3. Urine specific gravity below 1.012 4. Urine output below 1 ml/hour

4. Urine output below 1 ml/hour Urine output below 1 ml/hour is a sign of dehydration. Other signs of dehydration include depressed fontanels, excessive weight loss, decreased skin turgor, dry mucous membranes, and urine specific gravity above 1.012.

The nurse manager of the pediatric unit discovers that she's over budget on supplies. How could each nurse assigned to the unit help with cost containment? 1. Order only brand-name supplies instead of the generic equivalent. 2. Use the supply closet at work to stock personal medicine cabinets because the supplies are free. 3. Offer clients' parents the use of unit phones. 4. Use care pathways to specify care and identify daily outcomes.

4. Use care pathways to specify care and identify daily outcomes. Using care pathways ensures that clients progress toward a timely discharge and that resources are used appropriately. A longer hospital stay requires more resources, which, in turn, leads to a more costly health care bill. Generic brands are less expensive than brand name products; their use should be encouraged. Filling a personal medicine cabinet with supplies from work constitutes stealing. Offering the unit phones to parents generates higher phone bills.

Which safeguard should the nurse employ with I.V. fluid administration for an infant? 1. Administering the fluid at the slowest possible rate 2. Using a gravity infusion set 3. Using a small I.V. infusion set 4. Using an infusion pump to regulate the flow rate

4. Using an infusion pump to regulate the flow rate The nurse should use an infusion pump to regulate the flow rate. This is necessary because infants and children with compromised cardiopulmonary status are particularly vulnerable to I.V. fluid overload. Administering fluid at the slowest possible rate may not benefit the infant. Using a gravity infusion set or a small I.V. infusion set won't protect against fluid overload when I.V. administration is too rapid.

When administering an oral medication to an infant, the nurse should take which action to minimize the risk of aspiration? 1. Administering the oral medication as quickly as possible 2. Placing the medication in the infant's formula bottle 3. Keeping the infant upright with the nasal passages blocked 4. Using an oral syringe to place the medication beside the tongue

4. Using an oral syringe to place the medication beside the tongue When administering an oral medication to an infant, using an oral syringe is the best way to prevent aspiration because it allows controlled administration of a small amount of medication. Administering the medication too quickly could cause aspiration. Putting the drug in a bottle of formula isn't preferred because the infant may not take the entire dose of medication and because the contents of the bottle could interfere with drug absorption or action. Blocking the nasal passages could cause aspiration.

Which step should the nurse take first when administering a liquid medication to an infant? 1. Hold the infant securely in the crook of her arm and raise the infant's head to about a 45-degree angle. 2. Place the dropper at the corner of the infant's mouth so the drug runs into the pocket between the cheek and gum. 3. Identify the infant by checking the armband. 4. Verify the physician order.

4. Verify the physician order. The nurse should first verify the physician's order. Next, the nurse should make sure she has the right drug, dose, route, and time. She should then make sure she has the right client by checking the infant's armband. Next, the nurse should hold the infant securely in the crook of her arm and raise the infant's head to about a 45-degree angle. Then, the nurse should place the dropper at the corner of the infant's mouth so the drug runs into the pocket between the infant's cheek and gum. Doing this keeps him from spitting out the drug and reduces the risk of aspiration.

The school nurse is examining a student at an elementary school. Which findings would lead the nurse to suspect impetigo? 1. Small, red lesions on the trunk and in the skin folds 2. A discrete pink-red maculopapular rash that starts on the head and progresses down the body 3. Red spots with a blue base found on the buccal membranes 4. Vesicular lesions that ooze, forming crusts on the face and extremities

4. Vesicular lesions that ooze, forming crusts on the face and extremities Impetigo starts as papulovesicular lesions surrounded by redness. The lesions become purulent and begin to ooze, forming crusts. Impetigo occurs most often on the face and extremities. Small red lesions on the trunk and in the skin folds are characteristic of scarlet fever. A discrete pink-red maculopapular rash that starts on the face and progresses down to the trunk and extremities is characteristic of rubella (German measles). Red spots with a blue base found on the buccal membranes, known as Koplik's spots, are characteristic of measles (rubeola).

A child is receiving chemotherapy for treatment of acute lymphoblastic leukemia. During discharge preparation, which topic is most important for the nurse to discuss with the child and parents? 1. How to help the child adjust to an altered body image 2. How to increase the child's interactions with peers 3. The need to decrease the child's activity level 4. Ways to prevent infection

4. Ways to prevent infection Because overwhelming infection is the most common cause of death in clients with leukemia, preventing infection is the most important teaching topic. Although promoting adjustment to an altered body image and increasing peer interactions are important, they don't address life-threatening concerns and therefore take lower priority. The nurse should advise the parents to let the child's desire and tolerance for activity determine the child's activity level.

Which intervention provides the most accurate information about an infant's hydration status? 1. Monitoring the infant's vital signs 2. Accurately measuring intake and output 3. Monitoring serum electrolyte levels 4. Weighing the infant daily

4. Weighing the infant daily Weighing an infant daily provides the most accurate information about the infant's hydration status. Vital signs, intake and output, and electrolyte levels provide helpful information about an infant's hydration status, but they aren't as accurate as weighing daily.

A physician has prescribed oxygen as needed for an infant with congestive heart failure. In which situation should the nurse administer the oxygen to the infant? 1. During sleep 2. When changing the infant's diapers 3. When the mother is holding the infant 4. When drawing blood for electrolyte level testing

4. When drawing blood for electrolyte level testing Congestive heart failure (CHF) is the inability of the heart to pump a sufficient amount of oxygen to meet the metabolic needs of the body. Crying exhausts the limited energy supply, increases the workload of the heart , and increases the oxygen demands. Oxygen administration may be prescribed for stressful periods, especially during bouts of crying or invasive procedures. Options 1, 2, and 3 are not likely to produce crying in the infant.

A toddler is hospitalized for evaluation and management of congenital heart disease (CHD). During discharge preparation, the nurse should discuss which topic with the parents? 1. The need to withhold childhood immunizations 2. The importance of restricting the child's fat intake 3. How to perform postural drainage 4. When to administer prophylactic antibiotics

4. When to administer prophylactic antibiotics In CHD, areas of turbulent blood flow provide an optimal environment for bacterial growth. Therefore, a child with CHD is at increased risk for bacterial endocarditis, an infection of the heart valves and lining, and requires prophylactic antibiotics before dental work and invasive procedures. These children should receive all childhood immunizations. They don't require postural drainage or dietary fat restriction.

A preschooler goes into cardiac arrest. When performing cardiopulmonary resuscitation (CPR) on a child, how should the nurse deliver chest compressions? 1. With two hands 2. With two fingertips 3. With the palm of one hand 4. With the heel of one hand

4. With the heel of one hand When performing CPR on a child between ages 1 and 8, the nurse should use the heel of one hand to compress the chest 1" to 1½" (2.5 to 3.75 cm). The use of two hands (one placed on top of the other) is appropriate for adult CPR but can injure a child. The use of two fingertips is appropriate for infant CPR but can't compress the chest sufficiently on an older child. The palm is never used for chest compressions in CPR.

A 4-month-old infant is taken to the pediatrician by his parents because they're concerned about his frequent respiratory infections, poor feeding habits, frequent vomiting, and colic. The physician notes that the baby has failed to gain expected weight and recommends that the baby have a sweat test performed to detect possible cystic fibrosis. To prepare the parents for the test, the nurse should explain that: 1. the baby will need to fast before the test. 2. a sample of blood will be necessary. 3. a low-sodium diet is necessary for 24 hours before the test. 4. a low-intensity, painless electrical current is applied to the skin.

4. a low-intensity, painless electrical current is applied to the skin. Because cystic fibrosis clients have elevated levels of sodium and chloride in their sweat, a sweat test is performed to confirm this disorder. After pilocarpine (a cholinergic medication that induces sweating) is applied to a gauze pad and placed on the arm, a low-intensity, painless electrical current is applied for several minutes. The arm is then washed off, and a filter paper is placed over the site with forceps to collect the sweat. Elevated levels of sodium and chloride are diagnostic of cystic fibrosis. No fasting is necessary before this test and no blood sample is required. A low-sodium diet isn't required before the test.

When evaluating a severely depressed adolescent, the nurse knows that one indicator of a high risk for suicide is: 1. depression. 2. excessive sleepiness. 3. a history of cocaine use. 4. a preoccupation with death.

4. a preoccupation with death. An adolescent who demonstrates a preoccupation with death (such as by talking about death frequently) should be considered at high risk for suicide. Although depression, excessive sleepiness, and a history of cocaine use may occur in suicidal adolescents, they also occur in adolescents who aren't suicidal.

A child experiences nausea and vomiting after receiving cancer chemotherapy drugs. To help prevent these problems from recurring, the nurse should: 1. provide a high-fiber diet before the next chemotherapy session. 2. administer allopurinol (Zyloprim) 2 hours before the next chemotherapy session. 3. encourage increased fluid intake before the next chemotherapy session. 4. administer an antiemetic 30 to 60 minutes before the next chemotherapy session.

4. administer an antiemetic 30 to 60 minutes before the next chemotherapy session. Antiemetics counteract nausea most effectively when given before administration of an agent that causes nausea. Antiemetics also work better when given continuously rather than as needed. A high-fiber diet or allopurinol wouldn't prevent or reduce nausea and vomiting. Increasing fluid intake before the next chemotherapy session would only worsen the nausea and could cause more vomiting.

A 2-year-old child is brought to the emergency department with suspected croup. The child appears frightened and cries as the nurse approaches him. The nurse needs to assess the child's breath sounds. The best way to approach the 2-year-old child is to: 1. expose the child's chest quickly and auscultate breath sounds as quickly and efficiently as possible. 2. ask the mother to wait briefly outside until the assessment is over. 3. tell the child the nurse is going to listen to his chest with the stethoscope. 4. allow the child to handle the stethoscope before listening to his lungs.

4. allow the child to handle the stethoscope before listening to his lungs. Toddlers are naturally curious about their environment and letting them handle minor equipment is distracting and helps them gain trust with the nurse. The nurse should only expose one area at a time during assessment and should approach the child slowly and unhurriedly. The caregiver should be encouraged to hold and console her child and to comfort the child with objects with which he's familiar and to child should be given limited choices to allow autonomy such as, "Do you want me to listen first to the front of your chest or your back?"

A girl, age 15, is brought to the pediatrician's office by her mother, who's concerned about her daughter's dramatic weight loss over the past 2 months. The nurse suspects that the child has anorexia nervosa. Besides weight loss, nursing assessment of this client is likely to reveal: 1. insomnia. 2. dysphagia. 3. diarrhea. 4. amenorrhea.

4. amenorrhea. Amenorrhea is common in girls and women with anorexia nervosa; researchers don't know whether it results from starvation or an underlying metabolic disturbance. Insomnia isn't associated with anorexia nervosa. Clients with anorexia nervosa are capable of eating and rarely have dysphagia (difficulty swallowing). Anorexia nervosa is more likely to cause constipation than diarrhea because limited oral intake decreases GI motility.

A child is diagnosed with Wilms' tumor. During assessment, the nurse expects to detect: 1. gross hematuria. 2. dysuria. 3. nausea and vomiting. 4. an abdominal mass.

4. an abdominal mass. The most common sign of Wilms' tumor is a painless, palpable abdominal mass, sometimes accompanied by an increase in abdominal girth. Gross hematuria is uncommon, although microscopic hematuria may be present. Dysuria isn't associated with Wilms' tumor. Nausea and vomiting are rare in children with Wilms' tumor.

The nurse is assessing an 8-month-old child for signs of neurologic deficit and increased intracranial pressure (ICP). These signs would include: 1. a depressed fontanel. 2. slurred speech. 3. tachycardia. 4. an altered level of consciousness.

4. an altered level of consciousness. One sign of neurologic deficit in an 8-month-old child includes a decreased or altered level of consciousness. The fontanel would bulge if he had increased ICP. The child isn't able to speak at this age, but a change in cry may be noted. Bradycardia — not tachycardia — is a sign of increased ICP.

The physician prescribes digoxin (Lanoxin) elixir for a toddler with heart failure. Immediately before administering this drug, the nurse must check the toddler's: 1. serum sodium level. 2. urine output. 3. weight. 4. apical pulse.

4. apical pulse. Because digoxin may reduce the heart rate and heart failure may cause a pulse deficit, the nurse should measure the toddler's apical pulse before administering the drug to prevent further slowing of the heart rate. The serum sodium level doesn't affect digoxin's action. For a child with heart failure, the nurse should check urine output and measure weight regularly, but not necessarily just before digoxin administration.

A child, age 8, complains of leg pain shortly after being admitted with a fractured tibia sustained in a fall. The most effective way for the nurse to assess the severity of the pain is to: 1. ask the child what makes the leg feel better. 2. ask the child what the pain feels like. 3. ask the child what makes the leg feel worse. 4. ask the child to rate the pain on a scale of 1 to 10.

4. ask the child to rate the pain on a scale of 1 to 10. Although all four options describe methods that would help the nurse assess the child's pain, having the child rate the pain on a scale of 1 to 10 is the most effective way to assess pain severity because it quantifies the pain objectively.

When assessing a child's cultural background, the nurse should keep in mind that: 1. cultural background usually has little bearing on a family's health practices. 2. physical characteristics mark the child as part of a particular culture. 3. heritage dictates a group's shared values. 4. behavioral patterns are passed from one generation to the next.

4. behavioral patterns are passed from one generation to the next. A family's behavioral patterns and values are passed from one generation to the next. Cultural background commonly plays a major role in determining a family's health practices. Physical characteristics don't indicate a child's culture. Although heritage plays a role in culture, it doesn't dictate a group's shared values, and its effect on culture is weaker than that of behavioral patterns.

Which of the following agents can't be administered by way of intraosseous infusion? 1. Sodium bicarbonate 2. dopamine (Intropin) 3. Calcium chloride 4. isoproterenol (Isuprel)

4. isoproterenol (Isuprel) The following agents can be safely administered by way of intraosseous infusion: sodium bicarbonate, calcium, glucose, crystalloids, colloids, blood, dopamine, epinephrine, and dobutamine. Isoproterenol, which doesn't fall into any of the above categories, can't be administered by the intraosseous route.

The nurse is interviewing a 16-year-old girl at a clinic. It's her first visit and she says that she has been exposed to herpes by her boyfriend. Initially, with primary genital or Type 2 herpes simplex, the nurse would expect the client to have: 1. dysuria and urine retention. 2. perineal ulcers and erosions. 3. bilateral inguinal lymphadenopathy. 4. burning or tingling on vulva, perineum, or vagina.

4. burning or tingling on vulva, perineum, or vagina. Burning and tingling genital discomfort is the most common initial finding. This symptom will advance to vesicular lesions rupturing into ulcerations, which then dry into a crusty erosion. The client may also experience fever, headache, malaise, myalgia, regional lymphadenopathy, and dysuria.

A nurse performs cardiopulmonary resuscitation (CPR) for 1 minute on an infant without calling for assistance. In reassessing the infant after 1 minute of CPR, the nurse finds he still isn't breathing and has no pulse. The nurse should then: 1. resume CPR beginning with breaths. 2. declare her efforts futile. 3. resume CPR beginning with chest compressions. 4. call for assistance.

4. call for assistance. After 1 minute of CPR, the nurse should call for assistance and then resume efforts. CPR shouldn't be stopped after it has been started unless the nurse is too exhausted to continue. A cycle usually ends with breaths, so the next beginning cycle after pulse check and summoning help would begin with chest compressions.

The parents of a school-age child request anticipatory guidance. When developing a care plan to address this matter, the nurse should keep in mind that this child's cognitive development is characterized by: 1. magical thinking. 2. transductive reasoning. 3. abstract thought. 4. conservation skills.

4. conservation skills. According to Piaget, a school-age child acquires cognitive operations to understand concepts related to objects, including conservation skills, classification skills, and combinational skills. Magical thinking and transductive reasoning are characteristic of the preschooler's preoperational thought. Abstract thought is characteristic of the adolescent's period of formal operations.

Tepid sponge baths effectively reduce hyperthermia in children. The best way to give such a bath is by: 1. leaving the bathroom door open to increase air movement. 2. waiting until the child has chills to give the bath. 3. adding isopropyl alcohol to the bath water. 4. continuing the bath for 20 to 30 minutes.

4. continuing the bath for 20 to 30 minutes. For cooling to be effective, the child must be bathed for 20 to 30 minutes. Leaving the bathroom door open wouldn't help cool the child. The timing of bath treatments for hyperthermia should be based on the child's temperature, not the presence of chills. Because of its neurotoxic effects, isopropyl alcohol should never be used for tepid sponge baths.

The nurse is caring for a toddler in respiratory arrest. The nurse will assist with endotracheal intubation and use an uncuffed tube because the: 1. vocal cords provide a natural seal. 2. trachea is shorter. 3. larynx is anterior and cephalad. 4. cricoid cartilage is the narrowest part of the larynx.

4. cricoid cartilage is the narrowest part of the larynx. The cricoid cartilage in the toddler is the narrowest part of larynx and provides a natural seal. This keeps the endotracheal tube in place without requiring a cuff. The vocal cords are narrower in an adult. The trachea is shorter and the larynx is anterior and cephalad, but these aren't reasons to choose an uncuffed tube.

An adolescent with ulcerative colitis hasn't responded to nutritional intervention and drug therapy. During physical assessment, the nurse expects to note: 1. jaundice. 2. decreased bowel sounds. 3. perianal lesions. 4. delayed sexual maturation.

4. delayed sexual maturation. In children with ulcerative colitis, frequent diarrhea and poor nutrient absorption from the bowel lead to malnutrition. Nausea, vomiting, and anorexia may further compromise the client's nutritional status. Malnutrition, in turn, may cause growth retardation and delayed sexual maturation. Jaundice isn't associated with ulcerative colitis. Because this disease causes increased bowel motility, bowel sounds may be hyperactive, not decreased. Perianal lesions are rare in clients with ulcerative colitis.

According to Erikson, the psychosocial task of adolescence is the development of a sense of identity. The nurse can best promote the development of a hospitalized adolescent by: 1. emphasizing the need to follow the facility regimen. 2. allowing parents and siblings to visit frequently. 3. arranging for tutoring in school work. 4. encouraging peer visitation.

4. encouraging peer visitation. Peer visitation gives the adolescent an opportunity to continue along the path toward independence and identity. Knowledge of the facility regimen prepares the adolescent for upcoming procedures but doesn't affect development. To achieve a sense of identity, the adolescent must gain independence from the family. Tutoring may help maintain a positive self-image relative to schoolwork but doesn't affect development.

The nurse is preparing for the discharge of a neonate with a cleft lip and palate. One of the nurse's major concerns is to: 1. institute prescribed antibiotic therapy. 2. administer supplemental vitamins. 3. apply a sterile dressing to the lip. 4. establish an adequate feeding pattern.

4. establish an adequate feeding pattern. Major concerns for a neonate with a cleft lip and palate relate to feeding. Establishing an adequate feeding pattern requires specific teaching and use of a special nipple. Antibiotics and vitamins aren't administered routinely to a neonate with this defect. Sterile dressings aren't indicated for this congenital disorder.

After a head injury, a child experiences enuresis, polydipsia, and weight loss. Based on these findings, the nurse should monitor closely for signs and symptoms of: 1. hypercalcemia. 2. hyperglycemia. 3. hyponatremia. 4. hypokalemia.

4. hypokalemia. Enuresis, polydipsia, and weight loss suggest diabetes insipidus, a disorder that may result from a head injury that damages the neurohypophyseal structures. Diabetes insipidus places the child at risk for fluid volume depletion and hypokalemia. It doesn't cause hypercalcemia, hyperglycemia, or hyponatremia.

In a 3-month-old infant, fluid and electrolyte imbalance can occur quickly, primarily because an infant has: 1. a lower percentage of body water than an adult. 2. a lower daily fluid requirement than an adult. 3. a more rapid respiratory rate than an adult. 4. immature kidney function.

4. immature kidney function. Because of immature kidneys, an infant's glomerular filtration and absorption are inadequate, not reaching adult levels until age 1 to 2 years. An infant has a greater percentage of body water and higher daily fluid requirements than an adult. Although the infant's respiratory rate is higher, causing insensible water loss, immature kidney function is more responsible for fluid balance in an infant.

What is the recommended treatment for scabies in a child who's under age 1? 1. lindane (Kwell) 2. tolnaftate (Tinactin) 3. thiabendazole (Mintezol) 4. permethrin (Elimite)

4. permethrin (Elimite) Permethrin is supplied in a cream. It should be massaged into the skin from the head to the soles. Although permethrin is the treatment of choice for children younger than age 1, its safety hasn't been established for clients younger than 2 months. Lindane, a treatment for scabies, isn't recommended for children younger than age 1, and it shouldn't be used on children older than age 1 if they won't be supervised. The hands and feet of a child should be covered during treatment to prevent the child from sucking the cream or lotion. Young children may be more sensitive to central nervous system toxicity from the drug. Tolnaftate is used to treat ringworm. Thiabendazole is used to treat hookworm, roundworm, threadworm, and whipworm.

When performing a physical examination on an infant, the nurse notes abnormally low-set ears. This finding is associated with: 1. otogenous tetanus. 2. tracheoesophageal fistula. 3. congenital heart defects. 4. renal anomalies.

4. renal anomalies. Normally, the top of the ear aligns with an imaginary line drawn across the inner and outer canthus of the eye. Ears set below this line are associated with renal anomalies or mental retardation. Low-set ears don't accompany otogenous tetanus, tracheoesophageal fistula, or congenital heart defects.

A child, age 5, takes theophylline orally three times a day to treat asthma. For the most accurate calculation of a safe dosage, the nurse should use: 1. the child's weight in kilograms. 2. Young's rule based on the child's age. 3. Clark's rule based on the child's weight in pounds. 4. the child's body surface area.

4. the child's body surface area. Using a child's body surface area may be the most accurate method for calculating safe drug dosages because body surface area is thought to parallel the child's organ growth and maturation and metabolic rate. Using the child's weight in kilograms, Young's rule based on the child's age, or Clark's rule based on the child's weight in pounds is likely to yield less accurate dosages.

A local elementary school has requested scoliosis screening for its students from the hospital's community outreach program. The school should be informed that: 1. these students are too young to screen; instead, older students should be screened. 2. these students are too old to screen and will no longer benefit from screening for scoliosis. 3. scoliosis screening requires sophisticated equipment and can't be done in school. 4. this is an appropriate request and arrangements will be made as soon as possible.

4. this is an appropriate request and arrangements will be made as soon as possible. Screening for scoliosis should begin at age 8 and be performed yearly thereafter. Screening for scoliosis involves inspection of the spine and use of a scoliometer, both of which can be done in a school setting.

A toddler has a temperature above 101° F (38.3° C). The physician prescribes acetaminophen (Tylenol), 120 mg suppository, to be administered rectally every 4 to 6 hours. However, the nurse should not administer this rectal medication if the child has: 1. sepsis. 2. leukocytosis. 3. anemia. 4. thrombocytopenia.

4. thrombocytopenia. A child with thrombocytopenia or neutropenia shouldn't receive rectal medication because of the increased risk of infection and bleeding that may result from tissue trauma. No contraindications exist for administering rectal medication to a child with sepsis, leukocytosis, or anemia.

The nurse is providing cardiopulmonary resuscitation (CPR) to a child, age 4. The nurse should: 1. compress the sternum with both hands at a depth of 1½" to 2" (4 to 5 cm). 2. deliver 12 breaths/minute. 3. perform only two-person CPR. 4. use the heel of one hand for sternal compressions.

4. use the heel of one hand for sternal compressions. The nurse should use the heel of one hand and compress 1" to 1½" (3 to 4 cm). The nurse should use the heels of both hands clasped together and compress the sternum 1½" to 2" for an adult. For a small child, two-person rescue may be inappropriate. For a child, the nurse should deliver 20 breaths/minute instead of 12 breaths/minute.

Erikson - school age

6 - 12 years Industry vs. inferiority Task: Developing social, physical, and learning skills Successful: Competence; ability to learn and work Unsuccessful: Sense of inferiority, difficulty learning and working

Piaget - Concrete operational stage

7 to 11 years The child is able to classify, order, and sort facts. The child moves from prelogical thought to solving concrete problems through logic. The child begins to develop abstract thinking.

Atrial septal defect (ASD)

ASD is an abnormal opening between the atria that causes an increased flow of oxygenated blood into the right side of the heart. Right atrial and ventricular enlargement occurs. Infant may be asymptomatic or may develop CHF. Signs and symptoms of decreased cardiac output may be present.

Precipitants Triggering an Asthma Attack

Allergens Outdoor: trees, shrubs, weeds, grasses, molds, pollen, air pollution, spores Indoor: dust, dust mites, mold, cockroach antigen Irritants: tobacco smoke, wood smoke, odors, sprays Exposure to occupational irritants Exercise Cold air Changes in weather or temperature Environmental change: moving to a new home, starting a new school Colds and infections Animals: cats, dogs, rodents, horses Medications: aspirin, nonsteroidal anti-inflammatory drugs, antibiotics, β-blockers Strong emotions: fear, anger, laughing, crying Conditions: gastroesophageal reflux disease, tracheoesophageal fistula Food additives: sulfite preservatives Foods: nuts, milk and other dairy products Endocrine factors: menses, pregnancy, thyroid disease

Piaget - sensorimotor stage

Birth to 2 years Development proceeds from reflex activity to imagining and solving problems through the senses and movement. The infant or toddler learns about reality and how it works. The infant or toddler does not recognize that objects continue to be in existence, even if out of the visual field.

Infant skills 10 - 11 months

Can change from prone to sitting position Walks while holding on to furniture Stands securely Entertains self for periods of time

Treatment of VSD

Closure during cardiac catheterization may be possible. Open repair may be done with cardiopulmonary bypass.

Infant skills 6 - 7 months

Creeps Sits with support Imitates Exhibits fear of strangers Holds arms out Frequent mood swings Waves "bye-bye"

Infant skills 4 - 5 months

Grasps objects Switches objects from hands Rolls over for the first time Enjoys social interaction Begins to show memory Aware of unfamiliar surroundings

Changes in vital signs from intrauterine to extrauterine life

Heart rate decreases b/c metabolic demands decrease Blood pressure increases as heart develops

Treatment for patent ductus arteriosus

Indomethacin (Indocin), a prostaglandin inhibitor, may be administered to close a patent ductus in premature infants and some newborns. The defect may be closed during cardiac catheterization , or the defect may require surgical management.

Signs & symptoms of Tetralogy of Fallot

Infants: may be acutely cyanotic at birth or may have mild cyanosis that progresses over the first year of life as the pulmonic stenosis worsens. A characteristic murmur is present. Acute episodes of cyanosis and hypoxia, called blue spells or tet spells, occur when the infant's oxygen requirements exceed the blood supply, such as during periods of crying, feeding, or defecating. Children: With increasing cyanosis, squatting, clubbing of the fingers, and poor growth may occur. Squatting is a compensatory mechanism to facilitate increased return of blood flow to the heart for oxygenation.

Signs and symptoms of RSV

Initial Manifestations: Rhinorrhea; Eye or ear drainage; Pharyngitis; Coughing; Sneezing; Wheezing; Intermittent fever Manifestations as Disease Progresses: Increased coughing and wheezing; Signs of air hunger; Tachypnea and retractions; Periods of cyanosis Manifestations in Severe Illness: Tachypnea more than 70 breaths/ min; Decreased breath sounds and poor air exchange; Listlessness; Apneic episodes

Treatment for coarctation of aorta

Management of the defect may be done via balloon angioplasty in children; restenosis can occur. Surgical management a. Mechanical ventilation and medications to improve cardiac output are often necessary before surgery. b. Resection of the coarcted portion with end-to-end anastomosis of the aorta or enlargement of the constricted section using a graft may be required. c. Because the defect is outside the heart, cardiopulmonary bypass is not required, and a thoracotomy incision is used. With coarctation of the aorta, the blood pressure is higher in the upper extremities than the lower extremities. Additionally, bounding pulses in the arms, weak or absent femoral pulses, and cool lower extremities may be present.

Interventions for CHF (peds)

Monitor vital signs Monitor I&O Daily weights Monitor for edema Provide rest Administer O2 Feed when hungry and soon after awakening, conserving energy and oxygen supply. Provide small, frequent feedings, conserving energy and oxygen supply. Administer medications as prescribed. (Digoxin, diuretics, antihypertensives)

Infant nutrition

Nutrition a. The infant may breast-feed or bottle-feed (with iron-fortified formula), depending on the mother's choice; however, human milk is the preferred form of nutrition for all infants, especially during the first 6 months. 6 b. All infants should receive daily vitamin D supplementation (200 IU) starting in the first 2 months of life to prevent rickets and vitamin D deficiency. c. Iron stores from birth are depleted by 4 months of age; if the infant is being only breast-fed, iron supplementation usually with iron-fortified cereal is needed. d. Whole milk, low-fat milk, skim milk, other animal milk, or imitation milk should not be given to infants as a primary source of nutrition because these food sources lack the necessary components needed for growth and have limited digestibility. e. Fluoride supplementation may be needed at about 6 months of age, depending on the infant's intake of fluoridated tap water. f. Solid foods (strained, pureed, or finely mashed) are introduced at about 5 to 6 months of age; introduce solid foods one at a time, usually at intervals of 4 to 5 days, to identify food allergens. g. Sequence of the introduction of solid foods varies depending on physician's preference and usually is as follows: iron-fortified rice cereal, fruits, vegetables, then meats. h. At 12 months of age, eggs can be given (introduce egg whites in small quantities to detect an allergy); cheese may be used as a substitute for meat. i. Avoid solid foods that place the infant at risk for choking, such as nuts, foods with seeds, raisins, popcorn, grapes, and hot dog pieces. j. Avoid microwaving baby bottles and baby food. k. Never mix food or medications with formula. l. Avoid adding honey to formula, water, or other fluid to prevent botulism. m. Offer fruit juice from a cup (12 to 13 months or at a prescribed age) rather than a bottle to prevent nursing (bottle-mouth) caries; fruit juice is limited because of its high sugar content.

Patent ductus arteriosus

Patent ductus arteriosus is failure of the fetal ductus arteriosus (artery connecting the aorta and the pulmonary artery) to close within the first weeks of life. A characteristic machinery-like murmur is present. An infant may be asymptomatic or may show signs of CHF. A widened pulse pressure and bounding pulses are present. Signs and symptoms of decreased cardiac output may be present.

Cardiac changes in transition to extrauterine life

SVR increases Foramen ovale closes Ductus arteriosis constricts - closes within 10-15 hours of birth Left ventricle enlarges due to increased pressure

Infant skills 8 - 9 months

Sits steadily unsupported Crawls May stand while holding on Begins to stand without help

Infant skills 2 - 3 months

Smiles Turns head side to side Cries Follows objects Holds head in midline

1 year old vital signs

Temperature: Axillary, 97 ° to 99 ° F Apical heart rate: 90 to 130 beats/ min Respirations: 20 to 40 breaths/ min Blood pressure: 90/ 56 mm Hg

Preschooler vital signs

Temperature: Axillary, 97.5 ° to 98.6 ° F Apical heart rate: 70 to 110 beats/ min Respirations: 16 to 22 breaths/ min Blood pressure: Average, 95/ 57 mm Hg

Toddler vital signs

Temperature: Axillary, 97.5 ° to 98.6 ° F Apical heart rate: 80 to 120 beats/ min Respirations: 20 to 30 breaths/ min Blood pressure: Average, 92/ 55 mm Hg

Newborn vital signs

Temperature: Axillary, 97.9 ° to 98 ° F Apical heart rate: 120 to 140 beats/ min Respirations: 30 to 60 (average 40) breaths/ min Blood pressure: 73/ 55 mm Hg

Adolescent vital signs

Temperature: Oral, 97.5 ° to 98.6 ° F Apical heart rate: 55 to 90 beats/ min Respirations: 12 to 20 breaths/ min Blood pressure: Average, 121/ 70 mm Hg

school age child vital signs

Temperature: Oral, 97.5 ° to 98.6 ° F Apical heart rate: 60 to 100 beats/ min Respirations: 18 to 20 breaths/ min Blood pressure: Average, 107/ 64 mm Hg

CONGESTIVE HEART FAILURE (Peds)

The inability of the heart to pump a sufficient amount of oxygen to meet the metabolic needs of the body. In infants and children, inadequate cardiac output most commonly is caused by congenital heart defects that produce an excessive volume or pressure load on the myocardium - a combination of left-sided and right-sided heart failure is usually present. The goals of treatment are to improve cardiac function, remove accumulated fluid and sodium, decrease cardiac demands, improve tissue oxygenation, and decrease oxygen consumption.

An infant who weighs 7.5 kg is to receive ampicillin (Omnipen) 25 mg/kg I.V. every 6 hours. How many milligrams should the nurse administer per dose?

The nurse should calculate the correct dose using the following equation: 25 mg/kg × 7.5 kg = 187.5 mg

A child with sickle cell anemia is being treated for a crisis. The physician orders morphine sulfate, 2 mg I.V. The concentration of the vial is 10 mg/1 ml of solution. How many milliliters of solution should the nurse administer?

The nurse should calculate the volume to be given using this equation: 2 mg/X ml = 10 mg/1 ml 10 X = 2 X = 0.2 ml

The nurse is preparing a dose of amoxicillin for a 3-year-old child with acute otitis media. The child weighs 33 lb. The dosage prescribed is 50 mg/kg/day in divided doses every 8 hours. The concentration of the drug is 250 mg/5 ml. How many milliliters should the nurse administer?

To calculate the child's weight in kilograms, the nurse should use the following formula: 2.2 lb/1 kg = 33 lb/X kg X = 33 ÷ 2.2 X = 15 kg Next, the nurse should calculate the daily dosage for the child: 50 mg/kg/day × 15 kg = 750 mg/day To determine divided daily dosage, the nurse should know that "every 8 hours" means 3 times per day. So, she should perform that calculation in this way: Total daily dosage ÷ 3 times per day = Divided daily dosage 750 mg/day ÷ 3 = 250 mg The drug's concentration is 250 mg/5 ml, so nurse should administer 5 ml.

A 3-year-old child is to receive 500 ml of dextrose 5% in normal saline solution over 8 hours. At what rate (in ml/hr) should the nurse set the infusion pump?

To calculate the rate per hour for the infusion, the nurse should divide 500 ml by 8 hours: 500 ml ÷ 8 hours = 62.5 ml/hr.

A 44-lb preschooler is being treated for inflammation. The physician orders 0.2 mg/kg/day of dexamethasone (Decadron) by mouth to be administered every 6 hours. The elixir comes in a strength of 0.5 mg/5 ml. How many teaspoons of dexamethasone should the nurse give this client per dose?

To perform this dosage calculation, the nurse should first convert the child's weight from pounds to kilograms: 44 lb ÷ 2.2 lb/kg = 20 kg Then she should calculate the total daily dose for the child: 20 kg × 0.2 mg/kg/day = 4 mg Next, the nurse should calculate the amount to be given at each dose: 4 mg ÷ 4 doses = 1 mg/dose The available elixir contains 0.5 mg of drug per 5 ml (which is equal to 1 teaspoon). Therefore, to give 1 mg of the drug, the nurse should administer 2 teaspoons (10 ml) to the child for each dose.

The nurse is preparing to administer I.V. methylprednisolone sodium succinate (Solu-Medrol) to a child who weighs 42 lb. The order is for 0.03 mg/kg I.V. daily. How many milligrams should the nurse prepare?

To perform this dosage calculation, the nurse should first convert the child's weight to kilograms: 44 lb ÷ 2.2 kg/lb = 20 kg Then she should use this formula to determine the dose: 20 kg × 0.03 mg/kg = X mg X = 0.6 mg

The physician orders an I.V. infusion of dextrose 5% in quarter-normal saline solution to be infused at 7 ml/kg/hr for a 10-month-old infant. The infant weighs 22 lb. How many ml/hr should the nurse infuse of the ordered solution?

To perform this dosage calculation, the nurse should first convert the infant's weight to kilograms: 2.2 lb/kg = 22 lb/X kg X = 22 ÷ 2.2 X = 10 kg Next, she should multiply the infant's weight by the ordered rate: 10 kg × 7 ml/kg/hr = 70 ml/hr

Erikson - Infancy

Trust vs. Mistrust Task: Attachement to the mother Successful: Trust in persons, faith and hope about the environment and future Unsuccessful: General difficulties relating to persons effectively, suspicion; trust-fear conflict; fear of the future

Ventricular septal defect (VSD)

VSD is an abnormal opening between the right and left ventricles. Many VSDs close spontaneously during the first year of life in children having small or moderate defects. A characteristic murmur is present. Signs and symptoms of CHF are commonly present. Signs and symptoms of decreased cardiac output may be present.

14 - 15 months skills

Walks alone Can crawl up stairs Shows emotions such as anger and affection Will explore away from mother in familiar surroundings

Infant skills 12 - 13 months

Walks with one hand held Can take a few steps without falling Can drink from a cup

A 4-year-old child is ordered to receive 25 ml/hour of I.V. solution. The nurse is using a pediatric microdrip chamber to administer the medication. For how many drops per minute should the microdrip chamber be set?

When using a pediatric microdrip chamber, the number of milliliters per hour equals the number of drops per minute. If 25 ml/hour is ordered, the I.V. should infuse at 25 drops/minute.

safety - adolescent

a. Adolescents are risk takers. b. Adolescents have a natural urge to experiment and to be independent. c. Reinforce instructions about the dangers related to cigarette smoking, caffeine ingestion, alcohol, and drugs. d. Help adolescents recognize that they have choices when difficult or potentially dangerous situations arise. e. Ensure that the adolescent uses a seat belt. f. Instruct adolescents in the consequences of injuries that motor vehicle accidents can cause. g. Instruct adolescents in water safety and emphasize that they should enter the water feet first as opposed to diving, especially when the depth of the water is unknown. h. Instruct adolescents about the dangers associated with guns, violence, and gangs. i. Instruct adolescents about the complications associated with body piercing, tattooing, and sun tanning.

play - adolescent

a. Games and athletic activities are the most common forms of play. b. Competition and strict rules are important. c. Adolescents enjoy activities such as sports, videos, movies, reading, parties, dancing, hobbies, computer games, music, communicating via the Internet, and experimenting, such as with makeup and hairstyles. d. Friends are important, and adolescents like to gather in small groups.

Physical development - school age

a. Girls usually grow faster than boys. b. Growth is about 2 inches per year between ages 6 and 12. c. Height ranges from 45 inches at age 6 to 59 inches at age 12. d. School-age children gain weight at a rate of about 4 ½ to 6 ½ pounds per year. e. Average weight is 46 pounds at age 6 and 88 pounds at age 12. f. The first permanent (secondary) teeth erupt around age 6, and deciduous teeth are lost gradually. g. Regular dentist visits are necessary, and the school-age child needs to be supervised with brushing and flossing teeth; fluoride supplements may be necessary if the water is not fluoridated. h. For school-age children with primary and permanent dentition, the best toothbrush is one with soft nylon bristles and an overall length of about 6 inches. i. Sleep requirements range from 10 to 12 hours a night.

skills development - adolescent

a. Gross and fine motor skills are well developed. b. Strength and endurance increase.

Physical development - Toddler

a. Height and weight increase in phases, reflecting growth spurts and lags. b. Head circumference increases about 1 inch between ages 1 and 2; thereafter head circumference increases about ½ inch per year until age 5. c. Anterior fontanel closes between ages 12 and 18 months. d. Weight gain is slower than in infancy; by age 2, the average weight is 22 to 27 pounds. e. Normal height changes include a growth of about 3 inches per year; the average height of the toddler is 34 inches at age 2 years. f. Lordosis (pot belly) is noted. g. The toddler should see a dentist soon after the first teeth erupt, usually around 1 year of age and oral hygiene measures should be instituted ; regular dental care is essential, and the toddler will require assistance with brushing and flossing of teeth (fluoride supplements may be necessary if the water is not fluoridated). h. A toddler should never be allowed to fall asleep with a bottle containing milk, juice, soda pop, sweetened water, or any other sweet liquid because of the risk of nursing (bottle-mouth) caries. i. Typically, the toddler sleeps through the night and has one daytime nap; the daytime nap is normally discontinued at about age 3. j. A consistent bedtime ritual helps prepare the toddler for sleep. k. Security objects at bedtime may assist in sleep.

Physical developmental characteristics of infant

a. Height increases by ¾ inch per month. b. Weight is doubled at 5 to 6 months and tripled at 12 months. c. At birth, head circumference is 33 to 35 cm (13.2 to 14 inches), approximately 2 to 3 cm more than chest circumference. d. By 1 to 2 years of age, head circumference and chest circumference are equal. e. Anterior fontanel (soft and flat in a normal infant) closes by 12 to 18 months of age. f. Posterior fontanel (soft and flat in a normal infant) closes by the end of the second month. g. Infant has 10 upper and 10 lower deciduous teeth by 2 ½ years of age. h. Lower central incisors are present by 6 to 8 months. i. Sleep patterns vary among infants; generally, by 3 to 4 months of age, most infants have developed a nocturnal pattern of sleep that lasts 9 to 11 hours.

Nutrition for the preschooler

a. Nutritional needs are similar to those required for the toddler although the daily amounts of minerals, vitamins, and protein may increase with age. b. The MyPyramid for Kids food guide is appropriate for preschoolers (see www.mypyramid.gov ). c. The preschooler exhibits food fads and certain taste preferences and may exhibit finicky eating. d. By 5 years old, the child tends to focus on social aspects of eating, table conversations, manners, and willingness to try new foods.

Infant safety

a. Parents must baby-proof the home. b. Guard the infant when on a bed or changing table. c. Use gates to protect the infant from stairs. d. Be sure that bath water is not hot; do not leave the infant unattended in the bath. e. Do not hold the infant while drinking or working near hot liquids or items such as a stove. f. Cool vaporizers should be used if needed, instead of steam, to prevent burn injuries. g. Avoid offering food that is round and similar to the size of the airway to prevent choking. h. Be sure toys have no small pieces. i. Toys or mobiles hanging over the crib should be well out of reach to prevent strangulation. j. Avoid placing large toys in the crib because an older infant may use them as steps to climb. k. Cribs should be positioned away from curtains and blind cords. l. Cover electrical outlets. m. Remove hazardous objects from low, reachable places. n. Remove chemicals such as cleaning or other household products, medications, poisons, and plants from the infant's reach. o. Keep the Poison Control Center number available.

safety - preschooler

a. Preschoolers are active and inquisitive. b. Because of their magical thinking, they may believe that daring feats seen in cartoons are possible and may attempt them. c. The preschooler can learn simple safety practices because they can follow simple verbal directions and their attention span is longer. d. Teach the preschooler basic safety rules to ensure safety when playing in a playground such as near swings and ladders. e. Teach the preschooler never to play with matches or lighters. f. The preschooler should be taught what to do in the event of a fire or if clothes catch fire; fire drills should be practiced with the preschooler. g. Guns should be stored unloaded and secured under lock and key (ammunition should be locked in a separate place). h. Teach the preschooler his or her full name, address, parents' names, and telephone number. i. Teach the preschooler how to dial 911 in an emergency situation. j. Keep the Poison Control Center number available.

Physical development - adolescent

a. Puberty is the maturational, hormonal, and growth process that occurs when the reproductive organs begin to function and the secondary sex characteristics develop. b. Body mass increases to adult size. c. Sebaceous and sweat glands become active and fully functional. d. Body hair distribution occurs. e. Increases in height, weight, breast development, and pelvic girth occur in girls. f. Menstrual periods occur about 2 ½ years after the onset of puberty. g. In boys, increases in height, weight, muscle mass, and penis and testicle size occur. h. The voice deepens in boys. i. Normal weight gain during puberty: Girls gain 15 to 55 pounds; boys gain 15 to 65 pounds. j. Careful brushing and care of the teeth are important, and many adolescents need to wear braces. k. Sleep patterns include a tendency to stay up late; therefore, in an attempt to catch up on missed sleep , adolescents sleep late whenever possible; an overall average of 8 hours per night is recommended.

skills development - school age

a. School-age children exhibit refinement of fine motor skills. b. Development of gross motor skills continues. c. Strength and endurance increase.

Nutrition - school age

a. School-age children will have increased growth needs as they approach adolescence. b. Children require a balanced diet from foods in the MyPyramid Food Guide; healthy snacks should continue to be emphasized to prevent childhood obesity. c. Children still may be picky eaters but are usually willing to try new foods.

Infant play

a. Solitary b. Birth to 3 months: Verbal, visual, and tactile stimuli c. 4 to 6 months: Initiation of actions and recognition of new experiences d. 6 to 12 months: Awareness of self, imitation, repetition of pleasurable actions e. Enjoyment of soft stuffed animals, crib mobiles with contrasting colors, squeeze toys, rattles, musical toys, water toys during the bath , large picture books, and push toys after he or she begins to walk

adolescent nutrition

a. Teaching about the MyPyramid Food Guide is important. b. Adolescents typically eat whenever they have a break in activities. c. Calcium, zinc, iron, folic acid, and protein are especially important nutritional needs. d. Adolescents tend to snack on empty calories and the importance of adequate and healthy nutrition needs to be stressed. e. Body image is important.

Toddler nutrition

a. The MyPyramid for Kids food guide provides dietary guidelines and applies to children as young as 2 years of age. b. The toddler should average an intake of two to three servings of milk daily (24 to 30 oz) to ensure an adequate amount of calcium and phosphorus (low-fat milk may be given after 2 years of age). c. Trans-fatty acids and saturated fats need to be restricted; otherwise fat restriction is not appropriate for a toddler (mothers should be taught about the types of food that contain fat that should be selected). d. Iron-fortified cereal and a high-iron diet, adequate amounts of calcium and vitamin D, and vitamin C (4 to 6 oz of juice daily) are essential components for the toddler's diet. e. Most toddlers prefer to feed themselves. f. The toddler generally does best by eating several small nutritious meals each day rather than three large meals. g. Offer a limited number of foods at any one time. h. Offer finger foods and avoid concentrated sweets and empty calories. i. The toddler is at risk for aspiration of small foods that are not chewed easily, such as nuts, foods with seeds, raisins, popcorn, grapes, and hot dog pieces. j. Physiological anorexia may occur and is normal because of the alternating stages of fast and slow growth. k. Sit the toddler in a high chair at the family table for meals. l. Allow sufficient time to eat, but remove food when the toddler begins to play with it. m. The toddler drinks well from a cup held with both hands. n. Avoid using food as a reward or punishment.

Play - Toddler

a. The major socializing mechanism is parallel play, and therapeutic play can begin at this age. b. The toddler has a short attention span, causing the toddler to change toys often. c. The toddler explores body parts of self and others. d. Typical toys include push-pull toys, blocks, sand, finger paints and bubbles, large balls, crayons, trucks and dolls, containers, Play -Doh, toy telephones, cloth books, and wooden puzzles.

Physical development - Preschooler

a. The preschooler grows 2 ½ to 3 inches per year. b. Average height is 37 inches at age 3, 40 ½ inches at age 4, and 43 inches at age 5. c. The preschooler gains approximately 5 pounds per year; average weight is 35 to 40 pounds at age 5. d. The preschooler requires about 12 hours of sleep each day. e. A security object and a nightlight help with sleeping. f. At the beginning of the preschool period, the eruption of the deciduous (primary) teeth is complete. g. Regular dental care is essential, and the preschooler may require assistance with brushing and flossing of teeth; fluoride supplements may be necessary if the water is not fluoridated.

Skills - preschooler

a. The preschooler has good posture. b. The child develops fine motor coordination. c. The child can hop, skip, and run more smoothly. d. Athletic abilities begin to develop. e. The preschooler demonstrates increased skills in balancing. f. The child alternates feet when climbing stairs. g. The child can tie shoelaces by age 6. h. The child may talk continuously and ask many "why" questions. i. Vocabulary increases to about 900 words by age 3 and to 2100 words by age 5. j. By age 3, the preschooler usually talks in three- or four-word sentences and speaks in short phrases. k. By age 4, the preschooler speaks five- or six-word sentences and, by age 5, speaks in longer sentences that contain all parts of speech. l. The child can be understood readily by others and can understand clearly what others are saying.

play - preschooler

a. The preschooler is cooperative. b. The preschooler has imaginary playmates. c. The child likes to build and create things, and play is simple and imaginative. d. The child understands sharing and is able to interact with peers. e. The child requires regular socialization with mates of similar age. f. play activities include a large space for running and jumping. g. The preschooler likes dress-up clothes, paints, paper, and crayons for creative expression. h. Swimming and sports aid in growth development. i. Puzzles and toys aid with fine motor development.

safety - school age

a. The school-age child experiences less fear in play activities and frequently imitates real life by using tools and household items. b. Major causes of injuries include bicycles, skateboards , and team sports as the child increases in motor abilities and independence. c. Children should always wear a helmet when riding a bike or using in-line skates or skateboards. d. Teach the child water safety rules. e. Instruct the child to avoid teasing or playing roughly with animals. f. Teach the child never to play with matches or lighters. g. The child should be taught what to do in the event of a fire or if clothes catch fire; fire drills should be practiced with the child. h. Guns should be stored unloaded and secured under lock and key (ammunition should be locked in a separate place). i. Teach the child traffic safety rules. j. Teach the child how to dial 911 in an emergency situation. k. Keep the Poison Control Center number available.

Toddler - skills

a. The toddler begins to walk with one hand held by age 12 to 13 months. b. The toddler runs by age 2 years and walks backward and hops on one foot by age 3 years. c. The toddler usually cannot alternate feet when climbing stairs. d. The toddler begins to master fine motor skills for building, undressing, and drawing lines. e. The young toddler often uses "no" even when he or she means "yes" to assert independence. f. The toddler begins to use short sentences and has a vocabulary of about 300 words by age 2.

Safety - Toddler

a. Use back burners on the stove to prepare a meal; turn pot handles inward and toward the middle of the stove. b. Keep dangling cords from small appliances away from the toddler. c. Place inaccessible locks on windows and doors, and keep furniture away from windows. d. Secure screens on all windows. e. Place safety gates at stairways. f. Do not allow the toddler to sleep or play in an upper bunk bed. g. Never leave the toddler alone near a bathtub, pail of water, swimming pool, or any other body of water. h. Keep toilet lids closed. i. Keep all medicines, poisons, household plants, and toxic products in high areas and locked out of reach. j. Keep the Poison Control Center number available.

play - school age

a. play is more competitive. b. Rules and rituals are important aspects of play and games. c. The school-age child enjoys drawing, collecting items, dolls, pets, guessing games, board games , listening to the radio, TV, reading, watching videos or DVDs, and computer games. d. The child participates in team sports. e. The child may participate in secret clubs, group peer activities, and scout organizations.

Signs of Digoxin toxicity (Peds)

anorexia, poor feeding, nausea, vomiting, bradycardia, and dysrhythmias

Tetralogy of Fallot

includes four defects— VSD, pulmonary stenosis, overriding aorta, and right ventricular hypertrophy. If pulmonary vascular resistance is higher than systemic resistance, the shunt is from right to left; if systemic resistance is higher than pulmonary resistance, the shunt is left to right.

Digoxin

increases the activity of the heart and the force of the contraction, decreasing workload of the heart Assess apical heart rate for 1 minute before administration. Hold digoxin if the apical pulse is less than 90 to 110 beats/ min in infants and young children and less than 70 beats/ min in older children, as prescribed. Be aware that infants rarely receive more than 1 mL (50 mcg or 0.05 mg) of digoxin in one dose. Monitor digoxin levels and for signs of digoxin toxicity.

Coarctation of the aorta

localized narrowing near the insertion of the ductus arteriosus Blood pressure is higher in the upper extremities than the lower extremities; bounding pulses in the arms, weak or absent femoral pulses, and cool lower extremities may be present. Signs of CHF may occur in infants. Signs and symptoms of decreased cardiac output may be present. Children may experience headaches, dizziness , fainting, and epistaxis resulting from hypertension.

Tonsillitis

refers to inflammation and infection of the tonsils, which is lymphoid tissue located in the pharynx


Ensembles d'études connexes

Spanish I Review-tener expressions

View Set

General Psychology, Chaps 12 & 13

View Set

Module 3: Go! Excel 2010 Introductory Chapter 5

View Set

Culture and Communication - Speech/Communications

View Set

Bullet points: U.S. Constitutional Amendments 11-27

View Set